Sie sind auf Seite 1von 116

Hypertension

I. Chest Pain (Top of Document)


1. Explain why the history is the most important diagnostic information in the
approach to a patient with chest pain.
EChing, Noble on-line, Ch16, P163
Although the diseases that present w/ chest pain are well described, precise diagnosis is
complicated by cultural differences in disease presentation, physiology of pain perception in the
chest area, and coexistence of other disease. Any evaluation of chest pain should consider
anxiety disorders, especially panic attacks, because of their frequency in primary care settings.
Also ask about substance abuse, particularly cocaine, because cocaine use may cause angina or
MI.
EChing, CMDT, P312
Chest pain and other forms of discomfort are common symptoms that can occur as a result of
pulmonary, pleural, or musculoskeletal disease, esophageal or other GI disorders, cervicothoracic
nerve root irritation, or anxiety states, as well as many cardiovascular diseases.
Aprilt Noble 2nd ed. p. 220
Several characteristics in the history will aid in distinguishing between ischemic chest pain and
other causes of chest discomfort. Determining quality, location, radiation, precipitating factors
and frequency are important.
Anonymous Goroll
The prevalence of angiographically confirmed coronary heart disease (CHD)
approaches 90% in patients with a "classic" angina history. Among the features of the chest pain
description with the greatest discriminant value are its timing in relation to precipitating and
relieving factors. Quality, location, radiation, and intensity of pain are notoriously non-specific.
Other important aspects of the history are: age and gender, cardiac risk factors (HTN, diabetes,
cigarettes), and family history of premature CHD.
Anonymous Noble pg 163-168
A careful history and physical exam are essential and can help the physician prioritize disease
risks. The first task is to r/o potentially life-threatening causes: MI, unstable angina, dissecting
aneurysm, PE, pneumothorax, or the depressed pt. with suicidal intent. Because chest pain can
be broad, a good history will narrow your differential diagnosis (angina, pericarditis, MVP,
esophageal spasm, GERD, anxiety, panic disorder, etc).
Anonymous Currents p 312/ Noble p 163
A) Chest pain and other forms of discomfort are common symptoms that can occur as a result of
pulmonary, pleural or musculoskeletal disease, esophageal or other gastrointestinal diseases,
cervicothoracic nerve root irritation, or anxiety states as well as many cardiovascular diseases. It
is important to take a thorough history to attempt to differentiate cardiac pain from all the other
possibilities of chest pain.

B) The history should include risk factors for atherosclerotic disease:


1. History of CAD (angina or prior MI)
2. Gender (males are more likely to have heart disease younger)
3. Hypertension
4. Diabetes
5. High LDL, high cholesterol, or low HDL
6. Smoking
7. Family history
8. Obesity
9. Substance abuse (especially cocaine)

2. Specify several symptoms that would exclude angina from consideration in a


person with chest pain.
EChing, Noble on-line, Ch 63, p547
Angina is NEVER sharp, stabbing, prickly, spasmodic, or pleuritic.
Aprilt Noble 2nd ed. p. 220
It is rarely described as a pain, and patients sometimes correct the physician who refers to it as
such.
* It is NEVER sharp, stabbing, prickly, spasmodic, or pleuritic.
Anonymous I used Goroll “evaluation of chest pain” but I have an old edition so the page
numbers don’t match up. .
Angina pectoris refers to chest pain caused by impairment of myocardial blood supply due to
CAD (coronary artery disease). (Can also be caused by aortic stenosis)
Is not angina if:
• Lasts longer than 20 minutes suggests acute coronary insufficiency(MI)
• Fleeting pains of a few seconds duration are not anginal in origin nor are prolonged
period of chest tightness that last for days.
• Unrelated to exertion and unrelieved by rest
• Heartburn, dysphagia, and absence of CHD risk factors indicate possible esophageal
disease.
• Anxiety, depression, panic, may suggest psychogenic origin
• Pain worsened by deep inspiration or cough is a hallmark of pleural irritation
Anonymous Noble 163-166, Goroll 94-96
In aortic dissection onset is often with “ripping” or tearing sensation, can radiate (through to
back). Assoc. neuro/vascular deficits distally (brain, spinal cord, limbs, does not relieve with
rest).
Musculo-skeletal - px reproducible with direct palp, positional px, aggravated with deep
inspiration, movement, cough.
Pleuritic px - cough/deep resp. will aggravate (not movement or palp).
GI (esophageal px) - unlike angina, can persist for hours as dull sensation after acute attack.
(also can be relieved with nitro though). Onset s/p food / drink.
Anxiety - can give CP unrelated to exertion, unrelieved by rest. Onset may be s/p event.
Anonymous Currents p 337/ Noble p 163-165
A) Angina can usually be diagnosed from a proper history. When atypical features are present
such as prolonged duration (hours or days); or darting, knifelike pains at the apex over the
precordium- ischemia is less likely.
1. Anterior Chest Wall Syndrome- sharp localized tenderness of intercostals muscles
that can be ruled out due to it being reproducible by palpation. May be caused by herpes zoster,
diabetes mellitus, etc.
2. Cervical or Thoracic Spine Disease- nerve root disease produces sharp severe chest
pain but can be ruled out by its relation to specific movements.
3. Peptic Ulcer, Chronic Cholecystitis, Esophageal Spasm, and Functional
Gastrointestinal Disease- this one is more difficult because these causes of pain may be relieved
by nitrates and calcium channel blockers, but they also may be improved by antacids, PPI's, not
reclining post meals, etc.
4. Degenerative and Inflammatory Lesions of the Left Shoulder- this may mimic
angina but usually can be ruled out due to the fact that they are usually elicited by movement of
the left shoulder.
5. Spontaneous Pneumothorax- this may cause chest pain with dyspnea and can also
lead to ECG changes arising from the changes in voltage due to the pneumo. This can be ruled
out by listening to breath sounds.
6. Disection of Aorta- this causes severe chest pain but usually can be ruled out by the
fact that it will lead to changes in pulses.

3. Distinguish typical angina pectoris from other causes of chest pain by


characteristics related to:
onset and precipitating factors,
duration,
location,
relief,
quality.
EChing, CMDT, P336-7;Noble on-line, Ch 63, p547-8
Onset and precipitating factors: Angina is caused by factors that increase myocardial demand,
except when due to pure vasospasm. Usually it’s provoked by exertion, such as climbing stairs,
walking up an inclined surface, vigorous work using the arms, or sexual activity. Other factors
that increase myocardial oxygen demand can result in angina include emotional excitement,
eating a large meal, and physical activities in cold weather. Threshold for angina is usually
lower in the morning hours.
Duration: Angina is of short duration and subsides completely w/o residual discomfort. If
attack is precipitated by exertion and pt promptly stops to rest, it lasts less than 3 mins. Attach
following a heavy meal or brought on by anger often last 15-20 mins.
Location: Distribution of distress vary widely in different pts but is usually the same for each pt
unless unstable angina or MI supervenes. In 80-90% of cases, discomfort is felt behind or
slightly to the left of the mid sternum. It may radiate to any dermatome from C8 to T4, most
often it radiates to left shoulder and upper arm, moving down the inner volar aspect of the arm to
the elbow, forearm, wrist, or 4th and 5th fingers. Occasionally, angina may be felt initially in the
lower jaw, the back of the neck, the interscapular area, high in the left back, or in the volar aspect
of the wrist.
Relief: Angina occurs most commonly during activity and is relieved by resting. Pts prefer to
remain upright rather than lye down. It is usually less after meals, during excitement, or on
exposure to cold. The diagnosis of angina pectoris is strongly supported if sublingual
nitroglycerin shortens an attack and if prophylactic nitrates permit greater exertion or prevent
angina entirely.
Quality: Pts describe a sensation of tightness, squeezing, burning, pressing, choking, aching,
bursting, “gas”, indigestion, or an ill-characterized discomfort. It’s characterized by clenching a
fist over the mid chest (Levine sign). The distress of angina is rarely sharply localized and is not
spasmodic. It is usually a steady discomfort that lasts a few mins, rarely more than 10, unless
unstable angina or an MI is evolving. It ALWAYS lasts more than a few secs, which helps
differentiate it from some types of musculoskeletal pain.
Anonymous
Heart and pericardium:
Angina:
• Onset - Typically sudden onset precipitated by exertion, emotional stress, or exertion.
• Duration - 2-20 minutes.
• Location - Pain radiates to jaw, neck, shoulder, back, or abdomen. (also arm numbness)
• Relief - from rest or nitro
• Quality - Described as squeezing, heaviness, or pressure, though quality is not diagnostic.
Others causes of chest pain related to heart and pericardium:
• Onset and precipitating factors - cocaine abuse can trigger ischemia by precipitating coronary
vasoconstriction, increasing myocardial oxygen demand, and enhancing platelet aggregation.
Atypical angina differs from typical angina in location, quality or other characteristics but is
similar in timing or other features. This is thought to be precipitated by coronary
microvascular dysfunction (microvascular resistance causes decreased perfusion).
Nocturnal angina patient awakes from sleep with angina pain, thought to be precipitated by
nocturnal andrenergic hyperactivity and pump failure. In pericarditis spread of the
inflammatory process from the insensitive pericardium to the sensitive parietal pleura
precipitates pleuritic pain. (this pain is sharp and aggravated by respiratory activity and
precipitated by swallowing if the posterior aspect of the heart is involved). Sudden onset of
severe chest pain, maximal from the start suggests aortic dissection.
• Duration - (?) I think all these other causes of chest pain related to the heart and pericardium
last longer than angina but it didn't say in Goroll.
• Location - in pericarditis pain may be referred to tip of shoulder. In aortic dissection may
radiate to interscapular region.
• Relief - in pericarditis patients may notice lessening of pain when sitting up and leading
forward.
• Quality - sharp, aggravated by respiratory activity = pericarditis
Severe w/ tearing or ripping quality = aortic dissection
Others more similar to typical angina

Chest wall:
• Onset - Can be precipitated by vigorous or unaccustomed exercise (muscular or ligamentous
strain), inflammation (costochondritis), rib fracture, metastatic cancer, herpes zoster (virus)
or cervical root compression (c-spine disease).
• Duration - varies from a few seconds to several days. The pain of herpes zoster may precede
the rash by 3-5 days. In the elderly it may persist for months, long after the rash subsides.
• Location - pain can be pinpointed by the patient being somatic in nature
• Relief (?)-aggravated by cough, deep inspiration, direct palpation and movement.
• Quality - ranges from sharp to dull or aching.

Lungs and pleura:


• Onset and precipitating factors - inflammation or distention of pleura triggered by infection,
pulmonary infarction, neoplasm, uremia, and connective tissue disease. Stretching of the
pleura from a spontaneous pneumothorax causes acute onset of pleuritic pain (and dyspnea).
Pleurodynia is a self-limited source of pleuritic pain precipitated by a viral syndrome. Cough
initiated injury or bronchospasm may precipitate chest pain from this source as well as
pneumonia, TB, or pulmonary embolus. An onset of sudden sharp pleuritic pain in young
healthy persons that is relieved by taking a deep breath is called precordial-catch syndrome.
• Location - no specific location given in Goroll.
• Relief - worsened by deep inspiration or cough but relatively unaffected by movement or
palpation. (except precordial-catch syndrome)
• Quality - the more florid the infection the greater the pain.

Esophageal motor dysfunction


• Onset and precipitating factors - precipitants can mimic angina (e.g. exposure to cold,
exertion) Unlike angina may occur with swallowing. "spasm” may occur spontaneously or in
the context of meals or acid reflux.
• Duration - unlike angina may persist as a dull sensation for hours.
• Location - some patients report dysphagia as an accompanying symptom
• Relief - nitrates and calcium channel blockers provide relief as w/angina
• Quality - generally similar to angina

Esophageal reflux
• Onset - brought on by large meal, lying down, or bending over.
• Duration -? longer than angina ?
• Location - may radiate to interscapular region. (acute cholecystits can produce substernal
nitrates relieved by nitro). PUD and pancreatitis can produce substernal chest pain also.
• Relieved by antacids

Psyche
• Onset – unrelated to exertion and unrelieved by rest
• duration – hours to days
• location – Goroll doesn’t specify
• relief - ? treat underlying anxiety?
• Quality - patients w/ anxiety or depression often describe feelings of chest tightness or
heaviness. Anxiety disorders can include a feeling of not being able to take a deep breath.
Aprilt Noble 2nd ed. p. 220
QUALITY, DURATION and RELIEF- Angina is described as a tightness, squeezing, heaviness,
pressure, burning, indigestion, or an ache. It usually a steady discomfort that lasts a few
minutes, rarely more than 10, UNLESS it becomes unstable angina or an MI. Always more than
a few seconds, which will differentiate it from a muscu/skel type discomfort. It is usually
relieved quickly with a SL (sublingual). Relief with Nitro (NTG) may be a distinguishing
feature.
LOCATION and RADIATION- angina is usually a diffuse sensation rather than at discrete spot.
If a pt. can localize with one finger ischemia is an unlikely cause. Usually most intense
retrosternal and left ant. chest but may occur anywhere from the jaw to upper abd. frequently
radiates to the shoulders, upper back, neck, or inner aspect of the arms, particularly on the left
side. When an MI is in progress it is generally more diffuse and severe.
PRECIPITANTS- Except when caused by pure vasospasm, angina is usually caused by things
that ^ myocardial demand. Provoked by exertion, climbing stairs, sex or work using the arms.
Eating large meals and activity in cold weather. The threshold for angina in usually lower in the
AM. The patients who primarily experience vasospasm usually acquire angina at rest
independent of activity.
Anonymous Noble-166
Onset and precipitating factors- gradual onset and resolution; continuous pain typically lasts 1-
10 min. The pain will have a “lag” time before onset after precipitating events, exercise, stress,
eating, cold temp, cocaine, etc.
Duration- lasts 1-10min after precipitating factors.
Location- substernal or anterior chest; May radiate to upper back, neck, teeth, shoulder or arm.
Relief- rest, Ntg. Pain that vanishes immediately on stopping activity is not likely to be angina.
Quality- heaviness, pressure or squeezing (occasionally burning) or deep dull ache;
accompanied by nausea, diaphoresis or hypotension.
Anonymous Currents p 336-337
A) Onset and Precipitating Factors: Angina occurs most commonly during activity with
exertion that involves straining the thoracic or upper extremity muscles. Rapidly walking uphill
and sexual activity also can precipitate angina due to their increased heart rate increasing oxygen
demands on the heart. The threshold for angina is often lower in the morning or after periods of
strong emotion.

B) Relief: Angina is usually relieved by rest and the diagnosis can be strongly supported if
relieved by sublingual nitroglycerin. Upright posture, time after meals, times of excitement, and
upon exposure to cold are instances where angina is less likely.

C) Duration: Angina is of short duration and subsides completely without residual discomfort.
If the attack is precipitated by exertion, and the patient promptly stops to rest the duration is
usually not longer than 3 minutes. Attacks after a meal or period of emotion may last up to 15
minutes and periods lasting longer than 30 minutes suggest unstable angina or myocardial
infarction.

D) Location: In 80-90% of cases the pain is felt behind or slightly to the left of the sternum.
When it begins to the left of the sternum or uncommonly to the right of the sternum it will
usually move to be more directly behind the mid sternum. Radiation is most commonly to the
left shoulder and upper arm, and occasionally can be felt initially in the lower jaw and back of
the neck.

E) Quality: Patients often do not describe angina as pain, rather as discomfort or a sensation of
tightness, squeezing, burning, pressing, choking, aching, bursting, or an ill characterized
discomfort. It is often characterized by the Levine Sign- clenched fist held at the sternum.

4. Compare and contrast the typical presentation of each of the following:


variant (Prinzmetal's angina),
unstable angina.
EChing, CMDT, P341, 345-6;Noble on-line, Ch 63, p558
variant (Prinzmetal's angina): a clinical syndrome in which chest pain occurs w/o the usual
precipitating factors and is associated w/ ST segment elevation. It often affects women under 50
yrs of age and can be exercise related. It characteristically occurs in the early morning,
awakening pts from sleep, tends to involve the right coronary artery, and is apt to be associated
w/ arrhythmias or conduction defects.
unstable angina: an acceleration of symptoms in which ischemic episodes occur more
frequently, are more intense, last longer, and are precipitated by less activity than previously, or
even by rest, and is less responsive to medication. May exhibit ST segment depression or T
wave flattening or inversion but sometimes, and more ominously, ST segment elevation during
pain. Pts may also exhibit signs of left ventricular dysfunction (pulmonary rales, S3, mitral
regurgitation) during pain and for a time therafter.
Aprilt Noble 2nd ed. p. 219
VARIENT(Prinzmetal’s)- Intense focal coronary artery spasm in the absence of underlying
atherosclerotic dz. The angina may occur at rest, because the spasm causes a marked reduction
in coronary blood flow. This is a rare cause of angina.
UNSTABLE- an increase in the frequency and duration of ischemic episodes occurring at lower
workloads than previously and at rest. Up to 20% of these pt. will sustain an MI in 3 months.
Anonymous
• variant (Prinzmetal's angina), refers to anginal pain occurring exclusively at rest in
conjunction w/ transient ST segment elevation on ECG. Associated with coronary artery
spasm at the site of high grade proximal fixed stenosis. Other forms of coronary disease
may present with a similar clinical picture.
• unstable angina. - according to our lecture unstable angina can be angina at rest but
refers to a changing scenario. Unstable angina is the onset of new chest pain within the
last two months, severe enough to inhibit activities. It also includes established angina
with an increase in frequency, severity of duration, or less provocation to exhibit
symptoms. It also includes resting or nocturnal pain in someone with previously stable
angina.
Anonymous
Variant (Prinzmetal's angina): Describes angina occurring at rest or in atypical patterns such
as after exercise or nocturnally. Prinzmetal's angina is caused by coronary artery spasm and is
associated with ECG changes (usually ST elevation) during symptoms.
Classically, this syndrome was associated with coronary artery spasm at the site of high-grade
proximal fixed stenosis. However, other forms of coronary disease may produce a similar
clinical picture, and coronary vasospasm may present in ways other than Prinzmetal's
description. Cocaine abuse can trigger ischemia by precipitating coronary vasoconstriction,
increasing myocardial oxygen demand, and enhancing platelet aggregation. It may present as
angina in a young person with no other coronary heart disease (CHD) risk factors.
Although the average survival at 5 yr is 89 to 97%, patients with variant angina and severe
coronary artery obstruction are at greater risk. Relief of variant angina is usually prompt after
sublingual nitroglycerin; Ca blockers appear to be highly effective.
Unstable angina: is one of the acute coronary syndromes, along with non- Q wave myocardial
infarction and Q-wave myocardial infarction. Pain which is new or which is changed in
character to become more frequent, more severe or both. The clinical presentations of unstable
angina include onset of new chest pain within the past 2 months severe enough to inhibit activity;
established angina now increasing in frequency, severity, and duration (crescendo angina) and
occurring with progressively less provocation; and development of rest pain or nocturnal angina
in a person with a previously stable anginal pattern. Unstable angina portends myocardial
infarction in a certain percentage of patients. Clinical features associated with greatest risk
include rest pain in excess of 20 minutes, signs of pump failure (hypotension, rales, S3), new or
worsening mitral regurgitation, and 1 mm or more of ST segment change with pain.
Anonymous
A) Variant (Prinzmetal's Angina)- this is a clinical syndrome in which chest pain occurs
without the usual precipitating factors and is associated with ST segment elevation rather than
depression. It often affects women under 50 years old and characteristically occurs early in the
morning awakening patients from sleep. This is apt to affect the right coronary artery and is
associated with arrhythmias or conduction defects. Currents p 341

B) Unstable Angina- this term is used to denote an accelerating or "crescendo" pattern of pain
in cases where previously stable angina occurs with less exertion or at rest, last longer, and is
less responsive to medication. This carries a poorer prognosis than stable angina. Newly
diagnosed angina may be termed unstable until it is determined whether or not it will be
responsive to rest and medication. Currents p 346

5. Differentiate by symptoms other diseases besides coronary artery disease


which may cause chest pain. In particular know common pulmonary,
esophageal, musculoskeletal, and neurologic causes.
EChing, Noble on-line, Ch 16, table 16-2, p166-7
Diseases that present w/ chest pain
Etiology Quality & Region & Severit Timing Provocati Palliative
associated sxs. distributio y ve interventio
n activities ns
Pulmonary Ranges from Substernal Variabl Sudden None; None
infarction from deep crushing , if e onset 50% have
pulmonary pain similar to massive; signs or
embolism MI or more lesser sxs of
pleuritic; may pulmonar DVT
be y peripheral
accompanied infarction ly
by weakness, pain in
n/v, dyspnea chest area
Pneumothorax Sharp, Often Severe Sudden Trauma, None
stabbing unilateral; onset sports,
chest wall spontaneo
area us & occa.
heavy
lifting
Pleural pain Sharp, Superficia Mild to Gen. Deep Splinting
knifelike; dull l chest severe Occurs breath, or
ache possible wall w/provocati cough, positional
between sharp ve laugh, change
pains maneuvers movement
s
Tracheobronch Burning or Substernal Mild to Accompani Cough
itis ache after ; middle modera es or
prolonged or upper te follows
coughing respiratory
tract
infections
Esophagitis, Deep burning, Epigastriu Variabl Gen. ETOH, Food,
esophageal may be m or e Gradual NSAIDs, water,
pain, GERD indistinguisha substernal onset & foods, antacids
ble from MI or ; as w/ more large
angina angina chronic w/ meals
fluctuating
course
Esophageal Indistinguisha
spasm ble from MI
Gastric/peptic Deep Epigastric Variabl 60-90 mins Anything Milk,
ulcer burning/dull or e after eating that ↑ antacids
ache substernal gastric or
duodenal
acidity
Cholecystitis Ache or Epigastric Variabl After meals, Fatty or Time
crampy pain , RUQ, or e mins to hrs; rich foods
substernal may be
constant or
fluctuating
(colicky)
Mediastinal Intense, sharp Substernal Severe After None
empnysema pain or trauma or
shoulders coughing
Mediastinitis, Similar to Greatest
tumors pleuritis in
substernal
region
Costochondriti Ranges from Substernal Variabl Variable May be
s sharp(secs) to , costal e but worsened
dull(days to margins rarely by motion
wks) ache to or lower severe or
tightness ribs exercise
Shoulder and Variable Shoulder Variabl Motion of Rest
muscle pain w/ e arms, but
radiation not
to anterior general
or exertion
posterior
chest
Aprilt Noble 2nd ed. p. 220
This exact answer was hard to find, it may answered better in the 3rd ed, but I don’t have it, sorry
guys.
Pulmonary- chest wall pain, sharp and localized, ^ with palpation
Esophageal- nonexertional, often associated with dysphagia, or reflux, worsened by certain
foods, ASA, or lying supine. May be relieved by antacids.
Musculoskeletal- only a few seconds of pain
Neurologic- cervical and thoracic spine pain- shooting type pains, or an ache worsened by
movement of neck or back. May be a dermatomal distribution.
Pericarditis- sharp, pleuritic, postional, friction rub. Diffuse ST^, PR depression on an EKG
Biliary- RUQ tenderness, fatty food intolerance.
Anonymous
• Pain brought on by exertion and relieved by rest - is indicative of angina but psychogenic
disease and esophageal spasm may behave in similar fashion. Psychogenic origin - anxiety
depression, panic, light-headedness, nervousness, weakness, fatigue, and lifelong history of
other bodily complaints (somatization). Esophageal origin - heartburn, dysphagia, and
absence of cardiac risk factors.
• Prompt response to nitroglycerin - classic feature of CHD (coronary heart disease) but
esophageal spasm, cystic duct spasm, and even some psychogenic etiologies may respond to
nitrates.
• Chest pain brought on by eating - may be due to angina, but in the absence of other risk
factors for CHD one must consider gastroesophageal or pancreaticobiliary pathology.
• Pain worsened by deep inspiration or cough - is a hallmark of pleural irritation but is
suggestive of pericarditis and chest wall pathology as well. Focal chest wall tenderness
worsened by movement narrows the differential to chest wall origin. In the absence of focal
chest wall pain one must search for intrathoracic pathology. Pneumothorax - sudden onset
pleuritic pain accompanied by dyspnea in young patient. Precordial catch syndrome - brief,
self-limited episodes in otherwise healthy young person. Pericarditis - pleuritic pain
worsened by turning but relieved by sitting up and leaning forward. Pleurodynia - pleuritic
pain in the setting of an epidemic of minor viral respiratory disease.
• Sudden onset of maximally severe pain - possible aortic dissection. If episode is
accompanied by a new neurological deficit or a syncopal episode then urgent hospitalization
is indicated.
Anonymous Goroll, p.108-111
Chest Wall – can be pinpointed, somatic in nature, deep inspiration, cough, direct palpation, and
movement makes it worse. Duration ranges from a few seconds to several days and quality from
sharp to dull or aching.
a. Costochondritis – inflammatory condition caused by localized swelling, erythema, warmth,
and tenderness
b. Rib fracture – history of trauma or metastatic CA.
c. Herpes Zoster - dermatomal distribution pain precedes the typical rash; hypoesthesia to
dysesthesia and hypersthesia.
d. Cervical root compression/Thoracic outlet syndrome – produces pain in the chest and
upper arm, resembling angina.
Lungs and Pleura - Inflammation or distention of the pleura worsened by deep inspiration and
cough but not movement or palpation. Causes include infection, pulmonary infarction,
neoplasm, uremia, and connective tissue disease.
Esophagus – can mimic anginal chest pain; persists as a dull sensation for several hours after an
acute attack and may occur with swallowing; may radiate to interscapular region.
a. Reflux
b. Spasm
c. Cholecystitis
d. PUD
e. Pancreatitis
Anonymous Currents p 337/ Noble p 163-165
1. Anterior Chest Wall Syndrome- sharp localized tenderness of intercostals muscles
that can be ruled out due to it bieng reproducible by palpation. May be caused by herpes zoster,
diabetes mellitus, etc.
2. Cervical or Thoracic Spine Disease- nerve root disease produces sharp severe chest
pain but can be ruled out by its relation to specific movements.
3. Peptic Ulcer, Chronic Cholecystitis, Esophageal Spasm, and Functional
Gastrointestinal Disease- this one is more difficult because these causes of pain may be relieved
by nitrates and calcium channel blockers, but they also may be improved by antacids, PPI's, not
reclining post meals, etc.
4. Degenerative and Inflammatory Lesions of the Left Shoulder- this may mimic
angina but usually can be ruled out due to the fact that they are usually elicited by movement of
the left shoulder.
5. Spontaneous Pneumothorax- this may cause chest pain with dyspnea and can also
lead to ECG changes arising from the changes in voltage due to the pneumo. This can be ruled
out by listening to breath sounds and recognition of a deviated trachea.
6. Disection of Aorta- this causes severe chest pain but usually can be ruled out by the
fact that it will lead to changes in pulses.
7. Pulmonary Friction Rubs- indicative of pleuritis
8. Pulmonary Consolidation- decreased tactile fremitus
9. Fine Crackles with Inspiration/Expiration- indicative of heart failure

6. Compare and contrast causes of chest pain which will have normal physical
examinations, against those which will demonstrate important physical
findings.
Tim Noble 164-168, 548 and 549, CMDT pgs 336-354 and 5 min. clinical consult palm
I divided these up according to non CV etiologies and CV etiologies because the books didn’t
divide things up the way the question is asking. The next 5 objectives are the best I could do
considering the vagueness and depth of what’s being asked and my inexperience with all things
CV, so if you think I missed the boat on something let me know. Thanks guys.

*Causes of chest pain w/ normal CV PE findings:


a.) Esophageal spasm: No pertinent chest PE findings.
b.) Gastric/peptic ulcer: abdominal distention, weight loss, succussion splash, hematemasis or
melena (if GI bleed) and possible vomiting (obstruction). No pertinent chest PE findings.
c.) Cholecystitis: possible vomiting, mild to moderate fever, positive Murphy’s sign, possible
jaundice, mild to marked hepatomegaly. No pertinent chest PE findings.
d.) Costochondritis (Tietze’s syndrome): Pain (sharp or pleuritic) to palpation of
costochondral or costosternal regions. Non-suppurative edema at rib articulations, redness and
warmth at sites of tenderness. (Mostly 2nd – 5th costal cartilage involved)
e.) Shoulder and neck arthritis (or nerve root involvement) and muscle pain: Positive
shoulder or neck PE may radiate to chest (i.e. when ROM or strength tests done, muscular, bone-
joint, nerve or ligamentous structures adjacent or attached to chest wall will elicit pain in the area
of concern indicating musculo-skeletal etiology).
f.) Pleurisy (pleural effusion, this could fit in either): depending on etiology cough may be
productive or not, dyspnea, chest wall splinting, tachypnea particularly w/ lung compression or
more severe infections, diminished chest wall excursion, decreased tactile fremitus, dullness to
percussion over effused areas, diminished or absent breath sounds, friction rub, weight loss,
hemoptysis, night sweats and general malaise.
g.) Esophagitis, esophageal pain, GERD: signs of aspiration pneumonia (rales, egophany,
tachypnea, tachycardia etc.), weight loss, and chronic cough.
h.) Pneumonia: fever, hypothermia, tachypnea, tachycardia, possible pleural effusion, and
altered breath sounds/rales.

*Causes of chest pain w/ important CV PE findings:


a.) Angina pectoris (includes ischemic and CAD)- dx heavily reliant on symptoms and hx, PE
can show either increased BP or hypotension, pt may describe pain as levine’s sign, occasional
gallop rhythm and an apical systolic murmur due to transient mitral regurgitation, SVT or
ventricular arrhythmias may be present as the precipitating event or as a result of ischemia.
Relieved by rest or nitroglycerin.
b.) Unstable Angina: Also called accelerating or “crescendo” pattern of pain in cases where
previously stable angina occurs with less exertion or at rest, lasts longer, and is less responsive to
medication.
c.) Pulmonary embolism: tachycardia, tachypnea, syncope, distended jugular veins (JVD),
hypotension or cardiac arrest, S3 gallop, possible signs of DVT (holman’s sign) and clear lung
sounds initially (until infarct).
d.) Pulmonary infarct: Pleuritic chest pain, possible hemoptysis, tachypnea, tachycardia,
rales, wheezes and/or pleural effusion and possible signs of DVT.
e.) Myocardial Infarction (MI): pallor, syncope, vomiting, cough, diaphoresis, rales,
wheezing, S4 heart sound, arrythmias, hyper or hypotension can occur and JVD.
f.) Pericarditis: Sharp retrosternal pain of sudden onset that is exacerbated by inspiration or
movement and relieved by leaning forward and sitting up. Splinted breathing, odynophagia,
fever, pericardial friction rub, intermittent cardiac arrythmias, SVT, tachypnea, localized rales.
g.) Pericardial tamponade: Dyspnea, tachycardia, DJV, cyanosis, hypotension, quiet
precordium with little cardiac activity, pericardial friction rub, lungs clear, Ewart’s sign (dullness
and bronchial breathing between the tip of the left scapula an vertebral column), rapid thready
pulse, change of consciousness, pulsus paradoxus (> 10mm Hg decrease in systolic pressure w/
inspiration, Beck’s Triad (DJV, hypotension and muffled heart sounds).
h.) Aortic dissection: abrupt onset of shearing chest pain which radiates to the interscapular
region, back pain, syncope, symptoms of CHF, stroke, limb ischemia, abdominal pain, hyper or
hypotension, wide pulse pressure, features of tamponade, dullness in left lung base, fever and
pulse deficits.
i.) Pneumothorax: Pleuritic chest pain, cough, dyspnea, cyanosis, moderate to severe
respiratory distress, shock, circulatory collapse, asymmetry of respirations, diminished breath
sounds on the affected side, tachycardia, rapid-shallow breathing, hyperresonance to percussion,
crepitus over chest wall. Tension pneumothorax- weak-rapid pulse, pallor, JVD, tracheal
deviation.
j.) Valvular Heart Disease: any of the heart valves may be involved, may be stenotic or
congentially deformed. The variety of signs and symptoms are to numerous to name here
(different heart sounds w/ different valvular involvement and chamber enlargements etc). Some
may be asymptomatic for years and appear as chest pain later.
k.) Psychological (anxiety): somatic symptoms such as cardiac palpitations, tachycardia,
tachypnea, dyspnea, hot or cold flashes, abdominal pain, dizziness and tremor. While these signs
and symptoms would end up being normal with no physical etiology, the findings would still
have to be worked up.
Kate Current Ch. 10, Noble Ch 16
This is a compilation from a lot of reading, there isn't one specific source.
Normal PE: Prinzmetal's angina, esophagitis and esophageal spasm, GERD, cervical dorsal
arthritis, Psychogenic causes (Panic or anxiety disorder)

Positive or normal PE: angina pectoris, MI

Positive PE: Aortic dissection, Musculoskeletal causes, Mitral Valve prolapse, pleural process,
Myocarditis, pulmonary causes and most cardiogenic disorders.
Anonymous Noble 1005-6
There is an excellent table on page 111 (Goroll) that outlines the complete differential diagnoses
for chest pain; I think it’s worth looking at in detail, but I’ll summarize it here. Basically, there
are five categories of chest pain: chest wall, cardiopulmonary, aortic, GI, and pyschogenic. All
except for the pyschogenic causes will produce important physical exam findings. Pyschogenic
causes should be considered when more serious causes have been worked-up and ruled-out.
Also, about one-third of patients with chest pain of unknown origin (CPUO), have concurrent
panic disorder.
Anonymous Noble p 163-165/ Currents p 337
Cause of Chest Pain Important Physical Findings Normal Findings
Anterior Chest Wall Sharp intercostals pain than may be
Syndrome associated with warmth, swelling,
and may be painful to touch
Intercostal Neuronitis Intercostal pain due to herpes
zoster or diabetes and again
reproducible with palpation
Cervical and Thoracic Sudden sharp pain that is
Nerve Root Disease reproducible with certain
movements of the neck or spine
GERD Normal exam with the exception of
increased discomfort upon bending
over after heavy meals
Degenerative/ Normal exam but pain may be
Inflammatory Lesions of reproduced with manipulation of
the left shoulder the left arm or shoulder
Spontaneous Decreased breath sounds on side of
Pneumothorax pneumo as well as tracheal
deviation
Pleuritis Chest pain with auscultation of
pulmonary friction rubs
Pulmonary Consolidation Decreased tactile fremitus over
lobes involved
Disection of Aorta Changes in peripheral pulses in
different extremities
Valvular Disorders Appropriate murmurs
Pericarditis Cardiac friction rubs
Anxiety/ Panic Disorder Normal Exam Findings with
the exception of mild
tachycardia and tachypnea
Hypertension Normal exam findings
although may cause
headaches
Acute Myocardial Exam may be acutely normal
Infacrtion
Angina Exam may be normal
however history will elucidate
decreased discomfort with
rest
Esophageal Spasm Exam may be normal
Functional Exam may be normal
Gastrointestinal Disease

7. Be able to differentiate several common chest-wall syndromes.


Tim Noble 548 and 5 min clinical consult
I’m not sure what they’re looking for here, there isn’t a whole lot about chest wall syndromes in
any book I have.
a.) Costochondritis: explained above
b.) Fibromyalgia: Pain syndrome defined by pattern of trigger points (some of which are in
areas of the chest)
c.) Slipping rib syndrome: chronic subluxation of a rib.
d.) Costovertebral arthritis: just what it says.
e.) Rib trauma with swelling: yup.
f.) Alkylosing spondylitis: inflammatory condition occurring at tendonous and ligamentous
attachment to bone (spine)
g.) Precorial catch syndrome: pain due to the small joints in the chest wall. Typically the
pain is sharp, lasts seconds, and may occur with a deep breath. It does not get worse with
age.
h.) Painful xiphoid syndrome: similar to precordial catch except centered at the xiphoid
process.
Kate Up to Date, Chest Wall Syndromes
The differential diagnosis of musculoskeletal chest pain has been divided into three categories:

* Isolated musculoskeletal chest pain syndromes (costosternal, posterior chest wall syndromes)
* Rheumatic diseases
* Nonrheumatic systemic diseases

Isolated musculoskeletal chest pain syndromes — There are a number of chest wall syndromes
with chest pain associated with musculoskeletal inflammation.

* "Costochondritis" is one of the more common presentations of musculoskeletal chest pain. It is


a diffuse pain syndrome, in which multiple areas of tenderness are found that reproduce the
described pain. The upper costal cartilages at the costochondral or costosternal junctions are
most frequently involved. The areas of tenderness are not accompanied by heat, erythema, or
localized swelling.

* Chest wall pain occurring after coronary artery bypass surgery may be a result of incisional
discomfort, of internal mammary artery grafting, or related to sternal wires.

* Costovertebral joint dysfunction syndrome is an uncommon condition that causes posterior


chest wall pain and may mimic a pulmonary embolism. Thoracic disk herniation is another
unusual cause of posterior chest pain; the pain is sometimes dermatomal and "band-like," and
retrosternal or retrogastric pain has also been described.

Rheumatic diseases — Involvement of thoracic joints in rheumatic diseases can be associated


with musculoskeletal chest wall pain . Examples include rheumatoid arthritis, ankylosing
spondylitis, psoriatic arthritis, and fibromyalgia..
Nonrheumatic systemic diseases — Several systemic disorders are characterized by bony
involvement that can lead to chest wall pain. Examples include stress fractures, neoplasms, and
sickle cell anemia.

Skin and sensory nerves — Chest pain may be the presenting symptom of herpes zoster
(shingles); it may precede the characteristic rash and, rarely, zoster may occur without a rash.
Dysesthesia is usually present in the affected dermatome. Postherpetic and postradiation
neuralgia are other unusual causes of chest pain.
Anonymous
• Costochondritis (Tietze's syndrome) - inflammatory condition that produces localized
swelling, erythema, warmth, and tenderness at costochondral or costosternal junctions.
• Rib fracture - similar to costochondritis with different location. History of antecedent
trauma or metastatic cancer.
• Herpes zoster - dermatomal distribution. Pain may precede rash by 3-5 days. In elderly pain
may persist for months, long after rash subsides.
• Cervical root compression - due to cervical spine disease or thoracic outlet syndrome nerve
injury can produce pain in the chest and upper arm. In the outlet syndrome , a cervical rib can
compress part of the brachial plexus, resulting in motor and sensory deficits in an ulnar
distribution at the same time that there is discomfort in the chest and upper arm.
Anonymous Goroll, pp 108-109
Muscular or ligament strain- hx of vigorous, unaccustomed exercise, pinpoint pain with
quality of sharp to dull/aching, occasionally c/o chest tightness.
Costochondritis- inflammatory condition causing localized swelling, erythema, warmth and
tenderness of the costochondral or chondrosternal junction.
Rib fracture- hx of antecedent trauma or metastatic CA, similar to costochondritis, but located
over rib, rather than at junction.
Herpes Zoster- characteristic dermatomal distribution, pain may precede rash by 3-5 days. May
c/o hypoesthesia, dysesthesia or hyperesthesia.
Cervical root compression or Thoracic outlet syndrome- due to cervical spine disease. May
produce pain in the chest and upper arm, superficially resembling angina. With outlet syndrome,
often accompanied by motor/sensory deficits in ulnar distribution
Anonymous Currents p 337
A) Anterior Chest Wall Syndrome- sharply localized intercostals tenderness
B) Tietze's Syndrome- inflammation of the costo-chondral junctions which may be warm,
swollen, and red of which pain is reproducible by local pressure
C) Intercostal neuronitis- due to herpes zoster or diabetes mellitus disease of these nerves

8. Summarize common tests ordered for patients with chest pain, their possible
indications, and their limitations.
Tim Noble 548-550
Laboratory findings: Blood tests to evaluate for underlying risk factors include measurement
of the serum lipids and fasting serum glucose. The hematocrit and thyroid function tests should
be measured if clinically appropriate, since anemia and hyperthyroidism can exacerbate
myocardial ischemia.
Resting ECG: Many patients with CAD have normal baseline ECGs, or may demonstrate
pathologic Q waves indicative of previous infarction. In many patients minor ST and T wave
abnormalities are present but are not specific for CAD. However, the ECG can be diagnostically
useful if recorded during an episode of chest pain, whereupon ischemia often results in transient
horizontal or downsloping ST segments or T wave inversions, which normalize following
resolution of the pain. Less often, transient ST elevation may be observed, which suggests severe
transmural ischemia or coronary artery spasm.
Exercise Stress Testing: The most useful noninvasive studies in the evaluation of angina
involve exercise testing. In patients without resting ST or T wave abnormalities the standard
treadmill (or bicycle) exercise test, without additional imaging modalities, should be the initial
procedure, since it is the most convenient and cost-effective. For patients whose presentation
strongly suggests myocardial ischemia, the exercise test has a sensitivity and specificity greater
than 85%. However, when the probability of significant coronary disease is low (e.g., a young
woman with prickly chest pains), the test is less specific, and false-positive results are more
common. Exercise testing is most commonly used (1) to confirm the diagnosis of angina, (2) to
identify IHD patients at high risk of complications, (3) to assess the response of antianginal
therapy, and (4) as a screening procedure for certain asymptomatic populations, such as
individuals with strong cardiac risk factors, older patients about to begin exercise programs, and
individuals whose well being could affect public safety (e.g., airline pilots).
Radionuclide Studies: Two types of nuclear studies are used to enhance the diagnostic value of
standard exercise tests: myocardial perfusion imaging and radionuclide ventriculography. These
tests can provide additional information regarding the location and extent of CAD, and their
interpretations are not hampered by resting ECG abnormalities. However, they are more
expensive than standard treadmill tests and should be used judiciously.
Myocardial Perfusion Scintigraphy: Myocardial perfusion scintigraphy is generally more
sensitive and specific than conventional stress testing. During this test a radionuclide (e.g., 201
T1 or 99m Tc sestamibi) is used, which after peripheral venous injection distributes to the
myocardium in proportion to coronary blood flow. The radionuclide is injected at peak exercise,
and immediate imaging is performed. Perfusion defects (cold spots) indicate regions of prior
infarction or exercise-induced ischemia. Repeat imaging at rest several hours later shows filling
in of the zones that were ischemic, differentiating them from regions of previous infarction. The
location of perfusion abnormalities correlates with coronary disease in the respective territory
(e.g., left anterior descending artery disease results in perfusion abnormalities within the anterior
wall). Multiple large perfusion defects correlate with left main or severe three-vessel disease.
Echocardiography: Imaging of the left ventricle by ultrasound can reveal segmental wall
motion abnormalities indicative of ischemia or previous infarction. In exercise
echocardiography, left ventricular function is assessed before and during vigorous exercise
(supine bicycle or treadmill); exercise-induced segmental regional wall motion abnormalities are
an indication of ischemia. Exercise echocardiography is therefore analogous to exercise
radionuclide ventriculography and has similar sensitivity and specificity for the presence of
significant coronary disease. For patients who are unable to exercise, pharmacologic stress
testing with echocardiography can be performed in one of two fashions: (1) using potent
vasodilators such as adenosine or dipyridamole (analogous to pharmacologic 201 T1
scintigraphy described above), or (2) using an adrenergic stimulating drug (e.g., dobutamine).
Either of these techniques signifies the presence of myocardial ischemia by drug-induced left
ventricular wall motion abnormalities.
Coronary Arteriography: Coronary arteriography allows selective visualization of the
coronary arteries and their major branches and is the most accurate means to detect the presence
and extent of CAD. In experienced laboratories it is performed with low mortality
(approximately 0.2%) or severe vascular complications (0.7%). However, this technique is
costly, is not risk-free, and is seldom needed to simply establish the diagnosis of significant
coronary disease. The decision to proceed with arteriography should be dictated by the patient's
clinical presentation and only when a change in therapeutic plan is under consideration. Note that
an individual with mild to moderate angina that is reasonably controlled with medical therapy
does not generally require cardiac catheterization, since the long-term prognosis and quality of
life may not be significantly affected. However, if that individual has other markers of a poor
prognosis by exercise testing or impaired left ventricular function, then catheterization should be
performed.
Kate CMDT 316
ECG - indicates cardiac rhythm, reveals conduction abnormalities, and provides evidence of
ventricular hypertrophy, MI, or ischemia. Nonspecific ST segment and T wave changes may
reflect these processes but are also noted with electrolyte imbalance, drug effects, and many
other conditions. Routine x-rays and ECGs are not recommended to screen for heart disease and
have limited role in the follow-up of pts with known heart disease. However, a baseline ECG is
helpful in older pts.
Chest X-ray: provides information about heart size, pulmonary circulation (with characteristic
signs suggesting both pulmonary artery or pulmonary venous hypertension), primary pulmonary
disease and aortic abnormalities. When using X-ray, provider should already have high-suspicion
for specific diagnosis.
M-mode and 2-dimensional Echocardiography: Used in initial and serial evaluation of many
conditions. Provides measurements of left ventricular size, function and thickness. Left
ventricular segmental wall motion can be assessed, and the size of all four cardiac chambers can
be determined. The morphology of the heart valves can be examined. Hypertrophic
cardiomyopathy, pericardial effusion, mitral valve prolapse, valvular vegetations and cardiac
tumors may all be diagnosed.
Transesophageal echo: Provides information about posterior structures (especially atria and
atrioventricular valves) and prosthetic valves, and to monitor pts during surgery. It is superior to
surface echo in diagnosing left atrial thrombi, valvular vegetations and eccentric mitral
regurgitant jets (esp. with prosthetic valves). It is quite sensitive in detecting aortic dissection
and severe atherosclerosis of the ascending aorta.
Stress Echo: used to enhance the information available from ECGs and as an alternative to
nuclear medicine procedures. May be performed during/ after exercise. Transient depression of
segmental wall motion during or following stress suggests ischemia. Dobutamine infusions can
also be utilized in pts unable to exercise.
MRI: Currently available systems provide high-quality and high-resolution images of cardiac
and adjacent vascular structure, making this a preferred technique to evaluate many cardiac
conditions, including pericardial and congenital abnormalities. MRI also provides excellent
images that can be used to quantify cardiac function and structure. With the use of gadolinium
contrast agents, MRI has been used to assess myocardial perfusion and viability.
Cardiac catheterization and angiography:: These invasive procedures have been supplanted
by echocardiography for the initial and serial evaluation of many conditions, but remain
invaluable in the management of most pts with congenital, valvular and coronary disease. Risks
- pneumothorax, bleeding, arrhythmias, pulmonary artery rupture, PE, and infection. Role of this
procedure remains unsettled, evidence indicates that appropriate use of pulmonary artery
catheters to guide therapy may reduce morbidity. Right heart catheterization is convenient to
perform in the lab, bedside, or the OR. It allows measurement of right atrial, right ventricular,
pulmonary artery and pulmonary capillary wedge pressures, O2 saturation, and CO. These data
may diagnose intracardiac shunts, physiologically significant pericardial disease, and right-sided
valve lesions and can distinguish between pulmonary and cardiac disease.Menodynamic
monitoring may be very helpful in the assessment of treatment of shock, heart failure,
complicated MI, respiratory failure, and post-op hemodynamic instability. Left heart
catheterization is performed to assess the cardiac valves and left ventricular function (also
assessable by echo). The main indications for left heart cath. are for confirmation of the need for
valve surgery and for obtaining coronary angiograms. Increasingly, the catheterization lab is
used for performing coronary interventions.
Anonymous Note: this question seems like an easy one to answer – NOT!! I looked in several
books, but most of my answers came from Ravel chapter 21 and Goroll pgs 114&115 .
Chest X-ray
• Make sure it isn’t a lung problem!!
ECG
• First test to order on your chest pain patient
• Most useful direct test available in acute MI
o 50% of acute MIs show unequivocal ECG changes on first ECG
Cardiac Enzymes:
Creatine Kinase (CK)
• Useful in determining if patient had an MI
• Total CK drawbacks
o Relatively short time period after MI onset during the CK during which CK
enzyme is elevated
o False positive elevations due to skeletal muscle injuries (especially IM
injections)
CK-BB (CK-1)
• Found predominantly in brain and lung
CK MM (CK-3)
• Found predominantly in skeletal muscle
CK MB (CK-2)
• Predominantly found in cardiac muscle
• Level begins to rise 3-6 hours after onset of acute MI
• Peaks 12-24 hours
• Returns to normal in 24-48 hours (sometimes earlier)
• Rough correlation exists between size of infarct and degree of enzyme elevation
Troponin T
• Specific for myocardium
• Level begins to increase in 4-6 hours after onset of acute MI
• Peaks at approx 11 hours (10-24 hour range)
• Returns to normal in ten days or more
o Advantage over CK MB = stays elevated longer
Troponin I
• Specific for myocardium
• Elevates in 4-6 hrs after onset of acute MI
• Peaks at 11 hours
• Returns to normal in about 4 days
Radionuclide Heart Scan
• Scans for acute MI
• Agents localize in acutely damaged myocardium
• Scan not reliable less than 24 hours after onset of infarct
• Returns to normal by 6-7 days post-infarct
That was from Ravel . . . now let’s go to Goroll which is divided by probability of coronary heart
disease

Unstable Angina or PE . . . hospitalize immediately – do not stop to test them with any type of
test!!
At the ER . . .
• Resting ECG
o Look for S-T or T wave changes
If you have changes
• Proceed to coronary angiography
o More cost effective than stress testing in very high probability patients
If you have no changes
• Don’t discharge patient
• Observe X6 hours, then do a stress test
*always check cardiac enzymes too!!

Stable Angina – other tests you can do


Exercise Electrocardiography
o Excellent sensitivity and specificity for detection of high-risk coronary disease
o Low cost
o False-positives occur in younger women
o Sensitivity is lower in less serious forms of CAD
Stress Echocardiography
o Cost is somewhat higher than with ECG, but specificity and sensitivity are better
Radionuclide Imaging with thallium or technetium
o Enhances specificity and sensitivity by giving 3-D view
o Can be performed with treadmill
o High cost
PET scan
o Most sensitive and specific test for coronary insufficiency
o Most expensive
o Limited availability
Anonymous
Resting ECG-
Summary- lay ‘em down and slap some leads on ‘em.
Indications- very-high-probability patients, unstable angina, stable angina (to estimate
prognosis), single CHD risk factor + Hx of atypical chest pain, suspected PE.
Limitations- normal ECG in high-probability patient insufficient evidence for discharge.
Less useful than stress test for stable angina. Use of ECG in suspected PE is marginal and
normal ECG does not rule out PE.
Exercise ECG-
Summary- ECG is obtained while patient exercises.
Indications- a nondiagnostic resting ECG with stable angina or clinically intermediate
probability patient.
Limitations- false-positives in women, less sensitive in setting of less serious forms of
coronary disease, tests cannot be interpreted unless resting ECG is normal.
Stress echocardiography-
Summary- echo obtained while patient exercises on bicycle or after dobutamine
stimulation.
Indications- same as exercise ECG, but greater sensitivity/specificity.
Limitations- higher cost than exercise ECG.
Radionuclide imaging with thallium or technetium sestamibi-
Summary- uses planar scanning and (recently) single-photon emission computed
tomography to provide three-dimensional views. Performed with treadmill exercise or
adenosine injection.
Indications- same as exercise ECG, but greater sensitivity/specificity than exercise ECG
or stress echo.
Limitations- cost and availability.
PET scan-
Summary- more expensive version of above. Greatest sensitivity/specificity.
Indications- same as above.
Limitations- high cost and limited availability generally outweigh benefits of use.
Cardiac enzymes-
Summary- creatinine phosphokinase and troponin 1 are measured. Elevations are
evidence of myocardial damage.
Indications- unstable angina, clinically high-probability or intermediate probability
patients.
Limitations- not discussed in Goroll or Noble.
Chest X-ray-
Summary- for diagnosing PE, causes of pleuritic chest pain, aneurysm.
Indications- suspected PE, pneumothorax, infectious cause of CP.
Limitations- waiting for CXR can delay care- if clinical picture fits, act first, then get
CXR.
Manometry, 24-hour pH monitoring, edrophonium provocation and acid perfusion-
Summary- variety of tests done to evaluate possible esophageal disease.
Indications- suspected esophageal disease in the absence of obvious esophageal signs
(difficulty swallowing, etc.). .
Limitations- they don’t work. Generally unsuccessful at diagnosing esophageal disease.
Pulmonary Angiography-
Summary- gold standard for diagnosis of PE.
Indications- when definitive diagnosis of PE is essential. Urgent and not available from
noninvasive testing.
Limitations- non-invasive testing is generally sufficient in most cases and avoids the risks
inherent in an invasive procedure.
V/Q Scanning-
Summary- widely used in pts. w/ suspected PE. Normal V/Q scan with low pretest
probability of embolization rules out PE. High probability scan + high pretest probability
rules in PE.
Indications- suspected PE with a pretest probability of at least 20%.
Limitations- results are often equivocal. High false-positive rates, esp. in those with
preexisting lung conditions.
D-dimer Testing-
Summary- non-invasive screening for PE. Useful in ruling out PE ( neg. D-dimer + neg.
venous ultrasound rules out PE).
Indications- suspected PE.
Limitations- low specificity, so positive test requires V/Q scan for confirmation.
Venous ultrasound of LE-
Summary- more specific that V/Q and d-dimer in diagnosing PE. Performing serially
over 1-2 weeks greatly increases sensitivity.
Indications- suspected PE. Especially with moderate pretest probability and inconclusive
V/Q.
Limitations- single study sensitivity only 40%. Less useful for rapid detection.
Helical CT-
Summary- designed to detect emboli-induced pulmonary filling defects by means of a 2-
dimensional volumetric image of the lungs.
Indications- high pretest probability or pt. with poor pulmonary reserve who has both a
nondiagnostic V/Q and negative venous ultrasound.
Limitations- expensive, requires contrast media, lack of long-term data on effectiveness.
Transesophageal ultrasound-
Summary- can confirm dissecting aortic aneurysm.
Indications- suspected dissecting aortic aneurysm.
Limitations- Invasive.
CT with contrast-
Summary- less invasive and faster than angiography for Dx of dissecting aortic
aneurysm.
Indications- suspected dissecting aortic aneurysm.
Limitations- none mentioned.
All material comes from Goroll. Most from chapter 20, but some from chapter 36. Goroll is
much more detailed on stress testing, but I figured 1 ½ pages were enough.
Anonymous Currents p 315-317
A) Chest X-ray- provides information about heart size, the pulmonary circulation.

B) Electrocardiogram- provides more reliable information about chamber size, hypertrophy,


pericardial effusions, valvular abnormalities, and congenital abnormalities. ECG's indicate
cardiac rhythm, reveal conduction abnormalities, and provide evidence of ventricular
hypertrophy, myocardial infarction, as well as ischemia.

Routine chest x-rays and ECG's are not recommended to screen for heard disease and have a
limited role in the follow-up of patients with known heart disease.

C) Echocardiography- provides more precise measurement of left ventricular size, function,


and wall thickness. The actual size of all four chambers can be determined and the morphology
of the cardiac valves can be examined.

D) Doppler Ultrasound- can determine a quantitative estimation of transvalvular gradients and


pulmonary artery pressure as well as a qualitative evaluation of valvular regurgitation and
intraventricular shunts.

E) Color Doppler- shows patterns and directions of flow. Particularily useful in evaluating
congenital heart disease.

Doppler studies typically detect clinically insignificant valvular regurgitation so care should
be taken not to overinterpret these findings.
F) Transesophageal Echocardiography- this is superior to surface echocardiography in
looking at the posterior aspects of the heart. IT is often utilized during surgical procedures and is
quite sensitive at detecting aortic dissections.

G) Stress Echocardiography- used to enhance the information from ECGs and as an


alternative to nuclear medicine procedures. Transient depression of segmental wall motion
during or immediately following exercise suggests ischemia.

H) MRI- currently this provides high resolution images of cardiac and adjacent vascular
structures, making this the preferred procedure to evaluate many cardiac conditions including
pericardial and congenital abnormalities.

I) Cardiac Catheterization & Angiography- the standard test for assessment of many
hemodynamic and anatomic abnormalities of the heart. This procedure is more invasive than the
others but remains invaluable in the management of most patients with congenital, valvular, and
coronary heart disease.

9. Explain the general role of each of the following in the management of


angina pectoris. (Note: you do not have to know the indications and protocols
for angiography and surgery in detail —just have the general concepts down.
Similarly, you don't have to know specific drug names or dosages, just the
general reasons for using each class.)
coronary angiography
coronary artery surgery
sublingual nitroglycerin
long-acting nitrates
beta blockers
calcium channel blockers
aspirin
risk factor modification
exercise
Tim Noble pgs 552-557 and CMDT pgs 341-343
Coronary angiography: Definitve diagnostic procedure for CAD. Visualizes the location and
severity of stenoses.
Coronary artery surgery: CABG consists of suturing segments of saphenous vein between the
ascending aorta and to the coronary arteries distal to their stenotic narrowings. At present, in
most routine cases surgeons attempt to bypass at least one diseased vessel (normally the left
anterior descending coronary artery [LAD]) with an internal mammary artery, since the latter
results in a higher long-term patency rate (80% to 90% at 10 years) compared with venous
bypasses (10% occlusion in the first year, 2% per year for the next 6 years, 5% per year
thereafter). Antiplatelet therapy with aspirin has been shown to improve long-term graft patency
rates, and aggressive lipid-lowering to achieve an LDL cholesterol < 100 mg/dl has been shown
to slow the development of atherosclerosis within bypass grafts.
Sublingual nitroglycerin: Rapidly acting nitroglycerin remains the drug of choice to treat acute
anginal attacks. Sublingual or aerosol nitroglycerin spray typically relieves angina in less than 5
minutes. The major antianginal effect of nitrates is to reduce myocardial oxygen demand. These
agents relax vascular smooth muscle, particularly in the venous circulation at usual dosages.
Since this action reduces venous return to the heart, there is a corresponding decline in left
ventricular volume (a determinant of wall stress), which causes myocardial oxygen consumption
to fall.
Long-acting nitrates: Long-acting nitrates are useful in the chronic prevention of anginal
episodes and are available in oral and transdermal preparations. Low initial dosages should be
used to avoid headache and lightheadedness and can be augmented over time.
Beta blockers: beta-Adrenergic antagonists have become the mainstay of therapy to prevent
effort-induced angina and also have been shown to reduce mortality following MI. The main
antianginal effect of beta-blockers is to reduce myocardial oxygen demand by slowing the heart
rate, reducing the force of ventricular contraction, and lowering blood pressure.
Calcium channel blockers: The calcium channel blockers are effective antianginal agents when
used alone or in combination with beta-blockers or nitrates. They can prevent exertional angina
and are also helpful in patients with episodes of coronary vasospasm. Each drug in this class can
reduce myocardial oxygen requirements and increase myocardial oxygen supply.
Aspirin: Coronary thrombosis has been implicated in the majority of patients with MIs and in
unstable angina. Aspirin, as an antiplatelet antithrombotic agent, has demonstrated a beneficial
role in secondary prevention of coronary events post-MI (see below). Furthermore, a meta-
analysis of 300 studies, including 140,000 patients, has shown improved cardiovascular
outcomes among patients with angina, with prior MI, and following coronary artery bypass graft
(CABG) surgery.
Risk factor modification: Prevents future events and may markedly improve symptomatic
angina.
Exercise: Makes sense, but couldn’t find it.
Kate CMDT, 341
coronary angiography: Selective coronary angiography is the definitive diagnostic procedure
for coronary artery disease. It can be performed with low mortality and morbidity, but the cost
is high, and with non-invasive techniques it is usually not indicated solely for diagnosis. It is
indicated in any pt being considered for coronary revascularization, post revascularization with
continued symptoms and pts with chest pain of uncertain cause or cardiomyopathy of unknown
cause.

coronary artery surgery:

sublingual nitroglycerin: Sublingual nitro is the drug of choice; it acts in about 1-2 minutes.
Nitrates decrease arteriolar and venous tone, reduce preload and afterload, and lower the oxygen
demand of the heart. Repeated tablets may be placed every 3-5 minutes as needed. Sublingual
nitro can be taken prophylactically before activities likely to precipitate angina.

long-acting nitrates: A number of longer-acting nitrate preps are available. Depending on the
med, they are taken 2-4 times daily or as a transdermal patch to maintain nitro levels and
decrease episodes of angina. The main limitation to chronic nitro therapy is tolerance, which
occurs in most pts. Tolerance can be limited by utilizing a regimen of 8-10 hour period per day
without nitrates, usually overnight. Nitrate therapy is often limited by headache, nausea, light-
headedness andhypotension.

beta blockers: Beta-blockers prevent angina by reducing myocardial oxygen requirements


during exertion and stress. This is accomplished by reducing the HR, myocardial contractility
and to a lesser extent BP. The beta-blockers are the only antianginal agents demonstrated to
prolong life in pts with coronary disease (post-MI). Therefore, they should be considered for
first-line therapy in most pts with chronic angina. In US only a few beta-blockers are approved
for angina but all work. Major CI: bronchospastic disease, bradyarrhythmias, and overt heart
failure.

calcium channel blockers: Verapamil, diltiazem, and dihydropyridine group of calcium


blockers agents that prevent angina by reducing myocardial oxygen requirements and by
inducing coronary artery vasodilation. It is unclear whether they improve myocardial blood flow
in most pts with stable exertional angina. In those with coronary vasospasm, they may be
considered agents of choice. Calcium channel blockers should be considered third-line anti-
ischemic drugs in post infarct pts.

Aspirin: Coronary thrombosis is responsible for most episodes of myocardial infarction and
many unstable ischemic syndromes. Several studies have denonstrated the benefit of anti-
platelet drugs following unstable angina and infarction. Therefore, small doses of aspirin should
be prescribed for patients with angina.

risk factor modification: Pts with coronary disease should undergo aggressive risk factor
modification (decreasing BP, strenuous activity, cold temperature exposure and emotional
stress). This approach, with particular focus on lowering LDL cholesterol under 100mg/dL, not
only prevents future events but may markedly improve symptomatic angina.

Exercise: Information obtained from Up to Date Gradual institution of a regular aerobic exercise
program should be encouraged. Exercise can result in a lower oxygen requirement for a given
workload, thereby improving exercise tolerance and a sense of well-being, and reducing
symptoms. An exercise program is also beneficial in the management of risk factors for CHD
such as lipids, blood pressure, obesity, and diabetes mellitus.
Anonymous Goroll 195-201
• coronary angiography
o establishes degree of blockage (and location)
• coronary artery surgery
o revascularization of the heart
• angioplasty
• coronary artery bypass grafting (CABG)
• sublingual nitroglycerin
o provides short-term relief of anginal pain (up to 30 min)
o improvement of exercise tolerance when taken prophylactically
• long-acting nitrates
o more sustained nitrate activity than nitroglycerin
o better anginal prophylaxis
• beta blockers
o reduce the frequency of angina (especially exercise-induced)
o improves exercise tolerance
o prevention of sudden cardiac death
o prolongs survival post MI
• calcium channel blockers
o treatment of chronic stable angina
o reduce frequency of anginal episodes
o prolong exercise tolerance
o decrease need for nitroglycerin
• aspirin
o the only conventional platelet inhibitor proven to reduce cardiac risk
o prolongs survival in unstable angina
o reduces risk of MI in chronic stable angina
• risk factor modification
o reduction in these risk factors: hypercholesterolemia, HTN, DM, emotional stress,
and lack of exercise . . . all proven to reduce risk of heart disease
• exercise
o increase the strength of your heart!! Get in shape!!
Anonymous Goroll, pp. 196-204
coronary angiography: Dye is injected into the bloodstream or heart chamber, and x-rays are
taken of the heart and large blood vessels in the chest. (Lang. of Med., p. 370) Percutaneous
transluminal angioplasty (PTCA) is an alternative to surgical revascularization for patients who
remain unacceptable symptomatic yet do not have high-risk disease. PCTA involves passing a
balloon catheter into a stenosed vessel and inflating the balloon at the site of the narrowing to
widen the lumen. This procedure requires coronary angiography and surgical standby d/t risk of
sudden vessel occlusion and infarction.
coronary artery surgery: Surgical revascularization is the treatment of choice in persons with
high-risk disease. It not only improves symptoms and functional status but prolongs survival and
reduces risk of infarction. coronary artery bypass graft surgery (CABG) is superior to medical
therapy for prevention of infarction and prolongation of survival in stable high-risk anginal
patients, especially if accompanied by LV dysfunction. (High-risk patient: moderate to severe
angina; large amount of myocardium at risk; three-vessel, left-main, or left-main equivalent
disease; and reduced ejection fraction.) Patients at less risk who continue to be limited by their
angina despite a full medical regimen and aggressive Tx of risk factors are reasonable candidates
for revascularization. CABG is preferential Tx for diabetics and those who don’t want/can’t
tolerate repeat surgeries.
sublingual nitroglycerin: (TNG) provides short-term relief of anginal pain (30 minutes) and
improvement in exercise tolerance when taken prophylactically. Onset of action: 30 seconds – 3
minutes. TNG must be taken sublingually d/t denitrified & hepatic inactivation on first pass.
long-acting nitrates: (isosorbide dinitrate) provides sustained nitrate activity for good anginal
prophylaxis. Onset of action: 15-30 minutes. Duration of action: up to 4 hours. tid dosing with 8
hours off to minimize risk of nitrate tolerance. (isosorbide mononitrate) more convenient control
of chronic stable angina. Duration of action: 12 hours. Cost: 10x more than dinitrate. (also
ointment and patches).
beta blockers: reduce frequency of angina (especially exercise induced) and increases exercise
tolerance. Proven to prevent sudden cardiac death and prolong survival in patients surviving MI.
In patients with stable coronary dz, they reduce the frequency of silent ischemia and other
coronary events. Their benefits are attributed to their lowering of myocardial oxygen
consumption – they reduce contractility, blood pressure, and heart rate – and they raise the
ventricular fibrillatory threshold. Slowing the HR allows more time for myocardial perfusion
during diastole.
calcium channel blockers: They reduce frequency of anginal episodes, prolong exercise
tolerance, and decrease the need for nitroglycerin. They are especially useful in persons with
coronary vasospasm. Symptomatic benefits only – have not demonstrated any reduction in rates
of MI, need for revascularization, cardiac sudden death, or overall mortality. Shorter acting
preparations may increase risk of MI and cariac death especially in the setting of LV failure.
Currently, their use should be limited to patients who cannot tolerate beta-blockers, or if beta-
blockers & nitrates fail to achieve adequate control of symptoms.
aspirin: only conventional platelet inhibitor that reduces cardiac risk. ASA prolongs survival in
patients with unstable angina and reduces the risk of MI in persons with chronic stable angina.
risk factor modification: (hypercholesterolemia, HTN, smoking, DM, sedentary lifestyle, &
emotional stress) Patients with coronary Dz have 5 times the risk of coronary event. Secondary
preventive efforts can improve quality of life, reduce risk of coronary events, obviate the need
for revascularization, and prolong survival.
exercise: can significantly improve functional status and directly and indirectly improve skeletal
muscle efficiency, decrease heart rate & blood pressure, and enhances moral well-being.
Anonymous Currents p 340-350
A) Coronary Angiography- this is a definitive diagnostic procedure for coronary artery
disease. This is an invasive and expensive procedure and should not be performed solely for
diagnosis. This images the lumen of the vessel only, so it may underestimate atherosclerosis
since it is not specific for vessel wall thickening and cannot determine the amount of wall
reorganization due to atherosclerosis.

B) Coronary Artery Surgery- patients may be subject for revascularization surgery if they
have advanced angina that is not susceptible to pharmacologic therapies. These patients often
have severe enough angina that exercise is not possible due to its propensity to initiate an attack
of the angina. The degree of vessel involvement and severity are weighed prior to initiating a
revascularization procedure.

C) Sublingual Nitroglycerin- this is a fast acting agent that decrease arteriolar and venous tone
utilized in the treatment of an acute attack. This action reduces preload, afterload, and oxygen
demands of the heart. This can be used prophylactically prior to activities known to cause
angina as well.

D) Long-acting Nitrates- these deliver nitroglycerin at a predetermined rate over long periods
of time for prevention of attacks of angina in patients with more advanced CAD. This is a longer
acting nitrate and has some problems with tolerance.

E) Beta-blockers- these prevent angina by reducing myocardial oxygen demands during


exertion and stress. This is accomplished by the reduction of heart rate and myocardial
contractility as well as to a lesser extend blood pressure. These are the only agents proven to
prolong life in patients with CAD post myocardial infarction and should be considered a first line
defense for patients with chronic angina.

F) Calcium Channel Blockers- these drugs prevent angina by reducing myocardial oxygen
demands by inducing coronary artery dilation. Myocardial oxygen demand is lessened by
reducing the blood pressure and the left ventricular stress.

G) Aspirin- this is an agent that helps many ischemic events by inhibiting platelet aggregation
making the blood less viscous. This allows the blood to flow more freely and with less
resistance through coronary arteries that are partially occluded or narrowed.
H) Risk Factor Modification- this has an aggressive approach at lowering LDL cholesterol
below 100. Patients have been shown to have fewer subsequent ischemic events with this lipid
lowering therapy. This is accomplished with medication as well as diet modification.

I) Exercise- this conditions the heart and cardiovascular system as well as increasing muscle
tone which increases the venous return to the heart inadvertently reducing the venous pooling
and back pressure that increased the force required of the heart to move blood through the
vasculature.

10. Differentiate between common important causes of acute nonpleuritic chest


pain. Dropped 2004
Tim CMDT 299, Noble 164-168, and 5 min clinical consult palm program
There was no such category in any of the books, so here’s what I got.

Pleuritic chest pain = pain due to acute pleural inflammation is caused by irritation of the
parietal pleura. Pain is localized, sharp and fleeting and is made worse by cough, sneezing, deep
breathing or movement.

So, nonpleuritic chest pain would be:


MI, Angina (stable and unstable), CAD, Valvular heart disese, Aortic dissection,
Psychogenic and Pulmonary embolism that has not yet infarcted (pulmonary infarct will
cause pleuritic chest pain).
For differentiation of each refer to #6.
Kate CMDT Ch.10
I intend to make a chart with signs/symptoms and PE findings for each, check back on
Sunday evening;
Potentially life-threatening Non Life-Threatening
Common: Musculoskeletal
Acute MI Esophageal pain (reflux,spasm)
Unstable Angina Pericarditis
Pulmonary Embolism Pneumonia
Aortic Dissection Mitral valve prolapse
Pneumothorax

Uncommon:
Tension Pneumothorax Herpes Zoster
Cardiac Tamponade Cervical/ thoracic radicular pain
Esophageal Rupture
Anonymous Goroll, 109
Cardiac
○ CAD presenting as angina pectoris
○ MI
○ Variant or atypical angina
○ Mitral valve prolapse (presents as “atypical chest pain”)
○ Pericarditis (can also present as pleuritic)
Aortic
o Dissecting aortic aneurysm
Esophagus
o Acid reflux
o Motor dysphagia
Other GI sources
○ Cholecystitis
○ Pancreatitis
○ Peptic ulcer disease
Psyche
○ Anxiety, depression, etc.
Noncardiac chest pain of unknown etiology . . . when all else fails!!
Anonymous Noble pg
Angina Pectoris—caused by impaired myocardial blood supply from coronary artery Dz
including aortic stenosis.
Hallmarks sudden onset with exertion, emotional stress or eating a large meal and its relief in
minutes by rest or nitro Pain described as squeezing, heaviness or pressure, it may be burning or
sharp. Radiation to the neck back, jaw shoulder abdomen or arm are common and may present
in the absence of chest symptoms. The arm is reported to feel numb or tingling. Also
diaphoresis and nausea, and dyspnea.
Aortic dissection—a must not miss cause of severe chest pain or interscapular pain, maximal
from the start and tearing or ripping in quality. It begins in the chest and may radiate to the
interscapular region, neck, jaw, lower back, or even down into the legs. Symptoms include
neurological deficits from cutoff of blood supply to the brain, spinal cord, or limb. Loss or
diminution of a major peripheral pulse is a key finding, also new onset of aortic insufficiency
and pericardial tamponade due to dissection into the aortic root.
Esophageal pain—mimickes anginal chest pain. Persists as a dull sensation for several hours
after an acute attack and may occur with swallowing. Sometimes radiates to the interscapular
region. Pain can occur spontaneously or in the context of meals or acid reflux. Some report
dysphagia.
Other GI tract Sources—acute cholecystitis may resemble angina by producing substernal
chest pain that responds to nitrates which reduce cystic duct spasm. Pancreatitis or peptic ulcer
disease as well as gaseous distention of the bowel in the area of the splenic flexure can cause
precordial discomfort.
Psyche—people with anxiety or depression often describe feelings of chest heaviness or
tightness that can last for hours to days with unrelated exertion and unrelieved by rest. They may
complain of the inability to take a deep breath. Hyperventilation causes hypocapnia resulting in
light-headedness and tingling extremities. Cardiac neurosis—may lead to reports of chest pain
mimicking angina. Also lookout for personality disorder and malingering.
Anonymous Goroll p 111
A) Musculoskeletal Disorders- muscle strain
B) Skeletal Disorders- rib fracture
C) Neurologic Disorders- herpes zoster/ nerve root compression
D) Pericarditis
E) Myocardial Ischemia
F) Prolapsed Mitral Valve
G) Disecting Aortic Aneurysm
H) Esophageal Disorders- reflux or spasm
I) Other GI Disorders- PUD, Pancreatitis, Functional Motility Disorders, Cholecystitis
J) Psychogenic- anxiety/panic disorders, depression
11. Differentiate between patient presentation and common important causes of
acute pleuritic chest pain.
Zen Seeker
PLEURITIC CHEST PAIN -
implies that the pain varies with the respiratory cycle and is frequently exacerbated during
inspiration and coughing. Pleuritic chest pain is typically sharp and unilateral

workup three acute causes of pleuritic chest pain -


• acute pulmonary embolism
• pneumothorax
• acute pericarditis.
Kim R. CMDT 299 Goroll 112-113
- caused by irritation of the parietal pleura.
- In young, otherwise healthy individuals, pleuritis is usually caused by viral respiratory
infections or pneumonia. Rib FXs may cause severe pleurisy. Trauma is also a cause.

Pleural Effusion
-transudates- increased capillary fluid production due to increased hydrostatic or
decreased oncotic pressure.
-exudates- increased fluid production due to abnormal capillary permeability.
Decreased lymphatic clearance of fluid from the pleural space.
-emphyema- infection in the pleural space.
-hemothorax- bleeding into pleural space.

Precordial catch syndrome- is suggested by brief self-limited episodes in an otherwise


health young person.

-pleuritic pain worsened by turning, but relieved by sitting up and leaning forward is
indicative of pericarditis.

-cancer is also another cause.


Kate CMDT 299
Pain due to pleural inflammation is caused by irritation of the parietal pleura (the visceral pleura
is innervated by visceral afferents, hence doesn't cause readily localizied pain when inflammed).
Such pain is localized, sharp, and fleeting and made worse by coughing, sneezing, deep
breathing or movement. When the central portion of the diaphragmatic parietal pleura is
irritated, pain may be referred to the ipsilateral shoulder. There are numerous causes of
pleuritis. The setting in which pleuritic pain develops helps to narrow the differential diagnosis.
In young, otherwise healthy individuals, pleuritis is usually caused by viral respiratory infections
or pneumonia. The presence of pleural effusion, pleural thickening, or air in pleural spaces
requires further diagnosis. Pleural effusion: abnormal accumulation of fluid in pleural spaces
usually seen in patients witth existing cardiopulmonary diseases. Causes are numerous, see
CMDT 300.
Pneumothorax: accumulation of air in pleural space. Can be secondary to trauma, either
penetrating or blunt. Primary spontaneous pneumo occurs in absence of underlying lung disease,
typically affects tall, thin boys and men ages 10-30 years old. It is thought to occur from rupture
of subpleural apical blebs in response to high negative intrapleural pressures. Secondary
pneumothorax occurs as a complication of COPD, asthma, Cystic Fibrosis, TB, pneumonia,
menstruation (Zebra!!) and wide variety of interstitial lung diseases (ie: sarcoidosis).
Anonymous Goroll, 109
Inflammation or distension of the pleura produces true “pleuritic pain”. It is worsened by deep
inspiration and cough, but relatively unaffected by movement or palpation.
Causes:
○ infection
○ pulmonary infarction
○ neoplasm
○ uremia
○ connective tissue disease
Anonymous Goroll: Primary Care Medicine, 4th ed., Ch 000, pp. 112-113 electronic edition
Pain worsened by deep inspiration or cough is a hallmark of pleural irritation, but such pain is
also suggestive of pericarditis and chest wall pathology. Even aortic dissection may cause pain
worsened by movement due to respiration. Focal chest wall tenderness worsened by movement
quickly narrows the differential to a chest wall origin. In the absence of focal chest wall pain,
one needs to search promptly for evidence of intrathoracic pathology. Inquiry is needed into
fever, cough, sputum production, tuberculosis exposure, hemoptysis, smoking, HIV exposure or
high-risk behavior, unilateral leg edema, calf tenderness, shortness of breath, past history of
embolization, recent orthopedic surgery, and oral contraceptive use. Pneumothorax should come
to mind when pleuritic pain is sudden in onset and accompanied by dyspnea in a young patient
with a previous history of pneumothorax or when the patient has long-standing bullous
emphysema. Precordial-catch syndrome is suggested by brief self-limited episodes in an
otherwise healthy young person. Pleuritic pain worsened by turning but relieved by sitting up
and leaning forward is indicative of pericarditis, which can be further assessed by physical
examination. Attention to the context of the patient's pleuritic pain often suggests the diagnosis.
Onset in a person with known metastatic cancer may be due to a pathologic fracture, pleural
metastasis, or pulmonary embolization. The same pain in an otherwise healthy young person
with new onset of a dry cough, low-grade fever, and myalgias is consistent with viral-induced
pleurodynia and muscle soreness from coughing.
Anonymous Goroll p 111
A) Pleurisy of any origin
B) Pneumothorax
C) Pulmonary Embolism
D) Pneumonitis
E) Bronchiospasm

II. Cardiac Diseases (Top of Document)

1. Explain the role and limitations of the following enzymes in making the
diagnosis of acute myocardial infarction, including recognition of their initial
and peak elevation times.
LDH Dropped 2004
CK and CK-MB
Troponin-I & T
Megan Mosby 308
LDH: This intracellular enzyme (lactose dehydrogenase) is found in the cells of many body
tissues, especially the heart, liver, RBC’s, kidneys, skeletal muscle and lungs. Because it is
widely distributed the total LDH is not a specific indicator of any one disease or indicative of
injury to any one organ. Five separate fractions, isoenzymes, make up the total LDH. Each
tissue contains a predominance of one or more LDH enzymes. In general isoenzyme LDH-1
comes mainly from the heart. Isolated elevation of LDH-1 above LDH-2 indicates myocardial
injury. With myocardial injury the serum LDH level rises within 24-48 hours after MI, peaks in
2-3 days and returns to normal in approximately 5-10 days. This makes the serum LDH level
especially useful for a delayed diagnosis of MI. (e.g. When a patient complains of having chest
pain 4 days earlier) LDH-1 may become obscured if the patient has other existing problems, e.g.
pulmonary infarction or CHF.

Megan Mosby 190-195


CK and CK-MB: CPK is found predominantly in the heart muscle, skeletal muscle, and brain.
Serum CPK levels are elevated when these muscle or nerve cells are injured. CPK levels can
rise within 6 hrs after damage. If damage is not persistent, the levels peak at 18 hours after
injury and return to normal in 2-3 days. The CPK-MB isoenzyme portion appears to be specific
for myocardial cells. CKP-MB levels rise 3-6 hrs after infarction occurs. If there is no further
myocardial damage the levels peaks at 12-24 hours and returns to normal 12-48 hrs after
infarction. CPK-MB levels do not usually rise with transient chest pain caused by angina,
pulmonary embolism, or congestive heart failure. The CPK-MB isoenzyme level is helpful in
both quantifying the degree if MI and timing the onset of infarction.

Megan Mosby 464-466


Troponin I & T: This test is performed on patients with chest pain in order to determine if the
pain is due to cardiac ischemis. It is a specific indicator of cardiac muscle injury. It is also
helpful in predicting the possibility of future cardiac events. Troponins are biochemical markers
for cardiac disease. This test is used to assist in the evaluation of patients with suspected acute
coronary ischemic syndromes. They can be used to predict the likelihood of future cardiac
events. There are 2 cardiac specific troponins: cardiac troponin T and cardiac troponin I.

Because of their high specificity for myocardial cell injury, cardiac troponins are very useful in
the evaluation of patients with chest pain, their use is similar to CPK-MB, except that troponins
are more specific. They become elevated sooner and remain elevated longer than CPK-MB.
Cardiac troponins become elevated as early as 3 hrs after MI, and troponin T may remain n
elevated for 7-10 days after MI, and troponin I may remain elevated 10-14 days. However, if
reinfarction is considered, troponins are not helpful because they could be elevated from the first
ischemic event.

Timing of Appearance and Disapperance of Commonly Used Cardiac Enzymes


Enzyme Start to Peaks Returns to normal
rise
Total CPK 4-6 hrs 24 hrs 3-4 days
CPK-MB 4 hrs 18 hrs 2 days
LDH 24 hrs 72 hrs 8-9 days
Troponin 4-6 hrs 10-24 hrs 10 days
T
Troponin I 4-6 hrs 10-24 hrs 4 days
Stephen Mosby’s
LDH---enzyme found in the cells of many body tissues, especially the heart, liver, RBCs,
kidneys, sckeletal muscle, brain, and lungs. Total LDH is not a specific indicator of any one
disease or indicative of injury to any one organ. Although, LDH-1 comes mainly from the heart.
Isolated elevation of LDH-1 (above LDH-2) indicates myocardial injury.
Serum LDH level rises within 24-48 hours after an MI, peaks in 72 hours.
CMDT pg 349 The most valuable laboratory tests are cardiac-specific markers of myocardial
damage, including quantitative determination of CK-MB, Troponin I, and Troponin T.
All are relatively specific for necrosis (the Troponins somewhat more so), though they may be
elevated following severe ischemic episodes. Each of these tests may become positive as early as
4-6 hours after the onset of an MI and should be abnormal by 8-12 hours. Circulating levels of
Troponins may remain elevated for 5-7 days or longer and should obviate the use of less specific
LDH isoenzyme assays.
Mosby’s CPK-MB (specific for myocardial cells), levels rise 3-6 hours after MI and peaks at 12-
24 hours.
Total CPK starts to rise 4-6 hours and peaks at 24 hours.
Cardiac specific Troponin T (cTnT), starts to rise 4-6 hours and peaks at 10-24 hours.
Cardiac specific Troponin I (cTnI), starts also 4-6 hours and peaks at 10-24 hours.
Cardiac Troponins are new and promising biochemical markers for cardiac disease. Cardiac
Troponins are more specific for cardiac disease and can be separated from skeletal Troponins by
the use of monoclonal antibodies or enzyme-linked immunosorbent assay (ELISA).
Anonymous
LDH Lactic dehydrogenase (LDH) values refer to total serum LDH. Total LDH levels are
elevated at some time in 92%-95% (literature range, 82%-100%) of patients with acute MI.
Statistics for sensitivity in acute MI refer to multiple sequential LDH specimens and are
therefore not valid for any single determination. In acute MI, LDH becomes elevated 24-48
hours after MI, reaches a peak 48-72 hours after MI, and slowly falls to normal in 5-10 days.
Thus, LDH values tend to parallel AST values at about double the time interval. Total LDH is
slightly more sensitive than AST in acute MI and is reported to be elevated even in small infarcts
that show no AST abnormality. LDH is found in many organs and tissues. Since LDH fraction
1 (fraction 1 is found mainly in RBCs and in heart and kidney) is contained in red blood cells
(RBCs) as well as cardiac muscle, LDH is greatly influenced by accidental hemolysis in serum
and thus must be collected and transported with care. Heart valve prostheses may produce
enough low-grade hemolysis to affect LDH, and LDH levels are also abnormal in many patients
with megaloblastic and moderate or severe hemolytic anemias. Skeletal muscle contains LDH, so
total LDH (or even hydroxybutyric acid dehydrogenase [HBD]) values are not reliable in the first
week after extensive surgery. LDH levels may be elevated in 60%-80% of patients with
pulmonary embolism (reports vary from 30%-100%), possibly due to pulmonary tissue damage
or to hemorrhage. Finally, LDH becomes elevated in some patients with malignant neoplasms
and leukemia, and in some patients with uremia.The major drawback of total LDH, similar to
AST, is the many conditions that can elevate LDH values. In acute MI the typical pattern is
elevation of LDH-1 values with LDH-1 values greater than LDH-2.

CK & CK-MB: Creatine kinase (CK) is found in heart muscle, skeletal muscle, and brain. It is
elevated at some time in about 90%-93% (literature range, 65%-100%) of patients with acute MI.
In acute MI, CK behaves similarly to AST. In addition, elevations have also been reported in
myocarditis and also in some patients with tachyarrhythmias (mostly ventricular) for unknown
reasons. Acute liver cell damage, which frequently causes an abnormal AST value, has no effect
on CK. This is an advantage, since the situation often arises in which an elevated AST (or LDH)
level might be due to severe hepatic passive congestion from heart failure rather than from acute
MI. . A considerable number of conditions associated with acute muscle injury or severe muscle
exertion affect CK values. Thus, CK values are usually elevated in muscle trauma, myositis,
muscular dystrophy, after surgery, postpartum, after moderately severe exercise (e.g., long-
distance running), and in delirium tremens or convulsions. Increased serum values have been
reported in about 80% of patients with hypothyroidism (literature range, 20%-100%) and in
patients with severe hypokalemia, due to changes induced in skeletal muscle. CK elevation can
be due to effects of alcohol on muscle. For example, one study found that CK levels became
abnormal after 24-48 hours in the majority of persons following heavy drinking episodes as well
as in most patients with delirium tremens. Levels of CK are said to be normal in chronic
alcoholics without heavy intake. The major drawbacks of total CK are (1) the relatively short
time period after onset of infarction during which the CK value is elevated and (2) false positive
elevations due to skeletal muscle injury (especially intramuscular injections).

CK-MB (CK-2), found predominantly in cardiac muscle – The CK-MB level begins to rise
3-6 hours after onset of acute MI, reaches a peak in 12-24 hours, and returns to normal in 24-
48 hours (sometimes earlier). There is a rough correlation between the size of the infarct and the
degree of elevation. Since small infarcts may not elevate the MB fraction dramatically, it is
important to time the collection of specimens so as not to miss the peak values. Some
recommend five specimens: one immediately, then at 6, 12, 18, and 24 hours afterward. Others
use 4 specimens: one immediately, then at 8, 16, and 24 hours or at 6, 12, and 18 hours. Some
use 3 specimens: immediately, 12 hours, and 24 hours. Some laboratories do not perform CK-
MB assay unless the total CK value is elevated. However, reports indicate that about 10% of
acute MI patients (literature range 0%-16%) demonstrate elevated CK-MB levels with the total
CK value remaining within reference range limits.

Troponin I & T (also refer to the “Improved Laboratory Diagnosis” article in your syllabus)
Troponin-I is a regulatory protein in the troponin cardiac muscle complex. This protein, like
troponin-T, is specific for myocardium. The troponin-I level becomes elevated after onset of
acute MI in about 4-6 hours (range, 3-19 hours), peaks at about 11 hours (range, 10-24
hours), and returns to reference range in about 4 days (range, 3.5-6 days). Therefore, troponin-
I behaves much like troponin-T, except that return to reference range is faster. Response to acute
MI is also somewhat similar to CK-MB. However, troponin-I may become elevated a little
sooner than CK-MB in the first 4 hours after onset of acute MI (in one study, troponin-I was
elevated at or before the fourth hour in 44% of MI patients, whereas when using CK-MB only
17% had elevated levels). Troponin-T is a regulatory protein located in skeletal and cardiac
muscle fibers. Skeletal muscle and myocardium have different forms of this protein, so that the
antibody that detects cardiac troponin-T in serum is a specific test for myocardial fiber injury.
After onset of acute MI, the troponin-T level begins to increase in about 4-6 hours, peaks at
about 11 hours (range, 10-24 hours), and returns to reference range in 10 days or more.
Therefore, Troponin-T behaves like CK-MB but remains elevated much longer.
Anonymous
Role Limitations Initial
Onset/Peak
Times
LDH elevated in 92-95% -found in many organs 24hrs / 3 days
(total) of acute MIs -elevated under many conditions;
-affected by hemolysis
CK (total) elevated in 90-93% -affected by trauma or hypoxia of 4-6hrs / 24hrs
of acute Mis skeletal muscle;

CK-MB found predominantly -relative index must be considered 4hrs / 18hrs


(CK2) in cardiac muscle -critical timing of speciman collection
Troponin-I specific to -all I could discern is possilbly the 4-6hrs / 11hrs
myocardium relatively small window of time in
Troponin- which to collect + detect peak levels 4-6hrs / 11hrs
T specific to
myocardial fibre
injury
* see table 21.1 for reference ranges
Anonymous
A) Lactic Dehydrogenase (LDH)- this enzyme is found in many body tissues especially the
heart, liver, RBCs, kidneys, skeletal muscle, brain and lungs. Because the LDH is widely
distributed throughout the body, the total LDH level is not specific to any one disease. The
isozyme LDH-1 comes mainly from the heart and with myocardial injury the serum levels rise
within 24-48 hours of infarction with peaks in 2-3 days.
Pagana p 308-309

B) Creatine Phosphokinase (CPK)- this is an enzyme found in the heart muscle, skeletal
muscle and the brain. Serum levels of this enzyme become elevated when there is injury to these
cells. Levels can rise within 6 hours after damage and levels peak at about 18 hours after injury
and will return to normal levels within 2-3 days.
1. CPK-MB- this is an isozyme of this family specific for myocardial cells. The levels
rise within 3-6 hours after infarction and if there is no continuing damage occurring, the level
will peak within 12-24 hours and will return to normal 12-48 hours after infarction. This is
useful in quantifying the degree of myocardial infarction and timing of the onset of infarction.
Pagana p 190-191

C) Troponins- these are proteins that exist in skeletal and cardiac muscle that regulate the
calcium-dependant interaction of myosin with actin for the muscle contractile apparatus. There
are two cardiac specific troponins; Troponin T and Troponin I. Cardiac troponins are more
specific to myocardial muscle infarction than is CPK-MB, and they will always be normal in
noncardiac muscle disorders.
1. Troponin T- May become elevated within 3 hours after infarction and may remain
elevated for 7-10 days
2. Troponin I- May become elevated within 3 hours after infarction and may remain
elevated for 10-14 days
Pagana p 465

2. Explain the general use and limitations of radionuclide heart scanning.


Megan Mosby 742-744
Cardiac nuclear scanning is used to detect myocardial ischemia, infarction, wall dysfunction and
decreased ejection fraction. It is commonly used as the imaging method portion of cardiac stress
testing.
Cardiac radionuclear scanning is a noninvasive and safe method of recognizing alterations of left
ventricular muscle function and coronary artery blood distribution. In evaluating the patency of
the coronary arteries, the characteristic abnormality varies according to the type of
radiocompound used.
Stephen CMDT pg 350
Radionuclide angiography demonstrates akinesis or dyskinesis in areas of infarction and also
measures ejection fraction, which may be valuable.
Mosby’s With injected Technetium-99m sestamibi (isonitrite), IV, high quality images can be
obtained on the first pass, providing information similar to angiography. Perfusion images,
ventricular function, and gated-pool ejections can be all obtained with a single injection. This is
often called myocardial perfusion scan. Furthermore, with the use of isonitrite, an ischemic area
will be visible several hours after an eschemic event.
When eschemia or early infarction have occurred, intracellular CA++ leaks out of the cardiac
muscle cells. The CA++ level in the area of injury is very high.
The injected technetrium-99m pyrophosphate binds to that CA++ and creates an area of
increased radionuclide uptake (hot spot). This is often called a myocardial infarction scan. With
the radionuclides discussed, cardiac nuclear scanning is used to indicate myocardial ischemia or
infarction. It can be used in acute events of chest pain.
Contraindications--- Patients who are uncooperative or pregnant.
Anonymous
Heart scanning can now be performed in two ways. Scan for acute MI is done with technetium
pyrophosphate or certain other radiopharmaceuticals. These agents localize in acutely damaged
myocardium, producing a focal area of increased radioactivity. The scan is not reliable less than
24 hours after onset of infarct and returns to normal by 6-7 days after infarct. Best results are
obtained with transmural infarcts and with locations in the anterior and lateral walls of the left
ventricle. Subendocardial infarcts, especially small ones, are much more likely to be missed.
Another consideration is the necessity of transporting the patient to the scanner, unless the
institution is one of the few that have a portable unit. Combined use of CPK and LDH isoenzyme
determinations has lessened scan necessity in diagnosis. Heart scanning may, however, assist in
the diagnosis of acute MI that occurs during or after cardiac surgery, when enzyme or isoenzyme
diagnosis is not reliable. Some additional areas of difficulty in myocardial scanning include the
fact that size of the scan abnormality cannot at present be reliably correlated with infarct size.
Ventricular aneurysms may concentrate the phosphate radiopharmaceuticals in a manner
suggesting infarct, and this unexplained capability may persist for years. Heart scanning can also
be done by using radioactive elements, such as thallium, that localize in viable myocardium.
Scars and infarcts are seen as areas without uptake. Areas of ischemia (if sufficiently large) also
may be detected by scanning before and after exercise. The optimal scan time for acute MI is
within 6 hours after onset of chest pain. After 24 hours, reports indicate that as many as 25% of
patients may not demonstrate a lesion. Therefore, optimal scan times for thallium and for
pyrophosphate are quite different. Ultrasound imaging of the heart (echocardiography) in two-
dimensional B-mode is now widely used to visualize cardiac ventricle wall motion. A focal
motion defect is suggestive of myocardial damage, although acute injury and scar cannot always
be differentiated. This imaging technique is attractive because it is relatively easy to do with
present-day equipment; is relatively inexpensive; and has good sensitivity compared to other
scanning modalities. In addition, left ventricular ejection fraction (used as a parameter of left
ventricular function) can be estimated. However, like ultrasound in general, much depends on
operator scanning technique and interpretation. M-mode ultrasound can also be used to evaluate
small ventricular wall areas.
Anonymous Ravel p. 340
Heart scanning may assist in the diagnosis of acute MI that occurs during or after cardiac
surgery, when enzyme or isoenzyme (combined use of CPK and LDH) diagnosis is not reliable.
Scan for acute MI is done with technetium pyrophosphate or certain other radiopharmaceuticals.
These agents localize in acutely damaged myocardium, producing a focal area of increased
radioactivity.
Limitations:
-The scan is not reliable less than 24 hours after onset of infarct and returns to normal by 6-7
days after infarct.
-The necessity of transporting the patient to the scanner.
-Size of the scan abnormality cannot correlate with infarct size.
Anonymous Noble p 550
A) Exercise Radionuclide Ventriculography- this entails imaging blood flow through the left
vertricle during the cardiac cycle at rest and then with exercise. In normal individuals,
contractile function of the left ventricle increases with exercise. Myocardial ischemia is
suggested if the ejection fraction falls with exercise or if segmental left ventricular wall motion
abnormalities develop. This procedure has a high sensitivity but its specificity is not as good
because other etiologies of the left ventricular dysfunction can produce similar results.

III. Congestive Heart Failure (Top of Document)

1. Define "congestive heart failure."


Jennyb Noble 578
CHF aka heart failure is a clinical syndrome resulting from cardiac decompensation and
characterized by signs and symptoms of interstitial volume overload and/or inadequate tissue
perfusion. Heart failure occurs when the heart is unable to pump blood at a rate sufficient to
meet the metabolic demands of the body, or when it can only do so with an increased filling
pressure.
Anonymous Noble, 578
Heart failure, often referred to as congestive heart failure, is a clinical syndrome resulting from
cardiac decompensation and characterized by signs and symptoms of interstitial volume overload
and/or inadequate tissue perfusion.
Anonymous Noble, Ch 65, pp 578
Heart failure, often referred to as congestive heart failure, is a clinical syndrome resulting from
cardiac decompensation and characterized by signs and symptoms of interstitial volume overload
and/or inadequate tissue perfusion. In pathophysiologic terms, heart failure occurs when the
heart is unable to pump blood at a rate sufficient to meet the metabolic needs of the body or
when it can do so only with an elevated filling pressure.
Anonymous Noble p 578
A) Congestive Heart Failure- a clinical syndrome resulting from cardiac decompensation and
characterized by signs and symptoms of interstitial volume overload and/or inadequate tissue
perfusion.

2. Describe the basic pathologic mechanisms which may result in heart failure
and list one or two common examples of each.
Jennyb Noble 581
Ventricular dilatation occurs with chronic volume overload, and ventricular hypertrophy occurs
with chronic pressure overload
Anonymous Noble, 584, table 65-3
Contractile dysfunction
• ischemic heart disease
• dilated cardiomyopathy
pressure overload
• HTN
• Aortic stenosis
• Aortic coarctation
Volume overload
• Aortic regurgitation
• Mitral regurgitation
Impaired Filling
• Valvular dysfunction
○ Mitral stenosis
○ Tricuspid stenosis
• Diastolic dysfunction
○ Hypertrophic cardiomyopathy
○ Amyloidosis
○ Constrictive pericarditis
Increased demand
• Anemia
• Thyrotoxicosis
• Thiamine deficiency
• Paget’s disease
Anonymous Gorroll, pg 182-183
The “congestive” manifestations of CHF—leg edema, orthopnea, paroxysmal nocturnal dyspnea
(PND), rales, jugular venous distention—represent elevations in right or left ventricular filling
pressures. Traditionally, filling pressure elevations have been viewed as a consequence of
systolic dysfunction producing a backup of blood into the pulmonary and systemic venous
systems. The hallmark of systolic dysfunction is a reduced ejection fraction.
However, 40% of patients presenting with CHF appear to have reasonably well-preserved
systolic function but suffer from diastolic dysfunction, manifested by increased resistance to
diastolic ventricular filling—i.e. valvular heart disease, hypertrophic, ischemic and
cardiomyopathic diseases.
An initial fall in CO in a CHF patient activates renin-angiotensin production and sympathetic
discharge, which temporarily help preserve CO, but at the cost of an increase in preload and
afterload. Eventually, the resultant venous htn and increased left ventricular work affect the
failing heart, which cannot tolerate such increases in work, and CO drops. Normal hearts
respond to increased venous return via the Frank-Starling mechanism to increase cardiac output,
but not the failing heart. The result is increased pulmonary and systemic htn and no
improvement in CO.
Right-sided heart failure is easier to document than left-sided. It is defined as a right atrial
pressure greater than 6cm H2O, manifested as a vertical distance from the level of the right
atrium to the top of the jugular venous column that is greater than 6cm. Ankle edema and JVD
may (but not necessarily) indicate CHF. A hx of orthopnea and PND are suggestive of left-sided
heart failure, as are basilar rales on pulmonary exam. The finding of an S3 is among the more
specific of signs for systolic dysfunction, but is hard to appreciate. Other findings: CXR—
prominent interstitial markings, upper zone redistribution, cardiomegaly, perihilar haziness,
Kerley “B” lines. Left atrial enlargement by ECG criteria in conjunction wth cardiomegaly by
CXR also have diagnostic value. Echo to determine LV function.
Anonymous Currents p 380
A) Systolic function of the heart is governed by four major determinants; the contractile state of
the myocardium, the preload of the ventricle, the afterload of the ventricle, and the heart rate.
Dysfunction in any of these four areas can result in heart failure.
1. Contractility- contractility can be decreased as a result of loss of functional muscle
due to infarction or by any process that diffusely affects the myocardial muscle.
2. Preload- if the preload is excessively elevated the heart will stretch too far and not be
able to contract adequately as the muscle fibers lose association. This can result from valvular
regurgitation.
3. Afterload- If the afterload is excessive, again the ventricles stretch too far as they are
filled and the muscle fibers lose association reducing the contraction. This can result from
valvular stenosis or severe systemic hypertension.
4. Heart Rate- pump function can be affected if the heart rate is too slow or too fast.

The normal heart can tolerate wide variations in preload, afterload, and heart rate but the
diseased heart lacks the reserves for such alterations.

3. Compare and contrast the following terms as they relate to CHF:


preload,
afterload,
contractile (inotropic) state of the myocardium, Dropped 2004
ejection fraction,
cardiac output and stroke volume.
Jennyb Noble 580, Tabers, and Martini 553
Preload: passive stretch on the myocardium just prior to contraction, or the ventricular wall
tension at the end of diastole. Preload corresponds to the end-diastolic volume.
Afterload: Ventricular wall stress during systole or the force that the ventricle must overcome
to eject its contents.
Contractility or inotropic state is a fundamental property of the myocardium that determines
the strength of contraction.
Ejection fraction: The percentage of the blood emptied from the ventricle during systole.
Ejection fraction average is 60 -70% in healthy hearts.
Cardiac Output: The amount of blood pumped by the left ventricle each minute
Stroke Volume: The amount of blood ejected during a single beat.
Anonymous Noble, 580
• preload,
o In CHF, the body tries to increase the pre-load to compensate for a decreased
cardiac output
• afterload,
o the major determinant of afterload is systemic vascular resistance. Afterload is
typically increased in systemic HTN and aortic stenosis, leading to left-sided heart
failure
• contractile (inotropic) state of the myocardium,
o determines strength of cardiac contraction
o difficult to directly measure, so it is often assessed by assessing pre-load and
afterload
• ejection fraction,
o used to evaluate LV function
• cardiac output and stroke volume.
o Will both be reduced in CHF if the body cannot otherwise compensate
Anonymous
Preload- At its simplest, it is end diastolic pressure within the ventricles. This is the resting
pressure in the ventricles. The R. ventricle corresponds to central venous pressure and the L.
ventricle corresponds to a lower pressure within the pulmonary circulation. This is normally a
good thing, because Frank Starling found that if you pre-tension, or stretch the myocardium
before systole, the myocardium contracts with more force from the rebound of the pre-tensioned
ventricular walls. This increases cardiac output. As with anything, too much of a good thing is
ultimately bad. Beyond a certain point, the myocardium loses its contractility and force of
contraction is lost. With CHF, if the myocardium is damaged, even normal pre-load can be too
much.
Afterload- The pressure on the ventricular walls during systole, or the force the ventricular
walls have to overcome to eject its contents. Aortic or pulmonary stenosis can increase afterload
by increasing pressures needed to eject volume past the stenosis. Systemic or pulmonary
hypertension also raises the force needed to eject volume into the respective circulatory system.
IE: systemic vascular resistance.
Contractile State (inotropic)- Fundamental property of the myocardium that determines the
strength of contraction.
At a constant preload/afterload, an increase in contractility results in an increase in extent and
velocity of fiber shortening. This is hard to measure in the clinical setting. In CHF, contractility
can be manipulated with different drugs.
Cardiac output and stroke volume- Stroke volume is volume of blood ejected from L.
ventricle each beat, expressed as liters/beat. Multiply stroke volume by heartbeats per minute and
you get Cardiac Output measured in liters per minute. As related to CHF, any change in preload,
afterload, or contractility can alter stroke volume, which in turn alters cardiac output.
Ejection Fraction- not in assigned reading. Found in Merck manual.
Ejection fraction is a mathematically calculated percentage of the ratio of stroke volume to end
diastolic volume. In plain language, it is the percent of volume of the L. ventricle that is ejected
with each beat. Normally the heart ejects 50-60% of its L. ventricular volume with each beat. As
heart muscle fails, it ejects less volume, until it can no longer circulate enough blood to sustain
life.
Anonymous Currents p 380-381
A) Preload- This refers to the passive stretch on the myocardium just before contraction. This
corresponds to the end diastolic volume or end diastolic pressure within the ventricle. According
to Starling's Law the higher the preload, the greater the force of ventricular contraction and the
greater the stroke volume. In severe heart failure, the ventricular function may not increase with
the increasing preload and the end-diastolic volumes and pressures increase leading to
pulmonary venous congestion. Noble p 580

B) Afterload- This refers to the ventricular wall stress during systole or the force that the
ventricle must overcome to eject its contents. According to Laplace's Law, systolic wall stress is
directly proportional to ventricular pressure and chamber radius and inversely proportional to
ventricular wall thickness. When left ventricular afterload is increased due to aortic vavlular
stenosis or systemic hypertension the ventricle cannot keep up and upon failing as a pump the
systemic circulation becomes more static and peripheral edema results.

C) Contractile (inotropic) state of myocardium- Contractility or inotropic state is a


fundamental property of the myocardium that determines the strength of contraction. At a
constant preload and afterload, an increase in contractility results in an increase in the extent and
velocity of fiber shortening. Circulating catecholamines or positive inotropic agents such as
digoxin may increase contractility. Negative inotropic agents such as calcium channel blockers,
myocyte loss, or acidosis may decrease contractility. Noble p 580

D) Ejection fraction- This is the fraction of the end-diastolic volume that is ejected into
circulation with a ventricular contraction. If the pump begins to fail the ejection fraction will
decline.

E) Cardiac output- This is termed the amount of blood the heart is able to pump each minute
and is equal to the stroke volume multiplied by the heart rate. As the heart begins to fail the
amount of blood being pumped each minute begins to decline.

F) Stroke Volume- This is the volume of blood that is pumped out by the heart with each beat.
If the stroke volume of either ventricle is reduced the volume and pressure within that ventricle
will increase or the afterload will increase. This results in a stretched ventricle and a more
vigorous contraction according to Starling's Law. If this situation is chronic ventricular dilation
will occur to restore the resting cardiac output however the chronic elevation of diastolic
pressures will be transmitted to the atria, pulmonary and systemic venous systems ultimately
increasing their pressure and resulting in pulmonary and systemic edema.

4. Determine the role of each of the following as compensatory mechanisms in


CHF.
Frank-Starling relationship
ventricular dilitation
ventricular hypertrophy
increased sympathetic nervous system activity
excess salt and water retention
Jennyb Noble 581
Frank-Starlings: When the heart contracts and blood leaves the right ventricle, a comparable
amount of blood arrives at the left atrium, to be ejected by the left ventricle at the next
contraction.
Ventricular dilatation: Occurs with chronic volume overload
Ventricular hypertrophy: Results secondary to chronic pressure overload
Increased activity in the sympathetic nervous system is caused by decreased perfusion pressure
secondary to CHF the decreased pressure is sensed by the carotid sinus and aortic arch
receptors. The receptors cause an increase in sympathetic tone and decrease in parasympathetic
tone.
Reduced cardiac output also causes activation of the renin-angiotensin system. Release of
angiotensin causes an increase in aldosterone which has potent sodium retaining properties.
Remember where there is salt there is H20!!
Anonymous Noble 581-582
• Frank-Starling relationship
o Remember the rubber band theory – the greater the stretch on the rubber band, the
further it flies
o In CHF, the heart will compensate by increasing the amount of blood in the
preload stage
• ventricular dilatation
o the ventricles dilate to allow more blood to fill them thus delivering more blood to
the body
• ventricular hypertrophy
o the muscle builds up in order to move the blood with more force
• increased sympathetic nervous system activity
o the body tries to increase the heart rate to pump blood faster
• excess salt and water retention
○ when the cardiac output decreases, the body thinks it is dehydrated and saves salts
and water to compensate
Anonymous Noble Chp. 65 pp 580-582
Frank-Starling relationship: preload (stretching) corresponds directly to stroke volume
(contraction). In CHF output is maintained (to a point) w/ increased preload by increased
diastolic filling resulting from increased vasoconstriction + intravascular volume expansion.
As failure becomes more severe, these mechanisms become inadequate to maintain sufficient
diastolic pressure resulting in pulmonary venous congestion/pulmonary edema.
ventricular dilation: increased afterload = increased ventricular pressure + chamber radius;
basically, the ventricle stretches out (permanently)to accomadate a larger volume to maintain
cardiac output.
ventricular hypertrophy: increased dilation = increased wall thickness to help maintain
contractile function.
increased sympathetic nervous system activity: reduced cardiac output + increased filling
pressures = decreased perfusion pressure sensed @ carotid + aortic baroreceptors resulting in
increased catacholamine circulation(sympathetic) response.
excess salt and water retention: atrial strech receptors become desensitized to increased
pressures inhibiting sodium + water excretion which helps compensation by increasing
intravascular volume. The cumulative effect eventually becomes “congestive” because the body
is unable to eliminate the excessive fluid by normal mechanisms.
Anonymous Currents p 380-381/ Noble p 580-581
A) Frank Starling Relationship- this law states that as the myocardium is stretched as in
failure the contractile force is increased to an extent. If the fibers are stretched too far the
associations of the actin and myosin bridges dissociate and the muscle can no longer complete a
contractile force.
B) Ventricular Dilation- With an increasing preload or afterload the ventricle can stretch and
become dilated. The volume to be pumped has increased but the myocardial muscle within the
ventricle has remained the same or has thinned as the stretching occurs. This results in a
decreased ejection fraction.
C) Ventricular Hypertrophy- If the left ventricular dilation is chronic and not corrected the
myocardium within that ventricle responds by hypertrophying. This increase in ventricular wall
thickness normalizes wall stress and normalizes the ejection fraction. The ventricle wall thickens
and its blood and oxygen demands increase often to a point that the coronary artery system
cannot supply adequately.
D) Increased Sympathetic Nervous System Activity- The sympathetic nervous system will
respond to a reduced cardiac output by stimulating increased cardiac contractility, heart rate, and
venous tone. This will raise the central blood volume, which elevates the preload in attempts to
correct the decreased cardiac output.
E) Excess Salt and Water Retention- Reduced cardiac output that reduces renal blood supply
leads to reduced glomerular filtration which results in excess fluid and salt retention. This
activates the rennin-angiotensis-aldosterone system, which is attempting to increase blood supply
to the kidneys. Its actions result in more fluid and salt retention in attempts to increase volume
enough to increase the renal blood supply when in effect this compounds the cardiac failure
problem.

5. Distinguish between the following forms of heart failure, and give examples
of each.
acute vs. chronic heart failure
left-sided vs. right-sided heart failure
low-output vs. high-output heart failure
Jennyb Noble 583
Acute heart failure is due to sudden reduction in cardiac output with poor organ perfusion and/or
marked pulmonary congestion. i.e. acute MI. Chronic is caused by a reduction in cardiac output
that occurs gradually and compensatory mechanisms take place. i.e. CHF and the renin
angiotensin system,
Left heart failure causes pulmonary congestion which increases pulmonary pressures which leads
to pulmonary hypertension and subsequent right heart failure. Right heart failure is often
associated with generalized fluid accumulation.
Low output heart failure results when the heart is at rest or mildly exercised. ( the heart is taxed
with minimal effort) this is the most common type of failure (CHF)
High output results when the heart is unable to meet high metabolic demands of the body this is
less common and examples are anemia or Paget’s disease.
Anonymous Table 65-2, p. 583 Noble
Acute vs. Chronic:
2. Feature 3. Acute heart failure 4. Chronic heart failure
Symptom severity Marked Mild to moderate
Pulmonary edema Frequent Infrequent
Peripheral edema Rare Frequent
Total body fluid No change or mild increase Increased
Cardiomegaly Uncommon Common
Sympathetic activation Marked Mild to marked
Repairable lesion Common Occasional
• In Acute Heart Failure, symptoms are due to the sudden reduction in cardiac output
with poor organ perfusion and/or marked pulmonary congestion. Examples include
acute myocardial infarction, sustained tacky arrhythmia, and valve rupture due to
infective endocarditis.
• In Chronic Heart Failure-the clinical manifestations of heart failure depend on the rate
at which the syndrome develops, and specifically on whether enough time has elapsed for
compensatory mechanisms to become operative and interstitial fluid to accumulate.
• When the reduction in cardiac output occurs gradually (e.g. due to chronic valvular
regurgitation or remodeling post myocardial infarction), compensatory mechanisms
become operative and allow the patient to tolerate hemodynamic abnormalities with few
or no signs and symptoms. However, an intercurrent event such as infection, myocardial
ischemia, or medication noncompliance may precipitate manifestations of acute heart
failure.

Left-sided vs. right-sided heart failure:


• According to the "backward failure "hypothesis, fluid accumulates behind the ventricle
that is initially affected. Thus patients with left-sided heart failure (e.g. due to anterior
myocardial infarction or mitral regurgitation) initially develop pulmonary venous
congestion. Elevated pulmonary venous pressures in turn lead to pulmonary
hypertension and subsequent right-sided heart failure. Fluid accumulation becomes
generalized and patients develop signs of right-sided heart failure including lower
extremity edema, tender hepatomegaly, and pleural effusions. Pure right-sided heart
failure may occur secondary to primary pulmonary hypertension for chronic pulmonary
emboli.
Nobel pg 583

Low output vs. high output heart failure:


• Low cardiac output at rest, or in milder cases during exercise or stress, characterizes most
etiologies of heart failure. Peripheral vasoconstriction may result in cold, pale
extremities, a narrow pulse pressure, and widened arterial venous oxygen
• (A-V-02) gradient. High output states of which the pumping function of the heart is
unable to meet the abnormally high metabolic demands on the body less commonly lead
to heart failure.
• Examples include thyrotoxicosis, anemia, ateriovenous fistula, thiamine
deficiency(beriberi), and Paget's disease. The extremities are often warm and flushed,
the pulse pressure normal or widened, and the A-V-02 difference narrow.
Anonymous Noble, pg. 583
Acute vs. chronic heart failure. In acute heart failure, symptoms are due to the sudden
reduction in cardiac output with poor organ perfusion and/or marked pulmonary congestion.
Examples include acute myocardial infarction, sustained tacharrhythmia, and valve rupture due
to infective endocarditis. When the reduction in cardiac output occurs gradually (e.g., due to
chronic valvular regurgitation or remodeling postmyocardial infarction), compensatory
mechanisms become operative and allow the patient to tolerate hemodynamic abnormalities with
few or no signs and symptoms. However, an intercurrent event such as infection, myocardial
ischemia, or medication noncompliance may precipitate manifestations of acute heart failure.
Left-sided vs. right-sided heart failure. According to the “backward failure” hypothesis, fluid
accumulates behind the ventricle that is initially affected. Thus patients with left-sided heart
failure (e.g., due to anterior myocardial infarction or mitral regurgitation) initially develop
pulmonary venous congestion. Elevated pulmonary venous pressures in turn lead to pulmonary
hypertension and subsequent right-sided heart failure. Fluid accumulation becomes generalized
and patients develop signs of right-sided failure including lower extremity edema, tender
hepatomegaly, and pleural effusions. Pure right-sided failure may occur secondary to primary
pulmonary hypertension or chronic pulmonary emboli.
Low-output vs. high-output heart failure. Low cardiac output at rest, or in milder cases during
exercise or stress, characterizes most etiologies of heart failure. Peripheral vasoconstriction may
result in cold, pale extremities, a narrow pulse pressure, and a widened arterial-venous-oxygen
(A-V-O2) gradient. High-output states in which the pumping function of the heart is unable to
meet the abnormally high metabolic demands of the body less commonly lead to heart failure.
Examples include thyrotoxicosis, anemia, arteriovenous fistula, thiamine deficiency (beriberi),
and Paget’s disease. The extremities are often warm and flushed, the pulse pressure normal or
widened, and the A-V-O2 difference narrowed.
Anonymous Noble p 583
A) Acute Heart Failure- symptoms are due to sudden reduction in cardiac output with poor
organ perfusion and/or marked pulmonary congestion. Causes of this can be an acute
myocardial infarction, sustained tachyarrhythmia, and valve rupture due to infective
endocarditis.
B) Chronic Heart Failure- when reduction in cardiac output occurs gradually as in chronic
vavlular regurgitation or remodeling postmyocardial infarction, compensatory mechanisms
become operative and allow the patient to tolerate hemodynamic abnormalities with few or no
signs or symptoms.
C) Left-sided Heart Failure- fluid builds behind the defective ventricle so in left sided failure
the congestion builds in the left atria and is translated to the pulmonary circulation leading to
pulmonary congestion. This often leads to subsequent right sided heart failure.
D) Right-sided Heart Failure- fluid builds behind the defective ventricle so the right atria
becomes congested and systemic circulation backs up leading to peripheral edema especially in
the lower extremities, tender hepatomegaly, and pleural effusions are a possibility. Pure right
sided heart failure can develop as a result of primary pulmonary hypertension or chronic
pulmonary emboli.
E) Low-output Heart Failure- low cardiac output at rest or during exercise may result in cold
pale extremities, a narrow pulse pressure, and a widened atriovenous oxygen gradient.
F) High-output Heart Failure- the pumping function of the heart is unable to meet the
abnormally high metabolic demands of the body. Examples are thyrotoxicity, anemia, thiamine
deficiency, and Paget's disease. The extremities are often warm and flushed, the pulse pressure is
normal, and the atriovenous oxygen gradient is narrowed.

6. Compare and contrast a number of common precipitating or aggravating


factors in heart failure.
Zen Seeker Noble 584 http://home.mdconsult.com/das/book/body/0/959/444.html#top
TABLE 65-3 -- Etiology of Heart Failure
Impaired filling
Contractile Pressure Volume Valvular Diastolic Increased
dysfunction overload overload dysfunction dysfunction demand
Ischemic heart Hypertension Aortic Mitral Hypertrophic Anemia
disease regurgitation stenosis cardiomyopathy
Dilated Aortic Mitral Tricuspid Amyloidosis Thyrotoxicosis
Cardiomyopathy stenosis regurgitation stenosis
Myocarditis Aortic Left-to-right Atrial Constrictive Arteriovenous
coarctation shunt myxoma pericarditis fistula
Toxins (alcohol, Thiamine
doxorubicin) deficiency
Infection (HIV, Paget's disease
sepsis)

Box 65-2. Precipitating Factors


Patient-Related Factors
Excess exertion or emotional stress
Excess fluid and/or sodium intake
Noncompliance with prescribed medications
Moderate to heavy alcohol consumption

Physician-Related Factors
Use of medications that cause salt and water retention (e.g., nonsteroidal antiinflammatory
agents)
Use of negative inotropic agents (e.g., calcium channel blockers)

Heart Failure - Related Disease States


Uncontrolled hypertension
Unstable angina or acute myocardial infarction
Atrial or ventricular arrhythmias
Pulmonary emboli
Infective endocarditis

Other Disease States


Systemic infection
Renal or hepatic failure
High-output states (e.g., anemia, hyperthyroidism, pregnancy)
Noble 583 http://home.mdconsult.com/das/book/body/0/959/443.html#bottom
Precipitating Factors
Patients with heart failure may be asymptomatic or mildly symptomatic either because the
cardiac impairment is mild or because compensatory mechanisms help to normalize cardiac
function. However, symptoms of heart failure may develop when precipitating factors increase
cardiac workload and disrupt the balance in favor of decompensation. Precipitants may be
identified in 50% to 90% of hospital admissions and can be divided into patient-related factors,
physician-related factors, heart failure-related disease states, and other causes ( Box 65-2 ).
Inability to recognize and correct these factors promptly may lead to persistent heart failure
despite adequate treatment.
Anonymous Noble Box 65-2, p. 584
Heart failure
Patient related Physician related
related disease Other disease states
factors factors
states
• Excess exertion • Use of • Uncontrolled • Systemic infection
or emotional medications that hypertension, • Renal or hepatic
stress cause salt and unstable angina failure
• Excess fluid water retention or acute MI • High output states
and/or sodium (NSAIDs) • Atrial or (anemia,
intake • Use of negative ventricular hyperthyroidism,
• Noncompliance inotropic agents arrhythmias pregnancy)
with prescribed (calcium channel • Pulmonary
medications blockers) emboli
• Moderate to • Lack of listening • Infective
heavy alcohol • Made the patient endocarditis
consumption angry

Precipitating Factors-symptoms of heart failure may develop when precipitating factors


increased cardiac workload and disrupt the balance in favor of compensation. Precipitants may
be identified in 50% to 90% of hospital admissions. Inability to recognize and correct these
factors promptly may lead to persistent heart failure despite adequate treatment. Noble p. 583
Anonymous Noble p 584
A) Patient Related Factors:
1. Excess exertion or emotional stress
2. Excess fluid and/or sodium intake
3. Noncompliance with prescribed medications
4. Moderate to heavy alcohol consumption

B) Physician Related Factors:


1. Use of medications that cause salt and water retention (NSAIDs)
2. Use of negative inatropic agents (calcium channel blockers)

C) Heart Failure Related Disease States:


1. Uncontrolled hypertension
2. Unstable angina or acute myocardial infarction
3. Atrial or ventricular arrhythmias
4. Pulmonary emboli
5. Infective endocarditis

D) Other Disease States


1. Systemic infection
2. Renal or hepatic failure
3. High-output states (anemia, hyperthyroidism, and pregnancy)

7. Summarize the symptoms and physical findings of CHF.


Zen Seeker Noble 584- http://home.mdconsult.com/das/book/body/0/959/444.html#top
History
Symptoms of Pulmonary Venous Congestion.
Dyspnea or breathlessness is a cardinal manifestation of left-sided heart
failure. Mechanisms of dyspnea include pulmonary venous congestion and transudation
of fluid into the interstitium, leading to decreased lung compliance, increased airway
resistance, hypoxemia, and ventilation/perfusion mismatch. Stimulation of J receptors
leading to an increased ventilatory drive and reduced blood flow to respiratory muscles
may cause lactic acidosis and the sensation of dyspnea.
In the early stages of heart failure, dyspnea occurs primarily on effort. As
the disease progresses, the extent of effort required to provoke dyspnea decreases.
Finally, the patient becomes dyspneic at rest. The New York Heart Association (NYHA)
classification may be used to categorize patients based on the relation between symptoms
and the amount of effort required to provoke them ( Box 65-3 ). Although NYHA class
has been used to stratify patients in clinical studies and provides prognostic information,
its accuracy and reproducibility are limited. In addition, there is a poor correlation
between NYHA class and other, more objective functional measures of heart failure
severity such as exercise duration and peak oxygen consumption.
Orthopnea refers to dyspnea that occurs in the recumbent position. It is
due to redistribution of fluid from the abdomen and the lower body into the chest, an
increase in the work of breathing when a patient with decreased lung compliance lies flat,
and elevation of the diaphragm by ascites and hepatomegaly. Orthopnea usually occurs
within a minute or two of assuming recumbency, and develops when the patient is awake.
Initially, breathing at night is made easier by elevating the head on two or more pillows.
As heart failure progresses, the patient may have to sleep sitting up.
Two symptoms related to orthopnea are nocturnal cough and trepopnea
(or dyspnea limited to one lateral decubitus position). The exact mechanism for trepopnea
is unclear but may be related to distortions of the great vessels or alterations in coronary
perfusion pressure. With advanced biventricular failure, orthopnea may diminish as
symptoms of right-sided failure supervene (see below).
Paroxysmal nocturnal dyspnea is a form of orthopnea, and may occur with further
progression of LV failure. The patient awakens suddenly with a feeling of severe anxiety
and suffocation, and has to sit bolt upright for relief. In contrast to orthopnea, paroxysmal
nocturnal dyspnea usually occurs after prolonged recumbency, is less predictable in its
occurrence, and may require 30 minutes or longer in the upright position for relief.
Episodes are often accompanied by coughing and wheezing and may be extremely
frightening to the patient and family.
When significant wheezing is associated with paroxysmal nocturnal dyspnea, it
resembles an acute asthmatic attack and may be referred to as cardiac asthma.
Bronchospasm, which is caused by congestion of the bronchial mucosa and by interstitial
pulmonary edema compressing small airways, increases the work of breathing. Acute
pulmonary edema can be a further extension of paroxysmal nocturnal dyspnea.
Alternatively, acute pulmonary edema may occur as a primary manifestation of acute
myocardial infarction or accelerated hypertension. The patient is extremely short of
breath and coughs up pink, frothy sputum. Acute pulmonary edema occurs when there is
marked elevation of the pulmonary capillary wedge pressure leading to alveolar edema.
Untreated, it can be fatal.
Symptoms of Decreased Cardiac Output.
Symptoms related to decreased cardiac output can occur with right-sided or left-
sided heart failure but more commonly occur in patients with chronic biventricular
failure. Fatigue and weakness, particularly in the lower extremities, are nonspecific
symptoms thought to be due to decreased cardiac output to exercising muscles. Impaired
flow-mediated vasodilation, autonomic imbalance, and altered skeletal muscle
metabolism may also play contributory roles. Mental dullness and confusion, especially
in older patients with cerebrovascular disease, may result from decreased cerebral
perfusion. Other causes of fatigue in patients with heart failure include hyponatremia,
volume depletion, and medications (e.g., beta-blockers).
Symptoms of Systemic Venous Congestion.
Whereas symptoms of left-sided heart failure are related to pulmonary venous
congestion and fluid accumulation in the lungs, symptoms of right-sided heart failure
result from systemic venous congestion. One of the earliest symptoms of right heart
failure may be an inappropriate weight gain. In ambulatory patients, this is followed by
swelling in the feet or ankles at the end of the day, which generally resolves overnight. In
bedridden patients, edema first develops in the presacral region. With progressive right
heart failure, dependent edema becomes persistent. Finally, the development of massive
edema involving the entire body is termed anasarca.
Elevated systemic venous pressures may result in right upper quadrant abdominal
pain as the liver becomes engorged with fluid and its capsule is stretched. Other
symptoms associated with edema of the gastrointestinal tract include nausea, vomiting,
anorexia, early satiety, and constipation. These symptoms are nonspecific, and digoxin
toxicity should be ruled out. Ascites results in an increase in abdominal girth, and
unilateral or bilateral pleural effusions can contribute to the sensation of dyspnea.
Cardiac Cachexia.
Longstanding, severe heart failure may lead to chronic weight loss. Factors
contributing to cardiac cachexia include poor oral intake due to anorexia, and impaired
fat absorption due to bowel wall edema. Metabolic pathways that cause
catabolic/anabolic imbalance have also been implicated in this syndrome including the
growth hormone-insulin-like growth factor-1 system and the pituitary-thyroid hormone
axis. Circulating levels of tumor necrosis factor-alpha, a proinflammatory cytokine, are
elevated in patients with severe heart failure and contribute to cardiac cachexia.
Nocturia and Oliguria.
Urinary symptoms are common in heart failure. Nocturia may occur early in the
course of disease. During daytime activities, urine output is reduced due to the
redistribution of blood flow away from the kidneys. When the patient lies down at night,
improved cardiac output and renal vasodilation lead to increased urine formation. With
biventricular failure, insomnia due to nocturia may be exacerbated by orthopnea and
paroxysmal nocturnal dyspnea. Oliguria is a sign of end-stage heart failure and is due to
severe reductions in cardiac output and renal blood flow.

Physical Examination
The physical signs of left-sided heart failure relate to pulmonary venous
congestion, whereas signs of right-sided heart failure relate to systemic venous
congestion. In the discussion that follows, physical signs are presented in the order in
which they are typically assessed. Rigorous criteria for identifying heart failure based on
both the clinical history and physical findings were developed for the Framingham Study
( Box 65-4 ). However, heart failure may not be recognized in up to 40% of patients due
to the limited reliability of these findings.[14]
General Appearance.
Patients with compensated chronic heart failure often appear well nourished and
comfortable at rest. Even patients with moderate heart failure may appear to be in no
distress after resting for several minutes, but become dyspneic during or immediately
after activity. By contrast, patients with decompensated heart failure may appear anxious,
dusky, and diaphoretic, and are often dyspneic at rest or on lying down. Other findings
suggestive of severe heart failure include cool extremities and peripheral cyanosis
resulting from low cardiac output and systemic vasoconstriction. As noted, chronic
biventricular failure can result in cardiac cachexia. In severe right heart failure, hepatic
congestion can cause scleral icterus and jaundice. Patients with recent onset of heart
failure often appear acutely ill but are usually well nourished.
Vital Signs.
Resting sinus tachycardia is common, and is due to increased adrenergic tone. In
mild heart failure, the heart rate at rest may be normal, but increases excessively with
exercise and is slow to normalize with rest. The pulse may be irregular if atrial fibrillation
is present or in the presence of frequent premature ventricular complexes. In severe LV
failure, the peripheral pulse may be alternatingly strong and weak and is referred to as
pulsus alternans. Pulsus alternans is attributed to reduced LV contraction in every other
cardiac cycle due to incomplete recovery causing alternation in the LV stroke volume.
Rarely, weaker beats may fail to open the aortic valve, resulting in an apparent halving of
the pulse rate, a condition termed total alternans.
Tachypnea may be present in patients with severe LV failure and dyspnea at rest,
or secondary to pleural effusions or ascites in patients with right heart failure. The
respiratory rate may be normal in the sitting position, but increase in the patient with
pulmonary venous congestion on lying down. Advanced heart failure may be associated
with Cheyne-Stokes respiration, also called periodic breathing. Cheyne-Stokes respiration
consists of periods of hyperpnea alternating with apnea, and is probably caused by
prolonged circulation time from the heart to the brain, which affects the normal
regulation of breathing. In addition, there is diminished sensitivity of the respiratory
center to the arterial carbon dioxide pressure, which waxes and wanes during periods of
hyperpnea and apnea. The fall in oxygen pressure and rise in carbon dioxide pressure
during the apneic phase stimulate the respiratory center and result in hyperpnea, and the
cycle continues. Cheyne-Stokes respiration is common among elderly patients with LV
failure in whom the presence of cerebral arteriosclerosis and use of hypnotics may be
contributory. The patient is usually unaware of the altered breathing pattern, but other
family members may notice it and become alarmed. Cheyne-Stokes respiration may
contribute to daytime somnolence in patients who are awakened frequently during
periods of hyperpnea.
The systolic blood pressure may be elevated in diastolic heart failure due to
chronic hypertension, normal in compensated systolic heart failure, or low in advanced
heart failure. Diastolic blood pressure may be slightly elevated due to increased
adrenergic activity. A significant reduction in cardiac output is reflected by a narrow
pulse pressure, which is defined as the difference between the systolic and diastolic blood
pressures. For example, when the pulse pressure is less than 25% of the systolic pressure,
the cardiac index is generally less than 2.2 L/min/m2 . When the volume status is unclear
in a patient with dyspnea, a bedside Valsalva's maneuver may be used to detect elevated
left ventricular filling pressures.
A low-grade fever resulting from cutaneous vasoconstriction may occur in severe
heart failure, and subside when compensation is restored. Temperatures greater than
101°F should suggest infection.
Jugular Venous Pulse.
Elevation of the jugular venous pressure is a hallmark of elevated systemic
venous pressure. The upper limit of normal is approximately 4 cm above the sternal angle
when the patient is examined at a 45-degree angle, corresponding to a right atrial pressure
of less than 10 cm of water. Higher levels of venous pressure, approaching the angle of
the jaw, are common in right sided failure. When tricuspid regurgitation is present, the
descending limb of the a wave is attenuated, and the height of the v wave increases with a
rapid y descent. Rarely, venous pressure is so high that veins under the tongue or on the
dorsum of the hand are dilated. Kussmaul's sign is present when jugular venous pressure
increases with inspiration.
In patients with mild right heart failure, jugular venous pressure may be normal at
rest but increase with compression of the right upper quadrant. Hepatojugular reflux may
be elicited by gently applying firm, continuous pressure over the liver for up to 1 minute
while observing the neck veins. The patient must breathe normally and not strain during
this maneuver. Normally, abdominal or hepatic compression leads to a transient increase
in jugular venous pressure. In heart failure, the abnormal right ventricle is unable to
accept an increase in venous return and jugular venous pressure remains elevated. In
biventricular failure, elevated jugular venous pressure at rest or after hepatic or
abdominal compression is a moderately sensitive and highly specific marker of increased
pulmonary capillary wedge pressure.
Examination of the Heart
Precordial Palpation.
Chronic heart failure is accompanied by cardiac enlargement. Commonly, the
apical impulse is displaced downward and to the left, and may be either diffuse (in dilated
cardiomyopathy) or sustained (in pressure-overloaded states such as aortic stenosis). In
biventricular heart failure or severe right-sided heart failure, a right ventricular impulse
may be palpated along the lower sternal edge. A palpable third heart sound may also be
present. In acute heart failure or heart failure secondary to constrictive pericarditis or
restrictive cardiomyopathy, cardiac enlargement is usually not present.
Auscultation.
Although the presence of a third heart sound is common in healthy children and
young adults, in adults over age 40, an S3 gallop generally implies ventricular
dysfunction. In patients with mitral or tricuspid regurgitation or left-to-right shunts,
excessive flow into the ventricles can also cause a third heart sound without ventricular
dysfunction. In heart failure, the presence of a third heart sound is probably related to a
sudden deceleration of ventricular inflow that takes place after the early filling phase.
Abnormal compliance or diastolic dysfunction may also contribute to a gallop rhythm.
Ventricular remodeling in heart failure may lead to incompetence of the
atrioventricular (AV) valves. Thus holosystolic murmurs of mitral or tricuspid
regurgitation may be present in the absence of structural valvular abnormalities,
especially in advanced heart failure. These murmurs typically decrease in intensity or
disappear after successful treatment of decompensation. In biventricular failure or
isolated right heart failure, pulmonary hypertension is reflected in a loud pulmonary
component of the second heart sound.
Examination of the Lungs.
Rales result from the transudation of fluid into the alveoli and airways. In general,
rales are heard at the lung bases; but in severe heart failure, they may be heard throughout
the lung fields. Wheezing and rhonchi can occur with congestion of the bronchial
mucosa, and can lead to the misdiagnosis of reactive airways disease. In biventricular
failure, bilateral pleural effusions can occur, and are recognized as dullness to percussion
and decreased breath sounds at the bases. When rales or pleural effusion is limited to one
side, the right side of the chest is typically involved. Importantly, the absence of
pulmonary rales does not exclude significant elevation of the pulmonary capillary wedge
pressure in patients with chronic LV systolic failure.
Examination of the Abdomen and Extremities.
Hepatomegaly is an early sign of systemic venous congestion. In the early stages
of right heart failure, the liver may be tender due to stretching of its capsule, but with
progression of disease, tenderness may disappear. In patients with tricuspid regurgitation,
the liver may be pulsatile due to transmission of the v wave. Longstanding hepatic
congestion may lead to cardiac cirrhosis with portal hypertension and congestive
splenomegaly. Ascites results from increased pressure in the hepatic veins and the veins
draining the peritoneum. In most patients with heart failure, ascites is minimal. With
massive ascites, the physician should suspect constrictive pericarditis or primary liver
failure.
Dependent lower extremity edema is common in biventricular or isolated right
heart failure, and is typically symmetrical and pitting. In chronic heart failure, the amount
of edema does not correlate well with systemic venous pressure; in acute heart failure,
edema may be absent despite marked systemic venous hypertension. With advanced heart
failure, edema may become massive and generalized (anasarca). Chronic edema of the
distal lower extremities may cause reddening and induration of the skin.

Box 65-4. Framingham Study Criteria for Congestive Heart Failure

Major Criteria
Paroxysmal nocturnal dyspnea
Neck vein distention
Rales
Radiographic cardiomegaly
Acute pulmonary edema
S3 gallop
Increased central venous pressure
Circulation time >25 sec
Hepatojugular reflux
Visceral congestion or cardiomegaly at autopsy
Weight loss >4.5 kg in 5 days in response to treatment

Minor Criteria
Bilateral ankle edema
Nocturnal cough
Dyspnea on ordinary exertion
Hepatomegaly
Pleural effusion
Decrease in vital capacity by 1/3 from maximum value recorded
Tachycardia (rate >120/min)
Anonymous Goroll pg 216, Nobel pg 586.
Regardless of etiology, the clinical manifestations of CHF or quite stereotype and reflect the
magnitude of the fall in cardiac output and the rise in pulmonary and systemic venous pressures.
Initially and in mild cases, the patient may complain of fatigue, dyspnea on exertion, or
unexplained weight gain. There may be few overt physical signs of failure, and chest x-ray often
shows redistribution of pulmonary venous flow to upper lung fields and/or cardiomegaly.
Fatigue becomes increasingly prominent as cardiac output falls. As pulmonary congestion
increases, dyspnea worsens, orthopnea is noted, and paroxysmal nocturnal dyspnea may be
reported. At this stage, rales become evident on auscultation of the lungs, but their absence does
not well the presence of CHF. Sometimes failure-induced bronchospasm dominates the
pulmonary examination. In severe cases, chest film will show interstitial pulmonary edema. In
chronic CHF, right-sided or bilateral pleural effusions are common. Ankle edema, jugular
venous distention, and hepatojugular reflex are indicative of elevated systemic venous pressure;
if CHF is predominantly left-sided, these findings may not be present. An S3 gallop is among the
most specific physical signs of failure, but is often difficult to hear. If left ventricular dilation
becomes very marked, the mitral regurgitation murmur may become evident. Pedal edema is one
of the lease specific signs of CHF; in the elderly, isolated to you edema is more likely to be result
of venous insufficiency.

Major criteria Miner criteria


Paroxysmal nocturnal dyspnea Bilateral ankle edema
Neck vein distention Nocturnal cough
Rales Dyspnea on ordinary exertion
Radiographic cardiomegaly Hepatomegaly
Acute pulmonary edema Pleural effusion
S3 gallop Decrease in vital capacity by one-third
from maximum value recorded
Increased central venous pressure Tachycardia (rate > 120)
Circulation time > 25 seconds Diaphoresis
Hepatojugular reflex Tachypnea thewhat
Visceral congestion or cardiomegaly at autopsy,
weight loss > 4.5 kg in five days in response to
treatment
Box 65-4, Nobel pg 586
Heart failure may not be recognized in up to 40 percent of patients due to limited reliability of
these findings.
Anonymous Goroll, CH 32, pp 215-217
Physical Findings
Symptoms R side CHF L Side CHF X-Ray Studies
Fatigue Orthopnea Orthopnea Cardiomegaly EKG
Dyspnea PND DOE Interstitial Pulmonary Echocardogram w/
Edema Doppler
Weight JVD Basilar Rales Bilateral Pleural CXR
Gain Effusion
LE Edema MVR Kerley "B" Lines Cardiac Ultrasound
Murmur
S3 Gallop S3 Gallop Perihilar Haziness
Hepatojugular
Reflux
Notes:
1) Left sided CHF is harder to diagnosis than Right sided. Often the only presenting symptom is
fatigue and the physical findings maybe very slight or not present until full heart failure has
occurred.
2) The S3 Gallop is among the most specific findings but maybe difficult to elicit and maybe
present in the elderly patient with HTN and a normal ejection fraction.
Anonymous Currents p 381- 383/ Noble p 583-587
A) Symptoms Include:
1. Shortness of breath chiefly exertional dyspnea at first progressing to orthopnea and
paroxysmal
nocturnal dyspnea, and rest dyspnea.
2. chronic nonproductive cough, often worse in the recumbent position
3. Nocturia
4. Fatigue and exercise intolerance
5. RUQ pain due to hepatic enlargement
6. Loss of appetite and nausea due to edema in gut or impaired intestinal perfusion
7. Peripheral edema
8. Angina

B) Physical Exam Findings Include:


1. Dyspneic during conversation
2. Tachycardia, hypotension, and reduced pulse pressure
3. Cold extremities and diaphoresis
4. Neck: jugular venous distention/ hyperthyroidism
5. Lungs: crackles at bases, expiratory wheezes and ronchi
6. Abdomen: hepatic enlargement and tenderness and ascites
7. Peripheral pitting edema especially in the lower extremities
8. Cardinal Cardiac Signs:
*Parasternal lift indicating pulmonary hypertension
*Enlarged and sustained left ventricular impulse indicating left ventricular
dilation or hypertrophy
*Diminshed first heart sound suggesting impaired contractility
*S3 gallops originating in the left and sometimes right ventricle

8. Review, discuss and explain the role of each of the following laboratory
studies and diagnostic procedures in CHF.
CBC
electrolyte levels
BUN and creatinine levels
liver function tests Dropped 2004
arterial blood gases
EKG
chest X-ray
echocardiography
radionuclide angiography
cardiac catheterization
Zen Seeker Noble 587- http://home.mdconsult.com/das/book/body/0/959/445.html#top
CBC
Anemia is not diagnostic of heart failure, but when present may exacerbate underlying ischemic
heart disease and should be corrected. Rarely, severe anemia may cause high-output failure. The
erythrocyte sedimentation rate often decreases in heart failure because of impaired fibrinogen
synthesis and decreased fibrinogen concentration. A marked increase in sedimentation rate may
suggest infective endocarditis.

electrolyte levels
Dilutional hyponatremia is common in severe heart failure and is the result of prolonged sodium
restriction, diuretic therapy, and expansion of extracellular volume. Increased vasopressin levels
may also contribute to hyponatremia. Hyponatremia is a negative prognostic indicator at the time
of hospital admission for heart failure, and predicts decreased long-term survival.
Hypokalemia is most often due to thiazide or loop diuretics given without oral potassium
supplementation, but may also result from increased aldosterone levels due to activation of the
renin-angiotensin system. If uncorrected, hypokalemia may lead to ventricular arrhythmias,
especially in the presence of digoxin. Hyperkalemia may result from marked reductions in
glomerular filtration rate and inadequate delivery of sodium to the distal renal tubule. Excess
total body potassium may be exacerbated by the use of potassium-sparing diuretics or
angiotensin-converting enzyme inhibitors, and in particular, their concurrent use. During
hospitalization for heart failure, hyperkalemia is a common cause of iatrogenic morbidity, and
even mortality. Other electrolyte abnormalities seen in heart failure include hypophosphatemia
and hypomagnesemia, both of which are commonly associated with chronic alcohol use.

BUN and creatinine levels


Blood urea nitrogen and creatinine levels are often moderately elevated in severe heart failure
because of a reduction in renal blood flow and glomerular filtration rate. Proteinuria may also be
present, especially in the setting of longstanding hypertension or diabetes.

liver function tests


Chronic right-sided heart failure with congestive hepatomegaly leads to abnormal liver function.
Serum aminotransferase, lactic dehydrogenase, and alkaline phosphatase levels are elevated,
typically 2 to 3 times normal. Marked elevation in transaminases suggesting 'shock liver' can be
associated with severe low output states. Hyperbilirubinemia is common in heart failure, and in
severe cases of acute heart failure jaundice may occur. In patients with cardiac cirrhosis,
hypoalbuminemia may exacerbate fluid accumulation.

arterial blood gases found online


Arterial blood gas (ABG) may be of benefit in evaluation of hypoxemia, ventilation/perfusion
(V/Q) mismatch, hypercapnia, and acidosis. Blood gas abnormalities may show respiratory
alkalosis in mild forms of CHF or metabolic acidosis in patients with evidence of low cardiac
output or ductal-dependent congenital heart disease.

EKG
No specific electrocardiographic pattern is diagnostic of heart failure. However, the ECG may
provide important information regarding the nature of the underlying cardiac disease. For
example, LVH and left atrial enlargement suggest left heart failure resulting from antecedent
hypertension or aortic stenosis. Pathologic Q waves in ischemic heart disease indicate the
presence and location of myocardial infarction, while nonpathologic Q waves (pseudoinfarction)
may be seen with restrictive or dilated cardiomyopathy. Abnormal cardiac rhythms such as atrial
fibrillation may be secondary to heart failure or may represent inadequacy of therapy if the
ventricular response is uncontrolled. If present, ventricular ectopic activity may indicate
increased risk of sudden cardiac death, or reflect digoxin toxicity or electrolyte imbalance (e.g.,
hypokalemia).

chest X-ray
The size and shape of the cardiac silhouette provide important information regarding the nature
of the underlying heart disease. A cardiothoracic ratio greater than 0.5, for example, is a good
indicator of increased LV volume. The other major radiographic abnormality associated with left
heart failure is pulmonary venous congestion. The degree of pulmonary venous congestion often
parallels increases in the pulmonary capillary wedge pressure. Early radiologic signs of
pulmonary venous hypertension and interstitial edema include distention of the pulmonary veins
extending upward from the hila, haziness of hilar shadows, and thickening of interlobular septa
(Kerley's B lines). When the pulmonary capillary wedge pressure is moderate to severely
elevated, often greater than 25 mm Hg, alveolar edema is present as diffuse haziness extending
downward toward the lower portions of the lung fields (so-called butterfly pattern). In patients
with chronic LV failure, higher pressures can be accommodated with fewer radiologic signs due
to enhanced lymphatic drainage. Pleural effusions of varying size and distribution are common in
biventricular failure.

echocardiography
Echocardiography is commonly performed in the evaluation and management of heart failure.
Two-dimensional echocardiographic imaging provides an accurate and rapid determination of
ventricular size and function, and valvular morphology and function, and can detect intracavitary
thrombi and pericardial effusions. Important hemodynamic data including cardiac output,
pulmonary artery pressures, and valve areas can be obtained using Doppler echocardiographic
techniques. Diastolic function is more difficult to assess, although newer techniques may provide
accurate, load-independent measures of LV relaxation. The advent of transesophageal
echocardiography has made it possible to obtain reliable information when transthoracic
'windows' are inadequate.

radionuclide angiography
Two other noninvasive techniques commonly used in the assessment of cardiac function are
radionuclide ventriculography (RVG) and cardiac magnetic resonance imaging (MRI). RVG
provides a reliable quantification of right and left ventricular ejection fraction, and can
characterize wall motion abnormalities in ischemic heart disease. Recently, cardiac MRI has
emerged as a highly accurate and quantitative tool for the evaluation of ventricular function and
myocardial mass. Serial MRI studies can assess ventricular remodeling in response to therapy.

cardiac catheterization
In the intensive care setting, assessment of volume status and/or cardiac output may be necessary
to differentiate cardiogenic from noncardiogenic pulmonary edema, and manage hemodynamic
instability. The gold standard for evaluating cardiac hemodynamics is right heart (or Swan-
Ganz) catheterization using a balloon-tipped flotation catheter. This procedure may be
performed safely at the bedside, and is used primarily to determine response to parenteral
inotropic and/or vasodilator therapy in severe heart failure. Simultaneous measurement of right
and left heart filling pressures in the cardiac catheterization laboratory can be used to distinguish
restrictive cardiomyopathy from constrictive pericarditis.
Anonymous Nobel pgs 587-588
• CBC-detects anemia. Anemia is not diagnostic of heart failure, but when present may
exacerbate underlying ischemic heart disease and should be corrected. Rarely, severe
anemia may cause high-output failure. The erythrocyte sedimentation rate is often
decreased in heart failure because of impaired fibrinogen and synthesis and decreased to
fibrinogen concentration a. A marked increase in SED rate may suggest infective
endocarditis.
• Serum electrolytes-dilutional hyponatremia is common in severe heart failure and is a
result of prolonged sodium restricting, diuretic therapy, and expansion of extracellular
volume. Increased vasopressin levels may also contribute to hyponatremia. Hyponatremia
is a negative indicator at the time of hospital admission for heart failure, and predicts
decreased long-term survival. Hypokalemia is most often due to thiazide or loop
diuretics given without oral potassium supplementation, but may also result from
increased aldosterone levels due to activation of the renin-angiotensin system. If
uncorrected hypokalemia may lead to ventricular arrhythmias, especially in the presence
of digoxin. Hyperkalemia may result from marked reductions in glomerular filtration rate
and inadequate delivery of sodium to the distal renal tubule. Excess total body potassium
may be exacerbated by the use of potassium sparing diuretics or ACE inhibitors, and their
concurrent use. Other abnormalities seen in heart failure include hypophosphatemia &
hypomagnesiumia, both of which are commonly associated with chronic ETOH abuse.
• BUN and Creatinine levels-allow you to assess renal hepatic function. These levels are
often moderately elevated in severe heart failure because the reduction in renal blood
flow and glomerular filtration rate. Proteinuria may also be present, especially in the
setting of long-standing hypertension or diabetes. BUN & creatinine are essential to
avoid excess volume depletion and severe prerenal azotemia.
• Liver function tests-serum aminotransferase, lactic dehydrogenase, and alkaline
phosphatase levels or elevated, typically 2 to 3 times normal. Marked elevation in
transaminases suggesting "shock liver” can be associated with severe low output states.
Hyperbilirubinemia is common in heart failure, and in severe cases of acute heart failure
jaundice may occur. In patients with cardiac cirrhosis, hypoalbuminemia may exacerbate
fluid accumulation. Chronic right-sided heart failure with congestive hepatomegaly leads
to abnormal liver function.
• Arterial blood gases-ABGs are essential to monitor hypoexmia states. It checks the
ventilation and perfusion to the tissues (basically shows how well the heart and lungs are
working). It can tell you the workload on the heart and lungs. It lets you see if your
diuretics are working by dumping excess fluid off lungs by improving ABG results.
• EKG-no EKG pattern is specific for diagnostic of heart failure. However, the EKG and
may provide import information regarding the nature of the underlying cardiac disease.
For example LVH and left atrial enlargement suggests left heart failure resulting from
antecedent hypertension or aortic stenosis. Pathologic Q waves in ischemic heart disease
indicate the presence of and location of myocardial infarction, while nonpathologic Q
waves (psuedoinfarction) may be seen with restrictive or dilated cardiomyopathy.
Abnormal cardiac rhythms such as atrial fibrillation may be secondary to heart failure or
may represent inadequacy of therapy if the ventricular responses uncontrolled. If present,
ventricular ectopic activity may indicate increased risk of sudden cardiac death or reflect
digoxin toxicity or electrolyte imbalance.
• Chest x-ray-the size and shape of the cardiac silhouette provide import information
regarding the nature of the underlying heart disease. A cardiothoracic ratio > 0.5 for
example, is a good indicator of increased LV volume. The other major radiographic
abnormality associated with left heart failure is pulmonary venous congestion. The
degree of pulmonary venous congestion often parol's increases in the pulmonary capillary
wedge pressure. Early radiologic signs of pulmonary venous hypertension and interstitial
edema include distention of the pulmonary veins extending upward from the hila,
haziness of hilar shadows, and thickening of interlobular septa (Kerley’s B lines). When
the pulmonary capillary wedge pressure is moderate to severely elevated, off then >
25mmhg, alveolar edema is present as diffuse haziness extending downward toward the
lower portions of the lung fields (so-called butterfly pattern). In patients with chronic LV
failure, higher pressures can be accommodated with fewer radiologic signs due to
enhancedlymphatic drainage. Pleural effusions of varying size and distribution are
common in biventricular failure.
• Echocardiography-is commonly performed in the evaluation and management of heart
failure. Two-dimensional echocardiographic imaging provides an accurate and rapid
determination of ventricular size and function, and alveolar morphology and function,
and can detect intracavitary thrombi and pericardial effusions. Important hemodynamic
data including cardiac output, pulmonary artery pressures, and velvet areas can be
obtained using Doppler echocardiographic techniques. Diastolic function is more difficult
to assess, although the newer techniques may provide accurate, load independent
measures of LV relaxation. The advent of transesophageal echocardiography has made it
possible to obtain reliable information when transthoracic "windows " are inadequate.
• Radionuclide angiography-a noninvasive technique commonly used in the assessment
of cardiac function. (RVG) provides a reliable quantification of right and left ventricular
ejection fraction, and can characterize wall motion abnormalities in ischemic heart
disease.
• Cardiac catheterization-(invasive study) in the intensive care setting, assessment or
volume status and/or cardiac output may be necessary to differentiate cardiogenic from
noncardiogenic pulmonary edema, and manage hemodynamic instability. The gold
standard for evaluating cardiac hemodynamics is right heart(or Swan-Ganz)
catheterization using a balloon tipped flotation catheter. This procedure may be
performed safely at the bedside, and is used primarily to determine response to parental
inotropic and/or vasodialtor therapy in severe heart failure. Simultaneous measurement
of right and left heart filling pressures in the cardiac catheterization laboratory can be
used to distinguish restrictive cardiomyopathy from constrictive pericarditis.
Anonymous Noble pg 587-588
CBC Anemia is not diagnostic of heart failure. When present may exacerbate underlying
ischemic heart disease and should be corrected. Severe anemia may cause high output period.
ESR rate decreases in heart failure.
Electrolyte levels Dilutional hyponatremia is common in severe heart failure due to prolonged
sodium restriction, diuretic therapy, and expansion of extracellular volume. Hyponatremia is a
negative prognostic indicator at the time of hospital admission and predicts decreased long term
survival. Hypokalemia is due to thiazide or loop diuretics given without oral K
supplementation. It may lead to ventricular arrhythmias especially in the presence of Digoxin.
Hyperkalemia may result from reductions in glomerular filtration rate and inadequate delivery of
Na to the distal renal tubule. It is a common cause of iatrogenic morbidity and mortality. Other
abnormalities include hypophosphatemia and hypomagnesemia. Both are commonly associated
with chronic alcohol use.
BUN and creatinine levels These are moderately elevated in severe heart failure because of a
reduction in renal blood flow and GFR. Proteinuria may also be present.
Liver function tests Serum aminotransferase, lactic dehydrogenase, and alkaline phosphatase
levels are elevated, typically 2-3 times normal. Marked elevation in transaminases suggesting
“shock liver” can be associated with severe low output states. Hyperbilirubinemia is common
and jaundice may occur. Hypoalbuminemia may exacerbate fluid accumulation.
Arterial blood gases Reduced peripheral perfusion results in anaerobic metabolism and the
production of lactic acid, causing metabolic acidosis. ABG’s reveal a lowered bicarbonate as the
body attempts to buffer the excess H ions to maintain pH. If the pH falls the patient
hyperventilates. Hyperventilation compensates for the increased acidity by lowering arterial
CO2 tensions, causing a mild respiratory alkalosis, which helps to normalize pH. (Cardiac
Nursing Underhill pg 382).
EKG No specific EKG pattern is diagnostic of heart failure, but it may provide important
information regarding the underlying disease. LVH and Lt atrial enlargement suggest Lt heart
failure resulting from HTN or aortic stenosis. Pathologic Q waves in ischemia indicate the
presence and location of MI, while nonpathologic Q waves may be seen with restrictive or
dilated cardiomyopathy. A fib may be secondary to heart failure or inadequacy of therapy.
Ventricular ectopy may indicate increased risk of sudden death or reflect digoxin toxicity or
electrolyte imbalance.
Chest X-ray Size and shape of the cardiac silhouette provide important information regarding
the nature of the underlying heart disease. Cardiothoracic ratio greater than .5 is a good indicator
of increased LV volume. Pulmonary venous congestion parallels increases in the pulmonary
capillary wedge pressure. Early radiologic signs of pulmonary venous hypertension and
interstitial edema include distention of the pulmonary veins extending upward from the hila,
haziness of hilar shadows, and thickening of intralobular septa (Kerley’s B lines). Alveolar
edema is present as diffuse haziness extending downward toward the lower portions of the lung
fields (butterfly pattern).
Echocardiography These 2 dimensional images provide an accurate and rapid determination
of ventricular size and function, and valvular morphology and function, and can detect
intracavitary thrombi and pericardial effusions. Important hemodynamic data including cardiac
output, pulmonary artery pressures, and valve areas can be obtained using Doppler
echocardiographic techniques.
Radionuclide angiography Radionuclide ventriculography provides a reliable quantification of
right and left ventricular ejection, and can characterize wall motion abnormalities in ischemic
heart disease.
Cardiac catheterization Right heart catheterization (Swan Ganz) is used to determine response
to parenteral inotropic and/or vasodilator therapy in severe heart failure. Simultaneous
measurement of right and left heart filling pressures in the cath lab can be used to distinguish
restrictive cardiomyopathy from constrictive pericarditis.
Anonymous Currents p 382-383/ Noble p 587-588
A) CBC- this may reveal anemia which is a cause of high-output failure and an exacerbating
factor in other forms of heart failure.

B) Electrolyte Levels- this may reveal


1. Hypokalemia which increases the risk of arrhythmias
2. Hyperkalemia which may limit the use of the rennin-angiotensin system
3. Hyponatremia which is an indicator of marked use of the rennin-angiotensis system

C) BUN and Creatinine Levels- these are often moderately elevated in CHF due to reduced
renal blood flow and glomerular filtration rate.

D) Liver Function Tests- these may become elevated due to right sided failure and hepatic
congestion
E) Arterial Blood Gases- the arterial blood gases detect decreases in oxygenated hemoglobin as
in cases of pulmonary edema. It can also detect metabolic acidosis or alkalosis as in cases of
excessive salt retainment.

F) EKG- this may indicate an underlying or secondary arrhythmia, myocardialinfarction, or


nonspecific changes that often include low voltage, intraventricular conduction defects, left
ventricular hypertrophy, and nonspecific repolarization changes.

G) Chest X-ray- these provide information about the size and shape of the cardiac silhouette. It
will also show pulmonary venous dilation and haziness will elucidate perivascular edema and
alveolar fluid.

H) Echocardiography- this test will reveal the size and function of both ventricles and both
atria. It will also show pericardial effusion, valvular abnormalities, intracardiac shunts, and
segmental wall motion abnormalities indicative of old myocardial infarction.

I) Radionuclide Angiography- this measures specific left ventricular ejection fraction and
regional wall motion indicative again of old myocardial infarction.

J) Cardiac Catheterization- This is much more invasive but useful in determining significant
valvular disease and the presence and extend of coronary artery disease. Coronary artery disease
is extremely important because left ventricular revascularization often will reverse left ventricle
ischemia leading to CHF.

9. Compare and contrast in general terms the role of each of the following in
the treatment of CHF:
rest
diet
diuretics
digitalis
vasodilators(especially ACE inhibitors)
Zen Seeker Noble 589- http://home.mdconsult.com/das/book/body/0/959/446.html#top
Rest
Appropriate restriction of physical activity is essential in the treatment of patients with
heart failure. Physical rest reduces metabolic demands and thus the overall work of the failing
heart. Bed rest in the hospital is usually necessary in the management of acute heart failure and
other forms of severe cardiac decompensation. Progressive mobilization is initiated when the
patient's condition permits and is encouraged as further clinical improvement results. Explicit
instructions regarding physical activity are discussed before discharge. Patients who work full-
time may need to reduce their hours or stop working altogether, depending on the physical and
mental demands of the job. It may be beneficial to avoid emotional stress and use relaxation
techniques.
Exercise is not contraindicated in patients with heart failure. Supervised cardiac
rehabilitation in selected patients may increase exercise tolerance, reduce symptoms, and
improve quality of life. Reduction in morbidity and mortality has been reported. These clinical
benefits are associated with important physiologic changes including improved vascular
endothelial function and skeletal muscle metabolism and decreased sympathetic tone. Thus a
program of regular, aerobic exercise for patients with heart failure is recommended.
Diet
Expanded extracellular volume, due in part to avid sodium retention by the kidney, may
be treated in most patients with diuretics (see below) and reduction in the dietary intake of
sodium. The average American diet without salt restriction contains as much as 10 gm/day of
salt. Prohibiting the addition of salt to cooked food and eliminating some salty foods (e.g., potato
chips) often reduce salt intake to about 4 to 5 gm/day ( Table 65-5 ). A salt substitute or herbs
and spices may be used to flavor food. Removal of salt from cooking altogether reduces intake to
about 2 gm of salt per day but often results in unpalatable food and poor compliance. This degree
of salt restriction is often unnecessary unless edema persists after vigorous diuretic therapy.
Further reduction of salt intake requires elimination of most processed foods and when possible,
substitution with low-sodium foods (e.g., fresh vegetables, low-sodium milk, cheese, and bread).
In obese patients with heart failure, supervised weight reduction is of critical
importance in reducing the workload of the heart. Specific advice regarding caloric restriction is
given, and the therapeutic goal of weight loss is reinforced during follow-up.

Diuretics
Diuretic therapy is an important element in the treatment of edema associated with heart failure.
By reducing the reabsorption of sodium and water by the renal tubule, diuretics improve
symptoms related to excess volume and may prevent ventricular remodeling by reducing cardiac
filling pressures. Current recommendations are to reserve diuretic therapy for patients with signs
and symptoms of fluid retention on an ACE inhibitor (or other vasodilator) and moderate salt
restriction, with or without digoxin. Commonly used diuretics in heart failure are thiazides, loop
diuretics, and potassium-sparing diuretics ( Table 65-6 ).

Digitalis
Although digoxin and related compounds have been used for over 200 years to treat heart
failure, debate continues regarding their safety and efficacy. Multicenter trials have shown that
digoxin increases ejection fraction and exercise tolerance, and decreases symptoms in patients
with systolic heart failure, and withdrawal of digoxin leads to increased symptoms and
hospitalizations ( Table 65-9 ). The recently completed DIG trial showed no difference in
survival with digoxin compared with placebo when given with an ACE inhibitor and diuretic.[5]
In the digoxin-treated group, fewer deaths attributable to the progression of heart failure were
offset by an increase in deaths due to other causes.
Digoxin increases cardiac contractility by inhibiting sacrolemmal Na-K-ATPase, thereby
increasing the amount of intracellular calcium available to the contractile apparatus. Cardiac
output increases and diuresis ensues. Digoxin also increases baroreceptor sensitivity, attenuates
neurohormonal activation, slows heart rate, and decreases systemic vasoconstriction. Finally,
digoxin decreases AV nodal conduction velocity, which makes it a useful agent for treating heart
failure in patients with atrial fibrillation or flutter with rapid ventricular response.
Digoxin is given to outpatients as a maintenance oral dose of 0.125 to 0.25 mg per day. In
the presence of normal renal function, full digitalization occurs within 5 to 7 days. If rapid
digitalization is required (e.g., in a hospitalized patient with rapid atrial fibrillation), an initial
dose of 0.25 to 0.5 mg is given intravenously, followed by 0.125 to 0.25 mg every 2 to 4 hours
up to a total of 1 mg. The peak effect is usually achieved between 1.5 and 6 hours.
Electrocardiographic monitoring should be used to monitor for proarrhythmia, with the
knowledge that some changes (e.g., shortening of the QT interval, flattening or inversion of the T
wave) reflect drug effect rather than drug toxicity.
Digoxin has a low toxic to therapeutic ratio. However, there is little evidence to support
using serum levels to guide dose selection. Hypokalemia, renal insufficiency, and hypoxia
commonly predispose patients to the toxic effects of digoxin. Digoxin should also be used with
caution in older patients with slower renal excretion; in patients with acute myocardial infarction,
myocarditis, or hypothyroidism; and in patients treated with drugs such as quinidine, verapamil,
and amiodarone that increase serum digoxin levels. Digoxin is contraindicated in patients with
second-or third-degree AV block unless a temporary pacemaker has been inserted.
Toxic manifestations of digoxin include gastrointestinal upset, visual disturbances, and a
wide range of cardiac arrhythmias. The new occurrence, or increased frequency, of ventricular
premature beats is the most common rhythm disturbance related to excess digoxin. Other cardiac
arrhythmias include second-degree AV block, paroxysmal atrial tachycardia with block, and life-
threatening ventricular tachycardia and fibrillation. Digoxin toxicity is usually reversed by
simply withdrawing the drug. Hypokalemia, if present, should be corrected. High-grade AV
block may require temporary pacing, while significant ventricular arrhythmias are treated with
lidocaine, phenytoin, and propranolol. Severe digoxin toxicity associated with recurrent
ventricular arrhythmias (e.g., following massive overdose) can be treated with purified Fab
fragments of digoxin-specific antibodies. The digoxin antibody complex is excreted in the urine.

Vasodilators (especially ACE inhibitors)


An important advance in the treatment of heart failure was the recognition that pump
function was critically dependent on afterload.[2] Agents that preferentially dilate arteriolar
resistance vessels (e.g., hydralazine) shift the ventricular function curve upward and to the left,
resulting in an increase in cardiac output often with little or no change in blood pressure. Agents
that preferentially increase capacitance in the venous system (e.g., nitrates) redistribute blood
volume from the central to peripheral reservoirs and decrease the signs and symptoms of
elevated filling pressures. ACE inhibitors, which have a balanced effect on the arterial and
venous system, also slow the progression of heart failure by interfering with the renin-
angiotensin system. The dosage and characteristics of vasodilators commonly used in the
treatment of heart failure are shown in Table 65-7 .
Several large, prospective, controlled trials have demonstrated the beneficial effects of
ACE inhibitors and other vasodilators on exercise tolerance, clinical signs and symptoms,
neurohormonal activation, quality of life, and survival in patients with chronic heart failure
( Table 65-8 ).
ACE Inhibitors.
By inhibiting the enzyme that converts angiotensin I to angiotensin II, ACE inhibitors
exert several important hemodynamic and neurohormonal effects in heart failure. A decrease in
circulating angiotensin II causes balanced vasodilation and inhibition of aldosterone secretion;
inhibition of tissue angiotensin II prevents myocardial hypertrophy and fibrosis; and reduced
bradykinin metabolism stimulates prostaglandin and nitric oxide synthesis. Cardiovascular
effects include reduction in right and left ventricular filling pressures and increases in stroke
volume and cardiac output.
All patients with heart failure due to left ventricular systolic dysfunction should receive
an ACE inhibitor, unless contraindicated. Similar mortality benefits are observed with several
different agents, including captopril, enalapril, and lisinopril, and in a broad range of patients,[6]
and a recent study (ATLAS) demonstrated the superiority of high-dose therapy. ACE inhibitors
are generally added to diuretics and may be used together with beta-blockers or digoxin. In
clinical practice, the choice of an ACE inhibitor may be dictated more by cost and frequency of
administration.
Hypotension and lightheadedness are common side effects of ACE inhibitors, particularly
in patients with marked activation of the renin-angiotensin system (identified by the presence of
hyponatremia or the recent occurrence of rapid diuresis). Symptomatic hypotension can be
avoided by holding diuretic therapy on the first day of treatment, and by starting with low doses
and titrating slowly to target doses.
Anonymous
• Rest-reduces metabolic demands on the heart and thus decreases workload of the heart.
• Diet-reduce the intake of sodium, in obese pts with heart failure, supervised weight loss
is of critical importance in reducing the workload of the heart.
• Diuretics-essential for treating edema associated with heart failure. By reducing sodium
reabsorption and water by the renal tubule, diuretics improve symptoms of CHF.
• Digitalis-trials have shown that “Dig” increases ejection fraction and exercise tolerance,
and decreases symptoms in pts with systolic heart failure, and withdrawal of digoxin
leads to increased sxs and hospitalizations. Dig increases cardiac contractility, cardiac
output increases and diuresis ensues. It also increases baroreceptor sensitivity, attenuates
neurohormonal activation, slows heart rate, and decreases systemic vasoconstriction.
Finally digoxin decreases AV node conduction velocity, which makes it a useful agent
for treating heart failure in pts with A-fib or A-flutter with rapid ventricular response.
• Vasodilators(ACE inhibitors)-have a balanced effect on the arterial and venous system
that slows the progression of heart failure by interfering with the renin-angiotensin
system. By inhibiting the enzyme that converts angiotensin 1 to 2, ACE inhibitors exert
several important hemodynamic and neurohormonal effects in heart failure. A decrease in
circulating angiotensin 2 causes balanced vasodilation and blocks aldosterone secretion;
inhibition of tissue angiotensin 2 prevents myocardial hypertrophy & fibrosis; and
cardiovascular effects are a reduction in right & left ventricular filling pressures &
increases in stroke volume and cardiac output.
Nobel pgs 589-591
Anonymous Goroll, p. 217-223
REST: “the activity prescription” = minimizing myocardial work demands while maintaining
the patient’s ability to live as fully as possible. This is tailored to the patient’s medical status,
lifestyle, responsibilities, and level of symptoms. In most instances, the amount of allowable
activity can be determined from an office visit by a careful history that elicits the degree of
exertion that precipitates symptoms. Treadmill testing is sometimes necessary to gauge exercise
capacity especially with comorbid coronary Dz. If weight is increasing, orthopnea is worsening,
and DOE more severe and brought on by less exertion, activity should be further restricted. A
few days of bedrest (out of bed periodically to avoid venous stasis) are often beneficial and may
obviate the need for hospitalization.
DIET: Salt restriction is helpful in preventing unnecessary exacerbations of failure. Pt. are
placed on a 4g sodium/d and instructed to prepare and serve meals without addition of salt and to
avoid foods high in salt. Supportive therapy only.
Diuretics: Initial Therapy: The degree of compensatory fluid retention in CHF is typically
excessive, leading to pulmonary congestion and/or peripheral edema. Patients with mild to
moderate CHF, respond best acutely to diuretic therapy in comparison with ACE inhibition.
However, diuretic therapy alone is usually insufficient for chronic Tx of CHF, in part because it
stimulates the rennin-angiotensin system and raises serum catecholamines, leading to increased
afterload, reduced CO, and further sodium retention. Although diuretics may be effective in
controlling peripheral edema and pulmonary congestion, they do little to prevent the progression
of heart failure or improve prognosis especially in left ventricular failure. Later Stages: In severe
CHF, absorption of loop diuretics declines, which accounts for the oft-noted reduction in
efficacy during and exacerbation of heart failure. The maximal effect of a loop diuretic can be
achieved by using a large, single daily dose rather than divided doses. Sometimes parental
therapy can be used to supplement, as well as adding other types of diuretics like thiazide,
metolazone, or spironolactone can be useful in certain situations.
Digitalis: Cornerstone of Tx for CHF with systolic dysfunction (ejection fraction (EF) < 40%)
by virtue of its ability to inhibit the myocyte sodium pump which increases intracellular calcium
and increases contractility in the failing heart. Other beneficial effects: reduces neurohumoral
responses, vasodilation, increased baroreceptor responsiveness, and increased vagal tone.
Imporovements with most patients, but best results with severe systolic dysfunction (EF < 0.25,
audible S3, and marked left ventricular dilation). However, patients with CHF in the presence of
preserved systolic function and sinus rhythm do not appear to benefit. Indication: In patients
with significant systolic dysfunction who remain unacceptably symptomatic despites a program
of loop diuretic and ACE inhibitor therapy.
Vasodilators (especially ACE inhibitors): Act on both arterial (reduce impedance of the
ejection of blood from the left ventricle) and venous (decrease preload and reduce pulmonary
and systemic venous congestion) sides of the circulation, decreasing left ventricular filling
pressure and increasing cardiac output. They bind to a receptor on the Angiotensin-Converting
Enzyme, preventing the formation of angiotensin II – a potent vasoconstrictor and stimulant of
renin & aldosterone secretion. Does not set off neurohumoral counterregulatory mechanisms of
conventional vasodilator therapy (reflex vasoconstriction, adrenergic stimulation of the heart,
and sodium retention). In addition, their suppression of the renin-angiotensin-aldosterone axis
helps prevent the harmful myocardial and vascular fibrosis and remodeling that result in long
term activation of this system. Used in mild moderate and severe dz. Reduction in heart size,
symptoms, and need for other medications also found with their use
Anonymous Currents p 383-384
A) Rest- in severe CHF restricting activity may facilitate temporary recompensation. In stable
patients however, a moderate exercise regimen is associated with diminished symptoms and
substantial increases in exercise capacity.

B) Diet- moderate salt restriction will help with the fluid retention. Extreme salt restriction
should not be done because many of the diuretics are not sodium sparing so the patient will
develop hyponatremia if they limit salt intake to extremely.

C) Diuretics- this is the most effective means of providing symptomatic relief to patients with
CHF. The increased diuresis helps to reduce excess fluid volume however, excessive diuresis
can lead to electrolyte imbalance. Most of these agents block sodium resorption in the terminal
segment of the loop on henle.

D) Digitalis- these drugs are positive inotropic agents that increase cytosolic calcium which
enhances contractile protein cross bridge formation and force generation.

E) Vasodilators (especially ACE inhibitors)- agents that dilate arteriolar smooth muscle and
lower peripheral vascular resistance reduce left ventricular afterload. ACE inhibitors have
shown to be the most effective at improving prognosis due to their nonselective ability to reduce
preload of both ventricles.

10. Examine and explain the syndrome of acute pulmonary edema.


Zen Seeker noble 585- http://home.mdconsult.com/das/book/body/0/959/444.html#bottom
When significant wheezing is associated with paroxysmal nocturnal dyspnea, it resembles an
acute asthmatic attack and may be referred to as cardiac asthma. Bronchospasm, which is caused
by congestion of the bronchial mucosa and by interstitial pulmonary edema compressing small
airways, increases the work of breathing. Acute pulmonary edema can be a further extension of
paroxysmal nocturnal dyspnea. Alternatively, acute pulmonary edema may occur as a primary
manifestation of acute myocardial infarction or accelerated hypertension. The patient is
extremely short of breath and coughs up pink, frothy sputum. Acute pulmonary edema occurs
when there is marked elevation of the pulmonary capillary wedge pressure leading to alveolar
edema. Untreated, it can be fatal.
Internet
Severe congestive heart failure (CHF) and/or acute pulmonary edema is caused by acute left
ventricular failure, resulting in pulmonary congestion. Most commonly these conditions are the
result of myocardial infarction, diffuse infection, opiate poisoning, inhalation of toxic gases and
severe over-hydration. It is characterized by intense shortness of breath, cough, anxiety,
cyanosis, diaphoresis, rales and/or wheezing. In extreme cases, patients will exhibit diaphoresis,
restlessness, apprehension and may cough up pink frothy sputum.
Anonymous
• Acute onset or worsening of dyspnea at rest.
• Tachycardia, diaphoresis, cyanosis
• Pulmonary rales(crackles), Rhonchi(snoring sound), expiratory wheezing
• X-ray shows interstitial and alveolar edema with or without cardiomegaly
• Arterial hypoxemia
• Pink frothy sputum
Pulmonary edema may appear suddenly in the setting of CHF or may be the first
Manifestation of cardiac disease, usually acute MI, which may be painful or silent.
Anonymous Noble, p. 585
Etiology- can be an extension of PND or as a primary manifestation of acute MI or accelerated
hypertension.
Pathogenesis- Marked elevation of pulmonary capillary wedge pressure leads to alveolar edema
(alveoli fill with fluid).
Clinical manifestation- pt. presents with severe SOB and coughing up pink, frothy sputum.
Bronchospasm can result from increased interstitial fluid compressing small airways.
Prognosis- can be fatal if untreated.
Anonymous Currents p 390
A) Acute Pulmonary Edema- this is the development of sudden severe dyspnea, the production
of pink frothy sputum, diaphoresis and cyanosis. Rales are present in all lung fields, as are
generalized wheezing and ronchi. Pulmonary edema may develop suddenly in the setting of
chronic heart failure or may be the first manifestation of cardiac disease.

IV. Hypertension (Top of Document)

The 6th Report of the Joint National Commission on Detection, Education,


and Treatment of High Blood Pressure came out in 1997. I would recommend utilizing this
as a source for answering some of your objectives. I would pay particular attention to
"Chapter 4: Special Populations and Situations". 7th report for 2004
Since knowledge of specific antihypertensive medications is part of the Patient
Management course, you are not responsible for that information here. The
following objectives do not appear in this exact order in the reading, so you may have
to hunt around for them a bit. They should also be addressed in class; some of them
are answered for you.

1. Define and distinguish between the following:


essential (primary) hypertension,
secondary hypertension,
resistant hypertension
isolated systolic hypertension,
hypertensive emergencies and urgencies.
Pam Current, p. 409-432
• Essential (primary) HTN: the term applied to the 95% of cases in which no cause can be
identified. This occurs in 10-15% of white adults and 20-30% of black adults in the USA.
The onset is usually between ages 25-55; it is uncommon before age 20.
• Secondary HTN: Approximately 5% of pts with NTN have specific causes. The Hx,
examination, and routine lab tests may identify such pts. In particular, pts who develop
HTN at an early age without a positive family Hx, those who first exhibit HTN when over
age 50, or those previously well-controlled who become refractory to tx are more likely to
have secondary HTN.
• Resistant HTN: www.postgradmed.com Failure to achieve a BP of 150/90 mm Hg
despite the use of a rational triple-drug regimen in optimal doses.
• Isolated systolic HTN: www.postgradmed.com Elevated systolic blood pressure in
conjunction with normal diastolic blood pressure (<90 mm Hg). Previously, systolic
pressure of more than 160 mm Hg was classified as ISH, and pressure between 140 and
160 mm Hg was classified as borderline ISH. In 1993 the definition of ISH was changed to
any systolic blood pressure above 140 mm Hg together with diastolic blood pressure below
90 mm Hg (1). ISH is characterized by an increased pulse pressure, defined as the
difference between the systolic and diastolic blood pressures.
Hypertensive emergencies and urgencies: Urgencies—situations in which BP must be
reduced within a few hours. These include pts with asymptomatic severe HTN (systolic BP >
220 mm Hg or diastolic BP > 125 mm Hg that persists after a period of observation) and with
optic disk edema, progressive target organ complications and severe perioperative HTN.
Elevated BP levels alone—in the absence of sx’s or new or progressive target organ damage—
rarely require emergency tx. Parenteral drug tx is not usually required, and partial reduction of
BP with relief of sx’s is the goal. Emergencies—require substantial reduction of BP within 1 hr
to avoid the risk of serious morbidity or death. BP is usually strikingly elevated (diastolic > 130
mm Hg). Emergencies include hypertensive encephalopathy (HA, irritability, confusion, altered
mental status), hypertensive nephropathy, intracranial hemorrhage, aortic dissection,
preeclampsia-eclampsia, pulmonary edema, unstable angina, and MI.
Anonymous Noble 511- 524 Goroll 159 Tintinalli 402
• Essential (primary) hypertension- affects more than 90% of hypertensive patients and has
no identifiable cause.
• Secondary hypertension- has an identifiable underlying cause, potentially reversible.
• Resistant hypertension- refers to the hypertensive patient with persistent and
unacceptably high diastolic pressures (>105), even in lieu of medications.
• Isolated systolic hypertension- systolic BP greater than or equal to 140 with a diastolic
BP <90. Commonly seen in the elderly and thought to be linked to atherosclerotic
thickening of the aorta and it's large branches. Significantly increases the risk of
cardiovascular morbidity and mortality in persons over the age of 65.
• Hypertensive emergencies and urgencies- Hypertensive emergencies are those situations
that require immediate blood pressure reduction to prevent or limit damage to target
organs (brain, heart, eyes, kidneys). The term malignant hypertension or hypertensive
crisis may also be used to refer to this category. The treatment goal in HTE is the
immediate reduction of mean arterial pressure in a controlled, graded manner, using
improvement in the patient's condition as the guide. Blood pressure should not exceed a
20 to 25% reduction within the first 30 to 60 minutes. Hypertensive urgency- is less
clearly defined in the literature. Most sources concur that HTU occurs when the blood
pressure elevation presents a risk for imminent target organ damage. Although acute
organ injury has not occurred in HTU, the risk of injury is high if the elevated blood
pressure is allowed to persist. The treatment goal in HTU is the gradual reduction of
blood pressure within 24 hours by using oral antihypertensive agents, although the
recommended duration to reduce the blood pressure varies in the literature from a few
hours to 48 hours. Because a common cause of HTU is noncompliance with medication,
restarting a patient on a previously prescribed regime is an acceptable strategy.
Anonymous
Essential (primary) HTN- unknown cause and most common. Onset around age 50.
Secondary- has an underlying cause. Characterized by sudden onset/worsening the onset is
distinguishing (<20 or >50)
Resistant hypertension- persistently elevated diastolic BP possibly caused by nonadherence to
Rx regimen, alcohol consumption, secondary cause, ineffective Rx therapy.
Hypertensive emergencies and urgencies (hopefully will be covered in lecture) Noble 511, 523
Anonymous
A) Essential (primary) hypertension- this is the term applied to the 95% of hypertensive cases
in which no cause can be identified. Currents p 409

B) Secondary Hypertension- this term is applied to approximately 5% of patients with


hypertension who have specific identifiable causes. Currents p 411

C) Resistant Hypertension- Almost all patients with hypertension can achieve good blood
pressure control with minimal side effects, although a few patients may persistently have
unacceptably high diastolic pressures regardless of treatment. Noble p 523

D) Isolated Systolic Hypertension- this is a type of hypertension in which only the systolic
blood pressure is elevated. This condition, which usually affects the elderly, increases the risk of
stroke or heart attack.

E) Hypertensive Urgencies- situations in which blood pressure must be reduced within a few
hours. Many of these cases are asymptomatic or have target organ symptomology. Currents p
432

F) Hypertensive Emergencies- these are cases that require substantial reduction of blood
pressure within 1 hour to avoid the risk of serious morbidity or death. Currents p 432
Anonymous
Essential (Primary) HTN
95% of HTN cases
No cause identified. Currents p 409

Secondary HTN
5% of patients w/HTN
Specific identifiable causes. Currents p 411

Resistant HTN
Some pts have persistently unacceptably ^ diastolic pressures regardless of Tx. Noble p 523

Isolated Systolic HTN


Systolic BP elevated.
Usually affects Elderly
^ risk of stroke or heart attack.

HTN Urgencies
BP must be reduced within a few hours.
Many cases are asymptomatic or have target organ symptomology.
Currents p 432

HTN Emergencies
Require substantial reduction of BP w/i 1 hour
to avoid risk of serious morbidity or death. Currents p 432

2. Identify and explain the effects of prolonged uncontrolled hypertension on


the four "target organs" or sites of end-organ damage, which are: heart,
brain, kidneys, and retina.
Pam Current, p. 412-414
• Heart: Left ventricular enlargement with a left ventricular heave indicates severe or
longstanding hypertrophy. Older pts frequently have systolic ejection murmurs resulting
from calcific aortic sclerosis, and these may evolve to significant aortic stenosis in some
individuals. A presystolic (S4) gallop due to decreased compliance of the left ventricle is
quite common in pts with sinus rhythm.
• Brain: HTN is the major predisposing cause of stroke, especially intracerebral
hemorrhage but also ischemic cerebral infarction. CV complications are more closely
correlated with systolic than diastolic BP. Preceding HTN is associated with a higher
incidence of subsequent dementia, both the vascular and Alzheimer types.
• Kidneys: Chronic HTN leads to nephrosclerosis, a common cause of renal
insufficiency. HTN also plays an important role in accelerating the progression of other
forms of renal disease, most commonly diabetic nephropathy.
• Retina: Narrowing of arterial diameter to less than 50% of venous diameter, copper or
silver wire appearance, exudates, hemorrhages, or papilledema.
Anonymous McCance 1035
Site of
Mechanism of injury Potential pathologic effect
injury
Heart ¨ Increased workload combined
Myocardium with diminished blood flow ¨ Left ventricular hypertrophy,
myocardial ischemia, left sided heart
through coronary arteries failure
Coronary
¨ Accelerated atherosclerosis ¨ Myocardial ischemia, MI, or sudden
arteries
death
Renin and aldostrone secretion Retention of sodium and water leading to
stimulated by reduced blood flow, increased blood volume and perpetuation
Kidneys
reduced oxygen supply, high of HTN, Parenchymal damage
pressure in renal arterioles compromising filtration, nephrosclerosis
Reduced blood flow and oxygen TIA, cerebral thrombosis, aneurysm,
Brain
supply, weakened vessel walls hemorrhage
Reduced blood flow, High Retinal vascular necrosis, exudation,
Eyes
arteriolar pressure hemorrhage
Anonymous Noble 508 .
Heart
Left ventricular hypertrophy
Angina
Heart failure
Brain
Stroke
Transient Ischemic attacks
Kidney
Nephopathy
End stage renal disease
Retina
Retinopathy
Anonymous Currents p 412-413/ Noble p
A) Complications of hypertension are related either to sustained elevations of blood pressure,
with consequent changes in the vasculature and heart, or to atherosclerosis that accompanies and
is accelerated by long-standing hypertension.
1. Heart- left ventricular hypertrophy is evident in up to 15% of cases of chronic
hypertension. This increases risk of morbidity and mortality and may cause or facilitate many
cardiac complications including CHF, arrhythmias, myocardial ischemia, and sudden death.
2. Brain- hypertension is the main predisposing cause of stroke, especially intercerebral
hemorrhage, but also ischemic cerebral infarction. These problems are more closely related to
systolic blood pressure than to diastolic pressures. Wihtout treatment cognative decline is
eminent but with adequate treatment the risks of this decline.
3. Kidneys- chronic hypertension leads to nephrosclerosis, that in turn limits the kindey's
effectiveness at filtering and maintaining fluid and electrolyte balances. Hypertension also
accelerates other forms of renal disease such as diabetic nephropathy.
4. Retina- hypertension leads to atherosclerosis of the retinal arteries also giving them a
characteristic copper or silver wire appearance. The arterial diameter narrows to less than 50%
of venous diameter leading to exudates, hemorrhages, or papilledema.
Anonymous Currents p 412-413/Noble
HTN Complications
Related to Sustained elevations of BP
W/consequent changes in vasculature & heart, or
Atherosclerosis that accompanies & is accelerated by long-standing HTN.
Heart
L ventricular hypertrophy up to 15% of chronic HTN
^ risk of morbidity & mortality
May cause/facilitate cardiac complications including CHF
Arrhythmias
Myocardial Ischemia
Sudden Death.

Brain
HTN = Main Predisposing cause of Stroke
especially intercerebral hemorrhage, but also ischemic cerebral infarction.
These are more closely related to Systolic BP.
W/o Tx cognitive decline eminent
W/Tx risks decline.

Kidneys
Chronic HTN leads to Nephrosclerosis
This in turn limits Kidney's effectiveness at filtering & maintaining
fluid & electrolyte balances.
HTN accelerates other forms of Renal Dz such as Diabetic Nephropathy.

Retina
HTN leads to Atherosclerosis of Retinal Arteries
giving them a characteristic copper or silver wire appearance.
Arterial diameter Narrows to < 50% venous diameter leading to Exudates
Hemorrhages
Papilledema.

3. List diseases that can result from hypertension.


Pam Current, p 412-414
• Cardiovascular: CHF, ventricular arrhythmias, MI, sudden death, left ventricular
diastolic dysfunction, artherosclerosis, dissecting aorta
• Cerebrovascular: stroke, dementia, Alzheimer’s
• Renal: nephrosclerosis, renal insufficiency
• Retina: hypertensive retinopathy (eventual loss of vision with inadequate tx)
Anonymous
• Coronary artery disease
• Congestive heart failure (6X increased risk)
• Stroke (4X)
• Renal failure
Anonymous. Goroll, p. 81
Stroke, congestive heart failure, renal failure, and left ventricular hypertrophy
Anonymous Currents p 412-413
A) Congestive heart failure, ventricular arrhythmias, myocardial ischemia and sudden death
B) Embolic and ischemic strokes, dementia
C) Hypertensive renal nephropathy and renal failure
D) Aortic dissection
E) Atherosclerosis
F) Retinal disease
Anonymous Currents p 412-413
A) CHF
Ventricular Arrhythmias
Myocardial Ischemia
Sudden Death
B) Embolic and Ischemic strokes
Dementia
C) Hypertensive Renal Nephropathy
Renal Failure
D) Aortic Dissection
E) Atherosclerosis
F) Retinal Dz

4. Compare and contrast the most common causes of secondary hypertension


by their characteristic signs and symptoms, or lab values uncovered on
routine tests.
Pam Current, p411-412
1. Estrogen use: Considerable rises in BP with contraceptive use is sometimes noted. This is
caused by volume expansion due to increased activity of the renin-angiotensin-aldosterone
system. The primary abnormality is an increase in the hepatic synthesis of renin substrate.
5% of women taking OCPs chronically will exhibit a rise in BP about 140/90 mm Hg.
Contraceptive related HTN is more common in women over 35 years of age, in those who
have taken OCPs for more than 5 years, and in obese individuals. It is less common is those
taking low-dose estrogen tablets. In most, HTN is reversible by discontinuing the
contraceptive, but may take several weeks. Postmenopausal estrogen does not cause HTN.
2. Renal disease: Most common cause of secondary HTN. HTN may result from glomerular
diseases, tubular interstitial disease, and polycystic kidneys. Most cases are related to
increased intravascular volume or increased activity of the renin-angiotensin-aldosterone
system. HTN accelerates progression of renal insufficiency. Diabetic nephropathy is another
cause of chronic HTN. This process is exacerbated by intraglomerular HTN, itself worsened
by systemic HTN.
3. Renal vascular HTN: Renal artery stenosis is present in 1-2% of HTN pts. Its cause in
most younger individuals is fribromuscular hyperplasia, which is most common in women
under 50. The remainder of renal vascular disease is due to artherosclerotic stenoses of the
proximal renal arteries. Renal vascular hypertension should be suspected in the following
circumstances: 1. if the documented onset is below age 20 or after age 50; (2) if there are
epigasteric or renal artery bruits; (3) if there is artherosclerotic disease of the aorta or
peripheral arteries; (4) if there is abrupt deterioration in renal function after administration of
angiotensin-converting enzyme inhibitors.
4. Primary hyperaldosteronism and Cushing’s syndrome: Pts with excess aldosterone
secretion make up less than 0.5% of all cases of HTN. The usual lesion is an adrenal
adenoma. The diagnosis should be suspected when pts present with hypokalemia prior to
diuretic tx associated with excessive urinary potassium excretion and suppressed levels of
plasma renin activity; serum sodium usually exceeds 140 meq/L. Aldosterone concentrations
in urine and blood are elevated. Less commonly, pts with Cushing’s syndrome
(glucocorticoid excess) may present with HTN.
5. Pheochromocytoma: although HTN due to pheochromocytoma may be episodic, most pts
have sustained elevations. The majority of pts have orthostatic falls in BP, the converse of
essential HTN; some develop glucose intolerance.
6. Coarctation of the aorta: Consists of localized narrowing of the aortic arch just distal to
the origin of the left subclavian artery. Collateral circulation develops through the
intercostals arteries and the branches of the subclavian arteries. Should be considered in
young pts with elevated BP. A bicuspid aortic valve is also present in 25% of cases.
7. HTN associated with pregnancy: HTN occurring de novo or worsening during pregnancy
is one of the commonest causes of maternal and fetal morbidity and mortality.
8. Other causes: Hypercalcemia due to any cause, acromegaly, hyperthyroidism,
hypothyroidism, and a variety of neurologic disorders causing increased intracranial P. A
number of meds may cause or exacerbate HTN—most importantly cyclosporine and
NSAIDs.
Anonymous Noble 512-513
¨ Renovascular disease- accounts for approximately 1% of HTN and is suggested by severe or
resistant HTN (diastolic pressure greater than or equal to 125mm hg), other evidence of
vascular disease, a suggestive abdominal bruit (up to 50% of RVD patients in contrast to 9%
of those with essential HTN), worsening renal function, occurrence in younger women and
older men, and asymmetric renal size.

¨ Pheochromocytoma (medullary hyperfunction)- is associated with sustained or intermittent


episodes of tachycardia, tremor sweating, pallor, headache, and orthostatic hypotension. A
spot urine test for methanephrine is the initial screening test, rare condition.

¨ Cushing's syndrome- nearly 80% of patients have elevated blood pressure because the high
levels of cortisol have mineralocorticoid activity. The initial diagnostic test is a morning
plasma cortisol after suppression with 1 mg of dexamethasone at bedtime the night before. A
cortisol level less than 5 mg/dl excludes Cushing's syndrome with 98% certainty.

¨ Primary aldosteronism- induces HTN and potassium wasting, although hypokalemia is not
always present. The most useful screening test for primary hyperaldosteronism is measuring
the plasma aldosterone/plasma renin activity ratio. A ratio of greater than 20:1 with an
absolute aldosterone concentration of greater than 15 ng/dl suggests primary
hyperaldosteronism.

¨ Hyperthyroidism, hypothyroidism, hypercalemia- these diagnoses are suggested by


characteristic clinical findings (giantism, thyromegaly, thyroid nodules) or abnormal
laboratory studies (serum calcium, thyroid function test.

¨ Coarctation of the aorta-is suggested by delayed or absent femoral pulses and decreased lower
extremity blood pressure. A chest x-ray may show the E sign, formed by the abnormal
contour of the aortic knob and the uppermost portion of the descending aorta. Notching of
the rib may be noted. An aortagram confirms the diagnosis.
Anonymous Noble p. 511 –513
Secondary hypertension – by definition, has an identifiable underlying cause, which can be
curable.
Most common cause: is Renal parenchymal disease: e.g. glomerulonephritis, chronic nephritis,
diabetic nephropathy, hydronephrosis, connective tissue disease, polycystic disease - which is
responsible for 2% to 3% of all Hypertension.
Labs: The Dx is based on finding elevated Blood urea nitrogen (BUN) and creatinine, decreased
creatinine clearance, and an abnormal urinalysis.
-Renal Vascular Disease: (RVD), Abrupt onset of severe difficult to control HTN. Patient <30
or >50 yr old. Abdominal or flank bruit. End-organ damage. Etiologies are fibromuscular
dysplasia and atherosclerosis.
Labs: Captopril renogram or duplex Doppler study of renal arteries.
-Acute stress: e.g. alcohol withdrawal, surgery, and acute illness –
-Exogenous Drugs: e.g. nonsteroidal antiinflammatory drugs [NSAIDs], estrogen, appetite
suppressants, corticosteroids, thyroid supplements, and sympathomimetics, caffeine, cocaine,
amphetamines and antidepressants.
-Primary aldosteronism: Muscle weakness, Periodic paralysis, Muscle cramps, decrease [K],
increase [Na].
Labs: measuring the plasma aldosterone / plasma rennin activity ratio.
-Pheochromocytoma: Paroxysmal HTN, Headache, diaphoresis, Palpitations, Wt loss,
tachycardia.
Labs: Spot urine metanephrines level; 24 hr urine collection for VMA (vanillylmandelic acid),
metanephrines, catecholamines, and creatinine, consider clonidine suppression.
Cushing’s syndrome: Obesity, acne, Muscle weakness, easy brushing, edema, glucose
intolerance, and striae.
Labs: Overnight 1 mg dexamethasone suppression tests 24 hr urine free cortisol, low dose
dexamethasone suppression.
Coarctation of aorta: Murmur, pulse delay, femoral pulse delay or absent.
Labs: A CXR may show the “E” sign, formed by the abnormal contour of the aortic knob. An
aortogram confirms the Dx, but CT scan or MRI can demonstrate the coarctation
Anonymous Currents p 411-412/ Noble p 511-513
A) Estrogen Use- This may cause no symptoms but blood pressure is checked in all women on
oral contractptives. These are more likely to cause hypertension if on over five years, if over 35
years of age, and less likely to cause hypertension if postmenopausal.

B) Renal Disease- Renal parenchymal disease is the most common cause of secondary
hypertension. This is commonly associated with spilling of protein in the urine, elevated BUN
and createnine, decreased createnine clearance, but may have no other symptoms. Diabetic
nephropathy leads to the same kidney problem and hypertension.

C) Renal Vascular Hypertension- This is the cause of 1-2% of secondary hypertensives most
common in women under 50 years old as a result of fibromuscular hyperplasia. Renal arteries
become stenotic and do not allow free flow of renal blood supply. The kidneys in turn respond
by activating the rennin-angiotensis system which increases volume further in attempts to get
more blood to the kidney. There is no diagnostic test, but if suspicion is high renal arteriography
is the definitive test.

D) Hyperaldosteronism (Cushing's Syndrome)- patients with an excess aldosterone secretion


often due to an adrenal adenoma make up 0.5% of cases with secondary hypertension. This is
suggested with moon faces, truncal obesity, proximal muscle weakness, and hirsutism. Lab
values will reveal:
1. Cortisol in morning after dexamethasone at bedtime the night before. Values under 5
mg/dl excludes
Cushings (Number one test to rule out Cushing's)
2. Elevated Aldosterone
3. Hypokalemia prior to initiation of diuretic therapy
4. Excessive potassium excreted in urine
5. Elevated Serum sodium

E) Pheochromocytoma- This is tumor of the adrenal medulla or sympathetic paraganglia that


results in an excessive secretion of epinephrine and norepinephrine causing persistent or
intermittent hypertension. Typical signs include headache, palpitations, sweating and
nervousness. One symptom may be orthostatic hypotension and glucose intolerances. Spot
urine screen for metanephrine is the screening test.

F) Coarctation of Aorta- This may cause hypertension and is characterized by a tearing chest
pain usually detected by delayed or absent femoral pulses and decreased lower extremity blood
pressure. Chest x-ray reveals a "E" sign formed by the abnormal contour of the aortic knob and
uppermost portion of the descending aorta.

G) Pregnancy- there is a physiologic hypervolemia during pregnancy that increases blood


pressure usually more towards the end of pregnancy. IF this becomes elevated too high the
diagnosis becomes eclampsia as protein begins to spill into the urine and edema develops.

H) Hypercalcemia, hyperthyroidism, and hypothyroidism all may cause hypertension also.

I) Medications- NSAIDs, nasal decongestants (containing sympathomimetics), corticosteroids,


diet pills, caffine, cyclosporine, erythropoetin, antidepressants, amphetamines, and cocaine can
cause hypertension.
Anonymous Currents p 411-412/ Noble p 511-513
Estrogen Use
May cause no symptoms
Check BP in all women on oral contraceptives.
More likely to cause HTN if on Tx over five years
over 35 years of age
Less likely to cause HTN if postmenopausal.

Renal Dz
Renal Parenchymal Dz - Most common cause of 2 O HTN.
Commonly associated w/Spilling of protein in Urine
Elevated BUN & Createnine
Decreased Createnine Clearance
May have No other Symptoms.

Diabetic Nephropathy leads to same Kidney problem & HTN.

Renal Vascular Hypertension


1-2% of 2O HTN
Most common in women under 50 years old
Result of Fibromuscular Hyperplasia.
Renal Arteries become stenotic & do not allow free flow of renal blood supply.
Kidneys respond by activating Rennin-angiotensis Sys
^ volume further
Attempts to get more blood to kidney.
No diagnostic test
IF suspicion is high Renal Arteriography is definitive test.

Hyperaldosteronism (Cushing's Syndrome)


Excess Aldosterone secretion often due to Adrenal Adenoma
0.5% 2O HTN.
Moon faces
Truncal Obesity
Proximal Muscle Weakness
Hirsutism.

Lab values will reveal:


1. Cortisol in morning after dexamethasone at bedtime night before.
Values under 5 mg/dl excludes Cushing’s
(Number one test to rule out Cushing's)
2. Elevated Aldosterone
3. Hypokalemia prior to initiation of Diuretic Tx
4. Excessive Potassium excreted in Urine
5. Elevated Serum Sodium

Pheochromocytoma
Tumor of Adrenal Medulla or Sympathetic Paraganglia
Excessive secretion of Epinephrine & Norepinephrine
Causing persistent or intermittent HTN.
Typical signs include Headache
Palpitations
Sweating
Nervousness.
One symptom may be Orthostatic Hypotension & Glucose Intolerances.
Spot urine screen for Metanephrine is screening test.

Coarctation of Aorta
May cause HTN
Characterized by a Tearing Chest Pain
Usually detected by Delayed or Absent Femoral Pulses &
Decreased Lower Extremity BP.
Chest X-Ray reveals an "E" Sign
Formed by Abnormal Contour of Aortic Knob &
Uppermost Portion of Descending Aorta.

Pregnancy
Physiologic Hypervolemia during Pregnancy
^ BP usually more towards End of pregnancy.
IF becomes elevated too high = eclampsia as protein begins to spill into the urine & edema
develops.

Hypercalcemia, Hyperthyroidism, & Hypothyroidism


May cause HTN.

Medications
NSAIDs
Nasal decongestants (containing sympathomimetics)
Corticosteroids
Diet pills
Caffeine
Cyclosporine
Erythropoietin
Antidepressant
Amphetamines
Cocaine

5. Specify the items that should be covered in taking the medical history of a
patient with newly discovered hypertension.
Pam Current, p410-416
Family history of HTN; PMH of renal disease, dyslipidemia, neurological disease, heart
conditions, cerebrovascular disease, peripheral arterial disease, diabetes mellitus,
prior MI, retinopathy, prior coronary revascularization, other medical conditions;
Medications and Allergies; Lifestyle modification: Smoking; Exercise
Anonymous Noble 508 .
Hypertension is usually asymptomatic, and is often detected only during routine screening.
Patients with elevated pressures should be questioned about symptoms of cardiovascular,
cerebrovascular, and renal disease. Questions about a family history of high blood pressure,
cardiovascular disease, or diabetes. If HTN has already been diagnosed, the patient should be
asked about the duration and the levels of BP. The history should also identify other
cardiovascular risk factors such as, smoking, hyperlipidemia, inactivity, obesity, and DM. Other
important factors to ask about include drug use, dietary intake of salt, intake of cholesterol, and
saturated fats. A quick review of systems may help determine whether a secondary cause may
be present.
Anonymous Goroll p. 105 – 4th ed
(The syllabus mentions The 6th Report of Joint commission but I cannot find it in Goroll and it
has not been presented in our Pt. Management class yet).
Pts should be asked date of onset, any medications ( OTC and herbal as well), family history of
HTN. The pt. should be asked about hx renal disease, alcohol intake,salt intake and recent weight
gain. They should also be asked about any use of recreational drugs. Their cardio-vascular
history should also be asked. They should also be asked the date of their most recent blood
pressure reading and the results.SMOKING history or current use should be asked.
Anonymous Noble p 509
A) Patients should be questioned about symptoms of cardiovascular, cerebrovascular, and renal
disease
B) Patients should be questioned about family history of high blood pressure, cardiovascular
disease, or diabetes
C) Ask about any complications of the hypertension the patient has experienced
D) Identification of any hypertension risk factors including:
1. Tobacco use
2. hyperlipidemia
3. Inactivity
4. Obesity
5. Diabetes Mellitus
E) Questions about drug use should be asked
F) Dietary questions about daily intake of salt, cholesterol, and saturated fats.
G) A quick review of systems may reveal causes of secondary hypertension.
Anonymous Noble p 509
Symptoms of Cardiovascular - Cerebrovascular - Renal Dz
FMHx HTN
Cardiovascular Dz
Diabetes
Complications of HTN Pt has experienced

Identify HTN risk factors including:


1. Tobacco use 2. Hyperlipidemia
3. Inactivity 4. Obesity
5. Diabetes Mellitus
Drug use
Dietary questions
Daily intake of Salt
Cholesterol
Saturated fats.

ROS may reveal causes of 2O HTN

6. Determine and describe the physical exam that should be performed on a


patient with newly discovered hypertension.
Fassil Noble online pg 509
• A rational PE in the HTN pt ideally includes checking for evidence of increased
cardiovascular disease risk factors and target-organ damage and for findings that may
suggest secondary causes of HTN. The most efficient and accurate exam for these check
is unknown. Many experts recommend measuring B/P in both arms and using the arm
with the higher reading for subsequent monitoring. In pts less than age 30, B/P
differences between arms & legs may be measured if coarctation of the aorta is being
considered. In older pt & those with orthostatic symptoms, postural change in B/P are
important. Weight & height should be checked to estimate BMI. Waiste-to-hip
circumference ratio may be measured to help establish risk of cardiovascular disease.
KEVIN; Tintinalli 405
FOCUS THE PE ON
• Measure BP with cuff of accurate size (at least twice if elevated if severe, measure both
arms/legs)
• Palpate pulses in all extrem.
• Focus PE on target organ damage
• Neuro exam that reveals mental deterioration may indicate hypertensive encephalopathy,
hemorrhage, or stroke.
• Fundoscopic exam
• Hyperreflexia with peripheral edema in a preg woman is sugg. of preeclampsia
• Auscultate carotid bruits, murmurs, heart sounds/ pericardial rub.
Colleen Gor pg 149
Hypertension is likely to need lifelong treatment; therefore it is essential to confirm the
diagnosis.
• Rule out secondary causes
• Identify any additional CV risks
• Assess presence & degree of target organ damage
Anonymous Goroll 105
Coffee intake and smoking should be halted at least 30 minutes before taking the blood
pressures. BP is properly measured in both arms while the patient is seated comfortably and
after resting for 5 minutes. In the elderly the pressure should also be taken standing looking for
any postural changes. Any auscultatory gap should be noted because it correlates with arterial
stiffness and carotid atherosclerosis.
The remainder of the exam focuses on:
• Weight and pulse measurements
• The skin for stigmata of Cushing's syndrome, chronic renal failure, or
neurofibromatosis
• Fundoscopy for arteriolar narrowing, increased vascular tortuosity, arteriovenous
nicking, or hemorrhages
• Thyroid for enlargement or nodularity
• Carotid pulses for bruits or diminution of pulse
• Lungs for sign of heart failure
• Heart for left ventricular lift and S4 and S3 sounds
• Peripheral vasculature for pulses, bruits, and abnormalities in bilateral arm and leg
pressure measurements and simultaneous radial and femoral pulse palpitation
• Abdomen for masses and bruits
• Neurological exam for focal deficits
Anonymous Noble 508
Evaluate for evidence of increased CV risk factors, target organ damage, findings that may
suggest secondary cause for HTN. Take BP bilat in older pts., check wt/ht. (for body mass
index), check waist to hip circumference ratio. Links to CV dz of: >.8% in women, >1.0 % in
men = increased risk. Examine fundus for retinopathy, check neck for thyroid abnormalities,
increased jugular pressure or carotid bruit. Check CV for dysrhythmias, murmurs. Check for
rales, edema, chest/abd. bruit, esp. lateralizing bruit w/ diastolic component - suggests renal dz.
Palp. for AAA (abd. aortic aneurysm). Check lower peripheral pulses (atherosclerosis), do neuro
exam with cognitive status screen.
Anonymous Noble p 508-511
A) A rational physical examination in the hypertensive patient ideally includes checking for
evidence of increased cardiovascular disease risk factors and target-organ damage, and for
findings that may suggest secondary causes of hypertension. The most efficient and accurate
examination for these checks is unknown.

B) Many experts recommend:


1. Measuring blood pressure in both arms and using the arm with the highest reading for
subsequent monitoring.
2. In patients under 30 years of age, blood pressure differences between both arms and
both legs may be measured if coarctation of the aorta is being considered.
3. In older patients orthostatic vital signs are important.
4. Height, weight, and calculation of IBW (ideal body weight) are important
5. Waist-to-hip ratios may be measured to help establish risk of cardiovascular disease
6. Some experts recommend examining the optic fundus for evidence of retinopathy,
although the accuracy of this examination on undilated eyes is quite low.
7. Examine the neck for thyroid abnormalities, jugular vein distention, and carotid bruits
8. Cardiac examination should focus on dysrhythmias (atrial fibrillation is common in
older hypertensive patients), S3 gallops (indicative of systolic dysfunction), and murmurs.
9. Lungs should be evaluated for rales and extremities should be evaluated for edema
10. Abdominal exam should pay particular attention to bruits especially ones that have a
diastolic component which are indicative of renal disease. Palpation of the abdominal aorta is
also important as sometimes abdominal aortic aneurysms are palpable.
11. Diminished peripheral pulses may indicate atherosclerosis
12. Neurologic examination may establish cognative function to rule out end-organ
damage of the brain.
Anonymous Noble p 508-511
Check for evidence ^ Cardiovascular Dz risk factors
Target-organ Damage
Findings that may suggest 2O causes of HTN.
Many experts recommend:
1. Measure BP Both Arms using arm w/highest reading for subsequent monitoring.
2. Pts under 30 yrs of age
BP differences between both arms & both legs may be measured if Coarctation of aorta is
being considered.
3. In older patients Orthostatic VS.
4. Height - Wt - Calculation of IBW (ideal body wt)
5. Waist-to-hip ratios help establish risk Cardiovascular Dz.
6. Some recommend - Examining optic fundus for evidence of Retinopathy accuracy on
undilated eyes is quite low.
7. Examine Neck for Thyroid Abnormalities
JVD
Carotid Bruits
8. Cardiac Exam
Focus on Dysrhythmias -Atrial Fib common in older HTN Pts
S3 gallops (indicative of systolic dysfunction)
Murmurs.
9. Lungs for Rales
10. Extremities for Edema
11. Abdominal Exam
Bruits - especially W/Diastolic Component - Indicative of Renal Dz.
Palpate Abdominal Aorta - sometimes AAA are palpable.
12. Diminished Peripheral Pulses
May indicate atherosclerosis
12. Neurologic Exam
May establish Cognitive Function
R/O End-organ Damage of Brain.

7. List the four stages of hypertensive retinopathy. (Answer: Stage I =


narrowing; II = AV nicking; III = hemorrhages and exudates; IV = all of the
above plus papilledema. Stages I & II are more chronic, and Stages III & IV
reflect more acute changes.)
Fassil
• The question was answered above. Four stages of hypertensive retinopathy:
1. Stage I = narrowing
2. Stage II = AV nicking
3. Stage III = hemorrhages and exudates
4. Stage IV = all of the above plus papilledema
• Stages I & II are more chronic and Stages III & IV reflect more acute changes.
KEVIN;
FOUR STAGES OF HTN RETINOPATHY:
• Stage I = narrowing;
• II = AV nicking;
• III = hemorrhages and exudates;
• IV = all of the above plus papilledema.
• Stages I & II are more chronic, and Stages III & IV reflect more acute changes.)
Anonymous given in syllabus
Stages I and II are more chronic changes:
• Stage I- Arteriolar narrowing
• StageII-AV nicking

Stages III and IV reflect more acute change


• StageIII- Hemorrhages and exudates
• Stage IV- all the above plus papilledema
Anonymous
Stage I = narrowing,
II= AV nicking
III = hemorrhages and exudates
IV = all of the above plus papilledema. ( Stages I&II are more chronic, III&IV more acute.)
Anonymous
A) Stage I- narrowing
B) Stage II- AV nicking
C) Stage III- hemorrhages and exudates
D) Stage IV- all of the above plus papilledema

*Stages I and II are more chronic


*Stages III & IV reflect more acute changes
Anonymous
Stage I- Narrowing
Stage II- AV nicking
Stage III- Hemorrhages & Exudates
Stage IV- All above + Papilledema
*Stages I and II More Chronic
*Stages III & IV More Acute Changes

8. Establish which laboratory tests should be obtained on a patient with newly


discovered hypertension, and explain the reason why each one is necessary.
Fassil Noble online pg 511
• Routine lab testing should include urinalysis, fasting blood sugar, K, urea nitrogen,
creatinine, total cholesterol, HDL, EKG. The EKG, UA, renal panel help to
determined the extent of hypertensive target-organ damage; fasting blood sugar &
lipids identify other cardiovascular risk factors and K levels provide baseline values
for following the biochemical effects of therapy. Some experts also recommend CBC,
24 hr protein, LDL, glycosylate hemoglobin, and uric acid levels.
KEVIN; Tintinalli 405
LAB TESTS FOR NEWLY DX’d HTN:
• BUN (Blood Urea Nitrogen)
• Creatinine
• Electrolyte
• Glucose levels
• CBC
• Urinalysis
Colleen CMDT pg414
• Hemoglobin;
• Urinalysis & renal function studies – detect hematuria, proteinuria, and casts, signifying
primary renal disease or nephrosclerosis
• Serum K+ - to seek mineralocorticoid excess
• Fasting blood sugar level – since hyperglycemia is noted in diabetes & pheochromocytoma
• Plasma lipids – indicatior of atherosclerosis risk & an additional target
• Serum uric acid – if elevated it is a relative contraindication to diuretic therapy
Anonymous Noble 511 .
Routine laboratory testing should include the following:
• Urinalysis
• Fasting blood sugar
• Potassium
• Urea nitrogen
• Creatinine
• Total cholesterol
• HDL
• ECG
The ECG, urinalysis, and renal panel help to determine the extent of hypertensive target-organ
damage; fasting blood sugar and lipids identify other cardiovascular risk factors; and potassium
levels provide baseline values for following the biochemical effects of therapy. Some experts
also recommend CBC, 24hr protein, LDL, glycosylated hemoglobin, and uric acid levels.
Anonymous Noble pg. 511
Routine laboratory testing should include urinalysis, fasting blood sugar, potassium, urea
nitrogen, creatinine, total cholesterol, HDL, and an ECG. The ECG, urinalysis, and renal panel
help to determine the extent of hypertensive target-organ damage; fasting blood sugar and lipids
identify other cardiovascular risk factors, and potassium levels provide baseline values for
following the biochemical effects of therapy. Some experts also recommend CBC, 24-hour
protein, LDL, glycosylated hemoglobin, and uric acid levels.
Anonymous Noble p 511/ Currents p 414
A) Urinalysis and Renal Function Studies- detection of hematuria, proteinuria, and casts
signifying primary renal disease or nephrosclerosis
B) Serum K+- seeking mineralcorticoid excess
C) Fasting Blood Sugar Level/ Glycosylated Hemoglobin- rule out diabetes mellitus and
pheochromocytoma
D) Lipid Panel- rule out atherosclerosis
E) Serum Uric Acid Level- if elevated this is a contraindication to diuretic therapy
F) CBC- rule out anemia of chronic disease
Anonymous Noble p 511/ Currents p 414
Urinalysis and Renal Function Studies
Detect Hematuria
Proteinuria
Casts
- signifying primary Renal Dz or Nephrosclerosis

Serum K+
Seeking Mineralcorticoid Excess

Fasting Blood Sugar Level/ Glycosylated Hemoglobin


R/O Diabetes mellitus & Pheochromocytoma

Lipid Panel
R/O Atherosclerosis

Serum Uric Acid Level


IF elevated Contraindication to Diuretic Tx

CBC
R/O Anemia of Chronic Dz

9. Outline general principles of non-pharmacologic treatment of hypertension.


Fassil CMDT 414 Table 11-2
• Lose wt if overweight
• Limit alcohol intake no more than 1 oz of ethanol
• Increase aerobic physical activity
• Reduce sodium intake to no more than 2.4g of Na or 6g of NaCl
• Maintain adequate intake of dietary K
• Maintain adequate intake of dietary Ca and Mg for general health
• Stop smoking and reduce intake of dietary saturated fat and cholesterol for overall
cardiovascular health.
KEVIN; CMDT; 414
GENERAL PRINCIPLES OF NON PHARM MNGT OF HTN:
• Lifestyle mod
• Diet
Colleen Gor pg 150
• Salt restriction
• Reduction of excess weight
• Exercise
• Diet low in saturated fat & rich in fruits & vegetables
Anonymous
• Initiation of antihypertensive treatment should be preceded by careful evaluation that
includes ruling out secondary causes, identifying additional cardiovascular risk factors,
and assessing degree of target-organ damage. Treatment usually includes both.
• The foundation of non-pharmacologic treatment of HTN are: salt restriction, reduction
of excess weight, and exercise. Behavioral therapies and calcium and potassium
supplementation have been suggested, but more essential are the other nonpharmacologic
measures that helped to limit overall cardiovascular risk (cessation of smoking, reduction
of fat and cholesterol intake, reduction of alcohol intake). Examples of behavioral
therapies are relaxation techniques and biofeedback programs.
Anonymous Noble pg. 514
Lifestyle Interventions: weight reduction, salt restriction, regular aerobic exercise, and
decreased alcohol consumption.
Exercise: exercising aerobically at least three times a week for 30-45 minutes lowers BP. It is
also a valuable adjunct to weight control. The key is to begin gradually with a realistic program
that fits the patient’s lifestyle.
Diet: decrease saturated fat (switch to polyunsaturated and monounsaturated fat) to reduce BP
and control cholesterol. A high potassium and magnesium diet rich in fruits, vegetables, and
low-fat dairy products and reduced in saturated and total fat is advised.
Weight loss: Modest weight reductions of 3% to 9% of body weight can effectively reduce BP
in obese, hypertensive people who are sufficiently motivated to follow a long-term weight loss
regimen.
Reducing Alcohol Intake: Daily alcohol consumption greater than 2 oz has been associated
with hypertension. People with raised BP should reduce alcohol consumption to two drinks of
less daily.
Salt Restriction: Patients should refrain from salting food at the table and while cooking, and
avoid processed foods with a high salt content. Salt restriction may lead to modest reductions in
BP.
Potassium Supplementation: An increase in potassium intake of about 60 mEq or 2000 mg per
day on top of typical consumption of dietary potassium produces a reduction in BP of
approximately 4 mmHg systolic and 2.5 mmHg diastolic. Greater reductions may be seen in
persons with higher urinary sodium excretion and in African Americans compared with white
persons. About 2% to 10% of persons taking potassium supplementation have GI adverse effects
(belching, flatulence, diarrhea, and abdominal discomfort.)
Fish Oil Supplementation: Large daily intakes of fish oil reduced BP by approximately
4.5mmHg systolic and 2.5 mmHg diastolic. Such high intake may be difficult to maintain (200
gm/day in fish high in polyunsaturated fatty acids or 6-10 capsules/day). There is no evidence of
beneficial effect on BP at lower intakes. Belching, bad breath, fishy taste, and abdominal pain
may occur.
Calcium and Magnesium Intake: Calcium supplementation has minimal effects on BP in
people with hypertension and normal calcium levels. There is no evidence that magnesium
supplementation reduces BP in people with hypertension and normal magnesium levels.
Relaxation and Biofeedback: Some trials show minimal reductions, but others with sham
control groups show no effects.
Smoking Cessation: Smoking cigarettes raises BP slightly and may attenuate the cardiovascular
protective effect of antihypertensive therapy. Smoking cessation is strongly encouraged because
it is a major risk factor for cardiovascular disease that interacts synergistically with hypertension.
Anonymous Noble p 514-515
A) The aim of treating high blood pressure is to prevent premature death and disease by
reducing the risk of coronary heart disease and stroke, with minimum adverse effects. Treatment
options for hypertension include lifestyle modifications, dietary supplementations, and
pharmacologic therapy.

B) Non-pharmacologic treatment options:


1. Lifestyle Interventions: weight reduction, reduced salt intake, regular aerobic
exercise, decreased alcohol consumption.
2. Exercise- aerobic exercise at least 3 times a week for 30-45 minutes lowers blood
pressure and assists in weight control. Patients should be counseled to begin at five minutes and
work gradually up to the optimum 30-45 minute work outs.
3. Diet- Reduce saturated fat intake. Diets high in potassium and magnesium consisting
of fruits, vegetables, and low fat dairy products can reduce blood pressure.
4. Weight Loss- weight loss of 3-9% can effectively reduce blood pressure
5. Reduce Alcohol Intake- daily alcohol consumption greater than 2 oz. Has been
associated with hypertension.
6. Salt Restrictions- salt restrictions trying to remain under 1 gram intake daily have
been shown to reduce blood pressures
7. Dietary Supplimentation- There is no direct evidence of the effect of dietary
supplimentation on mortality or morbidity in people with hypertension. A few randomized trials
have shown potassium supplimentation to lower blood pressure but the studies are considered
inconclusive at this time.
8. Relaxation and Biofeedback- studies show from mild to no results
9. Smoking Cessation- tobacco is known to raise blood pressure and lead to
arteriosclerosis so smoking cessation should be strongly recommended.
Anonymous Noble p 514-515
Prevent premature death & disease
Reduce risk of Coronary Heart Dz & Stroke w/minimum adverse effects.
Tx options HTN include Lifestyle modifications
Dietary supplementations
Pharmacologic therapy.

Non-Pharmacologic Tx options:
1. Lifestyle Interventions:
Wt reduction
Reduced salt intake
Regular Aerobic Exercise
Decreased Alcohol Consumption.

2. Exercise
Aerobic Exercise @ least 3 times a wk for 30-45 mins lowers BP pressure and assists in Wt
Control.

Pts should begin @ five minutes & work gradually up to the Optimum 30-45 minute work outs.
3. Diet
Reduce Saturated Fat Intake.
Diets high in Potassium and Magnesium
consisting of fruits, vegetables, & low fat dairy products can reduce BP

4. Weight Loss
Wt loss of 3-9% can effectively reduce BP

5. Reduce Alcohol Intake


Daily alcohol consumption greater than 2 oz. Has been associated w/HTN

6. Salt Restrictions
Salt restrictions trying to remain under 1 gram daily shown to reduce BP

7. Dietary Supplementation
No direct evidence of effect of dietary supplementation on mortality or morbidity in people
w/HTN.
Few randomized trials shown Potassium supplementation to Lower BP
Studies considered inconclusive.

8. Relaxation and Biofeedback


Mild to no results

9. Smoking Cessation
Tobacco known to raise BP lead to Arteriosclerosis

Smoking Cessation should be strongly recommended.

V. Valvular Heart Disease (Top of Document)

1. Describe and distinguish the typical presentation of each of the following:


aortic stenosis,
aortic regurgitation,
mitral stenosis,
mitral regurgitation,
mitral valve prolapse.
Ky. Noble p596-608 & CMDT p322-332
aortic stenosis:
S/S: delayed & diminished carotid pulses. Sx of faiure may be sudden in onset or progress
gradually. Angina pectoris. Exertional syncompe due to arrhythmias. Hypotension. ↓
ceregral perfusion resulting from ↑ blood flow to exercising musle w/out compensatory
↑ in CO. Sudden death rare but possible.
Inspection: sustained PMI, prominent atrial filling wave.
Palpation: powerful, heaving PMI to left and slightly below MCL. Systolic thrill over aortic
area, sternal notch, or carotids. Small and slowly rising carotid pulse.
Heart Sounds: A2 normal, soft, or absent. Paradoxic splitting of S2 if A2 is audible. Prominent
S4. Systolic pressure normal with high diastolic or normal BP. Heard best when pt
resting, leaning forward, breath held in full expiration
Murmur location: Right second ICS parasternally or at apex, heard in carotids and
occasionally upper interscapular area.
Timing: midsystolic begins after M1, ends before A2, reaches maximum intensity in mid
systole.
Character: Harsh, rough.
X-Ray: concentric LV hypertrophy. Prominent ascending aorta, small knob. Calcified valve
common
aortic regurgitation:
S/S: usually asymptomatic until middle age. Presents with left sided failure or chest pain.
Wide pulse pressure with associate peripheral signs. Hyperactive, enlarged LV.
Exertional dyspnea and fatigue. Paroxysmal nocturnal dyspena and pulmonary edema.
Angina pectoris or atypical chest pain. Syncope.
Inspection: hyperdynamic PMI to left of MCL and down. visible carotid pulsations.
Palpation: apical implulse forceful and displaced significantly to left and down. prominent
carotid pulses. Rapidly rising and collapsing pulses.
Heart Sounds: S1 normal or reduces; A2 loud. Wide pulse pressure with diastolic pressure <
60. Best heard pt leaning forward, breath held in expiration.
Murmur location: Diastolic: lounder along left sternal border in 3rd to 4th interspace. Heard
over aortic area and apex. May be associated with low pitched middiastolic murmur at
apex in nonrheumatic dz.
Timing: begins immediately after aortic second sound and ends before first sound.
Character: blowing, often faint.
X-Ray: moderate to severe LV enlargement. Prominent aortic knob
mitral stenosis:
S/S: dyspnea, orthopnea and paroxysmal nocturnal dyspnea. Fatigue and fatigue on exertion.
Right side heart failure. Severe pulmonary congestion may alsoe be initiated by any
acute respiratiory infxn, excesinve salt and fluid intake, endocarditis, rheumatic
carditis. Hemoptysis due to brochial submucosal varices.
Inspection: malar flush, precordial bulge, and diffuse pulsation in young pts
Palpation: “tapping” sensation over area of expecte PMI. Middiastolic or prsystolic thrill at
apex. Small pulse. Right ventricular pulsation left 3rd to 5th ICS parasternally when
pulmonary HTN is present.
Heart sounds: loud snapping M1. Opening snap following S2 along LSB or at apex. Atrial
fibrillation common. Normal BP. Because it is thickened the valve opens early diastole
with snap. Best heard after exercise, left lateral recumbency. Bell chest piece lightly
applied
Murmur location: localized at or near apex. Rarely, short diastolic murmur along lower LSB
in severe pulmonary HTN
Timing: onset at opening snap (middiastolic) with presystolic accentuation if in sinus rhythm.
Character: low-pitched, rumbling. Presystolic murmur merges with loud M1 and ends at or
after A2. May be late systolic in papillary muscle dysfunction
X-Ray: straight left heart border. Large left atrium sharply indenting esophagus. Elevation of
left main stem bronchus. Large RV and pulmonary artery if pulmonary HTN is present.
Calcification occasional in mitral valve.
mitral regurgitation:
S/S: may be asymptomatic for many years or may cause L sided failure. Exertional dyspnea
and fatigue progress over years. Predisposed to infective endocarditis. Nonspecific
chest pain, palpitations. Most are female, many are thin and have minor chest wall
deformities.
Inspection: usually prominent and hyperdynamic apical impulse to left of MCL
Palpation: forceful brisk PMI systolic thrill over PMI. Pulse normal, small or slightly
collapsing
Heart sounds: M1 normal or buried in murmur. Prominent 3rd heart sound. Atrial fibrillation
common. BP normal. Midsystolic clicks may be present. Best heard after exercise;
diaphragm chest piece. In prolapse findings most prominent while standing.
Murmur location: loudest over PMI;. Transmitted to left axilla, left infrascapular area. With
posterior papillary muscle dysfunction, may transmit to base
Timing: pansystolic: begins with M1 and ends at or after A2. May be late systolic in papillary
muscle dysfunction.
Character: blowing, high pitched; occasionally harsh or musical
X-Ray: enlarged LV and Left atrium
mitral valve prolapse:
S/S: affects ~5% of general population. Prevalence men = women. Vast majority
asymptomatic. Associate dwith pectus excavatum or straight back. Most common
complaints 1) long lasting, sharp stabbing chest pain unrelated to exertion; 2)
palpitations
Timing: mid to late systolic click. Occurs reproducibly at critical systolic volume.Aoritic or
pulmonic ejection clicks with upstroke of carotid pulse. Pulmonic click varies with
respiration. Occasionally pansystolic. Best heard when ↓ cardiac filling—standing,
valsalva, amyl nitrite administration.
Character: occasionally honking in quality.
Echo: leaflet thickening. LA or LV enlarghement and mitral regurg.
EKG: 1/3 repolarization abnormalities like T-wave flattening or inversion in inferior leads.
Variable ST-segment changes
Anonymous Nobel pg 603-604
• Aortic stenosis-6th decade, can remain symptom free for a long time. Onsets of
symptoms are dyspnea, angina, and exertional syncope. Nobel pg 597
• Aortic regurgitation-can be acute, severe, & chronic. Pts. With chronic regurgitation are
often asymptomatic at time of dx. Essential features are-bounding,collapsing(water
hammer) peripheral pulses with a widened pulse pressure; a downward, laterally
displaced, hyperdynamic apex impulse; and the usual high pitched, blowing diastolic
murmur is heard along the LSB. Acute regurgitation presents with a bounding pulse and
low diastolic pressure of the chronic counterpart are absent. And, lowered aortic pressure
and tachycardia conspire to obscure the diastolic murmur, which often assumes a lower-
pitched, coarser quality and may evade detection. Severe regurgitation presents as the
cause of aortic dissection and needs prompt surgery. These pts verge on shock. Mortality
outweighs endocarditis(infection) with surgery. Nobel pg 598-600
• Mitral Stenosis-dyspnea, pulmonary venous congestion, fatigue & pulmonary HTN,
hemoptysis, s2-os(interval between the second heart sound and the opening snap). CXR
reveals dilation of the left atrium(lateral view) and dilation of right pulmonary artery to
15 to 18 mm, rapid tapering of vessels, and right ventricular enlargement strongly
suggests advanced mitral stenosis. Presence of Kerley B lines, perihilar haze, and other
manifestations of interstitial edema are seen in pts with severe dyspnea with mitral
stenosis; the absence of interstitial edema on CXR does not r/0 tight mitral stenosis.
CXR is not specific but it can provide supporting evidence. ECG- signs appear to be the
QRS axis; a right shift to >60 is assoc with a valve. Echo is a sensitive test for evaluating
mitral stenosis. Goroll pg 230.
• Mitral valve prolapse-a midsystolic click with or without a late systolic murmur. Occurs
at all ages. Equal in men and women. Can be asymptomatic. Two most common
complaints are chest pain and palpitations. Sharp or stabbing chest pain is felt, duration is
often protracted, no relation to exertion, no relief from nitro meds. Pt can present with
pectus excavatum or straight back. HALLMARK of MVP is a mid to late systolic
click.
Anonymous Noble pg. 596-9
-Aortic Stenosis: Most common acquired valvular disease. Usually develops as a consequence
of congenital bicuspid valvular anatomy in two of every three cases. Rheumatic fever may also
cause aortic stenosis. Pt’s are usually asymptomatic, but cardinal sxs are dyspnea, angina and
exertional syncope.
-Aortic Regurgitation: Usually caused by congenitally bicuspid aortic valve, can also be
caused by endocarditis, aortic aneurysm, rheumatic fever, and syphilis. Most pts. are
asymptomatic when diagnosed. Characteristic murmur: high-pitched, blowing diastolic heard
best along the left sternal margin.
-Mitral Stenosis: May be related to extensive calcification of the mitral annulus. Classic
rheumatic mitral stenosis is not very common anymore. Usually seen in the elderly or for pts.
who previously had undergone surgical palliation. The course of rheumatic mitral stenosis
includes a prolonged asymptomatic latency-over 20 yrs after the episode of rheumatic fever. Sxs
such as dyspnea may become worse during times such as pregnancy and cardiovascular stress.
-Mitral Regurgitation: Common causes of mitral imcompetence are mitral valve prolapse,
ruptured chordae tendineae, rheumatic fever, tissue erosion related to endocarditis, muscle
ischemia or infarction. Usually asymptomatic.
-Mitral Valve Prolapse: This is now the most prevalent and problematic concern for the
practitioner today. Primary MVP is caused from inherent abnormalities in the mitral valve.
Secondary MVP is caused from other conditions-these include: connective tissue disorders
(Marfan syndrome), secundum atrial septal defect, coronary artery disease. Common complaints
are chest pain and palpitations. Chest pain is usually sharp or stabbing, duration often protracted
and nitroglycerin usually doesn’t relieve the pain.
Anonymous Noble p 596-604
A) Aortic Stenosis- This stenosing of the aortic valve is due to a congenital bicuspid valvular
anatomy in two of every three cases. Rheumatic fever can also be implicated in aortic stenosis.
The cardinal symptoms are dyspnea, angina, and exertional syncope. Upon physical exam the
best place to discern this murmur is by palpation of the carotid arteries where reduced pulse
volume and delayed upstroke are noted. Systolic vibration or shudder may be palpable as
evidence of this stenotic jet-related turbulence.

B) Aortic Regurgitation- This is most likely due to a congenital bicuspid aortic valve however
endocarditis, connective tissue disorders (Marfan Syndrome), aortic aneurysm, rheumatic fever,
syphilis, and aortic involvement of rheumatoid arthritis can also be implicated. Most often these
are asymptomatic at the time of diagnosis. The heart can compensate for a long time, however
eventually the left ventricle weakens and signs of CHF or angina appear. The physical exam
reveals bounding peripheral pulses with a widened pulse pressure and a high-pitched blowing
diastolic murmur is usually heard along the left sternal border.
C) Mitral Stenosis- Classicaly mitral stenosis was due to rheumatic fever, however in the last 2
decades the prevalence of this has declined and now most commonly the causes are elderly or
patients who have previously undergone surgical or percutaneous palliation. The symptoms
often are not evident for years of mitral stenosis but become evident in situations of stress on the
heart such as pregnancy, anemia, and febrile conditions. Atrial fibrillation often develops as the
first sign in patients that have had undiagnosed, asymptomatic mitral stenosis for years.
Emblematic of mitral stenosis is accentuation of S1 or the opening snap due to the opening of the
stenotic valve best heard over the apex impulse.

D) Mitral Regurgitation- Competence of the mitral valve relies on valve leaflets, chordae
tendinae, papillary muscles, and mitral annulus. Mitral regurgitation may be caused by
dysfunction of any one of these commonly as a result of rheumatic fever, ruptured chordae
tendinae, papillary muscle ischemia or infarction, tissue erosion due to late endocarditis, and
calcification of the mitral annulus. The ventricle can compensate for some time but upon reserve
failures congestive symptoms develop of which pulmonary congestion may become rapidly fatal
if untreated. The apical impulse is enlarged, displaced downward to the left and is rapidly
retracted. An attenuated S1 coincides with the onset of a high-pitched holosystolic murmur
which may radiate to the axilla and extend beyond A2.

E) Mitral Valve Prolapse- Primary mitral valve prolapse is due to myxomatous degeneration
that disrupts the normal connective tissue architecture of the valve leaflets, chordae, or annulus.
Secondary mitral valve prolapse can be a result of many things such as rheumatic fever,
Marfan's, coronary artery disease, etc. This valvular abnormality is also associated with thoracic
cage abnormalities such as pectus excavatum and straight back. Most individuals are
asymptomatic but the most common complaints are chest pain and palpitations. The hallmark of
mitral valve prolapse is the mid to late systolic click which is the only detectible manifestation in
more than half the cases.

2. Describe the typical presentation, complications, most common causative


organisms, and diagnostic studies for infective endocarditis.
Ky. Noble p623-24. CMDT p1361.
Defn: infxn of valvular and nonvalvular endothelial surfaces of the heart. Most frequently
caused by bacteria. Can be caused by fungi, rickettsiae, mycoplasmas, or chlamydiae.
Classified as subacute bacterial endocarditis (SBE) or acute bacterial endocarditis (ABE).
SBE usu occurs in previously damaged valves and is due to relatively avirulent organisms
Factors that impact presentation: nature of organism; valve or valves infected; route of infxn.
Typical Presentation:
SBE: indolent course with night sweats, fevers, malaise. Anemia of chronic dz and normal
white count. Fever < 102. Anorexia. Fatigue. Unexplained stroke. Sudden unilateral
blindness. Splenomegally. Hematuria. Proteinuria. Negative CXR.
ABE: occurs on normal and damaged valves and in association with IV drug use or
intravascular catheters or devices. Usually more virulent bacteria. Fever > 102. CHF
findings. Pleuritic chest pain. CVA tenderness. Marked leukocytosis. CXR pneumonia
like = pulmonary emboli.
Other S/S: acute febrile illness (several days to 2 weeks); Non specific sx include cough,
dyspnea, arthralgias, diarrhea, abdominal or flank pain indicating embolization, stable
heart murmur, petechiae on palate/conjunctivae, under fingernails, Subungual “splinter”
hemorrhages; Osler nodes (painful violaceous raised lesions of fingers toes or feet);
Janeway lesions (painless erythematous lesions of palms or soles) Roth spots (exudative
lesion on retina) Pallor.
Complications: ABE: clinical course is fulminant with rapid valvular destruction and high
incidence of metastatic dz, early embolization, acute valvular regurgitation and myocardial
abscess formations.
Other complications: Renal/splenic dysfunction from emboli. Immunologically mediated
glomerulonephritis. Abscess can cause conduction disturbances. Sinus of valsalva
aneurysms. Cerebral and myocardial emboli resulting in infarction. Right side IE of tricuspid
valve often leads to septic pulmonary emboli causing infarction and lung abscesses.
Causative Organism: SBE: viridans streptococci & enterococal. ABE: staph aureas, strep
pneumo, pseudomonas aeruginosa
Diagnostic Procedures: Blood cultures. CXR. Echocardiography
Anonymous Nobel pg 623-625
• Presentation- the classic signs and symptoms of SBE(sub acute bacterial
endocarditis) are fever, anemia, splenomegaly, a cardiac murmur, and embolic phenomena.
Occur two weeks after infection after bacteremia. Non specific sx’s such as fever, weight
loss, night sweats, low back pain, arthralgias, or malaise may be present in SBE due to
relatively avirulent organisms(e.g. veridans streptococci).
Infection with more virulent pathogens such as Staph. Aureus result in ABE(acute bacterial
endocarditis). Sx’ of ABE are fever >102 F, most common finding. However, fever can be
absent in elderly or debilitated pts, in pts with CHF or renal failure, or in IVDAs(iv drug
users). Heart murmurs are present in 80% of pts but can be absent in right-sided or mural
endocarditis. PE may have findings of CHF in ABE. Some splenomegaly is seen in ABE.
Petechia or splinter hemorrhages are seen in small % of pts. Oslers nodes(tender nodular
lesions of the fingers & toes) and Janeway lesions(painless hemorrhagic macules on the
palms & soles) are common. Clubbing may occur in ABE 2nd to S. Aureus. Roth’s spots-
which are pale, retinal lesions surrounded by hemorrhage, are seen in <5% of pts. Severe low
back pain suggests enterococcal endocarditis. Up to 1/3 of pts may have monarticular &
polyarticular septic arthritis with ABE. Focal neurologic deficits may occur as a result of
systemic septic embolization in ABE or immunologic-mediated bland embolus (aseptic
meningitis) in SBE.
• Lab abnormalities
o Normochromic normocytic anemia
o Leukocytosis with a left shift
o Elevated ESR
o (+) VDRL
o Elevated RF
o UA-hematuria & proteinuria
o Blood cultures (+)
o Circulating immune complexes in SBE
o Snynovial fluid analysis ABE(septic arthritis) SBE(aseptic arthritis)
o CSF ABE(purulent meningitis profile) SBE(aseptic meningitis profile)
o CXR ABE(pneumonia-septic pulm emboli) SBE(-)
o Brain CT/MRI- ABE(+ if cerebritis, micro abscess, or hemorrhage) SBE(-
unless mycotic aneurisms or embolic CVAs)
Noble pg 264-625 or look at Table 69-1
• Complications-for most pts medical therapy with 4-6 weeks of ABX is curative.
However, valvular replacement may be necessary for certain complications:
Intractable heart failure
Recurrent systemic emboli
Inability to eradicate infection due to large vegitation or resisitant organism
Fungal endocarditis
Myocardial abscess
Prosthetic valve dysfunction
• Most common causative organisms-
Culture- True culture-negative
Culture-Positive Culture-Negative
Positive endocarditis(serologic
Acute (apparently)
Sub-Acute dx)
Normal valve: staph Native valve: Haemophililus species, Aspergillus
Aureus, viridans Acinetobacter legionella
pseudomonas streptococci, actinomycemcimitans,
Aeruginosa, Serratia enterococcal Cardiobacterium hominis,
marcescenes Prosthetic Eikenella corrodens,
Prosthetic valve valve(late) Kingella kingae
(early). Staph aureus, Staph
streptococcus epidermidis
pneumoniae, Viridans
pseudomonas streptococci
aeruginosa
Box 69-1 Nobel pg 624
Anonymous Noble, p.623-627—great table p. 25
Presentation: Classic S/Sx of SBE are fever, malaise, nightsweats, anemia, splenomegaly,
cardiac murmur, back pain, myalgias, arthralgias, and embolic phenomena.
Complications: Emboli, bacteremia, abscesses, heart block, dysrhythmias, murmurs, neurologic
deficits, CHF.
Causative organisms: ABE= staph aureus, strep pneumo, or pseudomonas (virulent orgs).
SBE= strep viridans (non-virulent orgs). EITHER can be caused by non-bacterial agents such as
fungi, rickettsiae, mycoplasmas, or chlamydia.
Diagnostic studies: Blood cultures, echocardiography, TEE.
SBE usually occurs on previously damaged valves, due to relatively avirulent organisms, is
indolent.
ABE occurs on normal/damaged heart valves or in assoc with IVDU, or intravascular catheters
Anonymous Currents p 1361-1362
A) Typical Presentation- Infective endocarditis can vary in presentation depending on the
nature of the infecting organism, which valve or valves are affected, and the route in infection.
Most patients present with a febrile illness that has lasted for several days to 2 weeks.
Nonspecific symptoms of cough, dyspnea, arthralgias, or arthritis are common. Characteristic
peripheral lesions include: petechiae on the palate or conjunctiva or beneath the fingernails;
subungual splinter hemorrhages; Osler nodes (painful violaceous raised lesions on the fingers,
toes or feet); Roth Spots (exudative lesions on the retina); as well as pallor and splenomegaly.

B) Complications- the clinical course of infective endocarditis is determined by the degree of


damage to the heart and the degree of metastatic infective loci. Complications to infective
endocarditis may include:
1. Destruction of infected valve leading to severe regurgitation
2. Infection into myocardium may lead to abscess formation that may alter electrical
conduction
3. Peripheral embilization of infective particles can result in cerebral, myocardial,
spleenic and kidney involvement. Peripheral emboli may also become established in vessel
walls leading to mycotic aneurysms.
4. Right-sided endocarditis usually involving the tricuspid valve often leads to septic
pulmonary emboli causing infarction and lung abscesses.

C) Most Common Causative Organisms Include:


1. 60% of cases are due to viridans streptococci
2. 20% of cases are due to S. aureus
3. 5-10% are due to enterococci
4. Gram negative organisms and fungi account for a very small percentage

D) Diagnostic Studies for Infective Endocarditis Include:


1. Blood cultures- isolate organisms known to cause endocarditis
2. Chest x-ray- may show underlying cardiac abnormality andin right-sided endocarditis
pulmonary infiltrates become evident on chest x-ray
3. ECG- although nondiagnostic this may show electrical current changes that may be a
result of endocarditis abscesses within the myocardium
4. Echocardiography- this is the test of choice providing the most information and
although more invasive the best results are evidenced with a transesophageal echo.

3. Determine who is at increased risk for developing infective endocarditis and


explain the importance of antibiotic prophylaxis before certain medical or
dental procedures in this population.
Ky. Noble p626. CMDT p1362
Prophylactic ABX given to pts with predisposing congenital or valvular anomalies.
High risk: prosthetic cardiac valves; previous IE; complex cyanotic congenital heart dz;
surgically constructed systemic pulmonary shunts
Moderate risk: most congenital cardiac malformations; rheumatic and other acquired valvular
dysfunction; hypertrophic cardiomyopathy; mitral valve prolapse with valvular
regurgitation.
Anonymous Goroll 87-89
Patients with valvular heart disease are at the highest risk of developing bacterial endocarditis.
Prosthetic valve endocarditis is a serious condition; overall mortality is substantial, even with
antibiotics and valve replacement. Untreated endocarditis is uniformly fatal. Current mortality
rates are 10% with natural valves and 25-65% with prosthetic valves.
(Goroll Table 16.1) Cardiac risk factors for endocarditis
1. High risk (prophylaxis recommended)
a. Prosthetic heart valves
b. History of endocarditis
2. Moderate risk (prophylaxis recommended)
a. Rheumatic or other acquired valvular heart disease
b. Congenital heart disease (excluding atrial septal defect of the secundum type)
c. Idiopathic hypertrophic subaortic stenosis
d. Mitral valve prolapse with regurgitation and/or thickened leaflets
3. Probable Moderate risk (Prophylaxis not recommended)
a. Mitral valve prolapse without regurgitation
b. Undiagnosed murmers
Data on incidence of bacteriema with procedures are fragmentary at best, but risk appears
greatest with dental procedures. Tooth extraction and periodontal surgery have incidence rates
of about 50%; other dental procedures approach that figure. Most medical procedures have
lower incidence except transurethral resection of prostate with infected urine which has a 50%
rate of bacteremia. Prophylaxis is recommended for tonsillectomy/adenoidectomy, surgery
involving intestinal or respiratory mucosae, cystoscopy, or urethral dilation. Clinical
effectiveness of endocarditis prophylaxis is difficult to demonstrate definitively. However, when
the serious morbidity and mortality associated with the disease are weighed against the small risk
associated with prophylaxis, vigorous preventive efforts have been considered justified.
Anonymous
Key risk factors are history of valvular disease, recent medical or dental procedures, infections
and intravenous drug abuse. (Noble, p. 624)
Although prophylaxis is estimated to prevent only 6% of cases of infective endocarditis
following medical/dental procedures, current guidelines recommend prophylaxis with antibiotics
for patients considered at high or moderate risk (see table). Prophylaxis is generally not
recommended for mitral valve prolapse unless significant regurgitation is present. (Goroll, pp 87-
88
High Risk Moderate Risk Probable Moderate
Risk
Prosthetic Heart Rheumatic or other acquired valvular disease Mitral Valve Prolapse
Valve(s)
History of endocarditis Congenital heart disease (excluding ASD Undiagnosed
secundum) murmurs
Idiopathic hypertrophic subaortic stenosis
Anonymous Noble p 623- 627
A) Patients at increased risk include:
1. Patients with or who have had rheumatic heart disease in the past
2. Congenital Heart Disease: bicuspid valves, ventricular septal defects, coarctation of
the aorta
3. Degenerative disease such as calcific aortic stenosis
4. IV drug users have an increased risk of introducing bacteria into the blood stream

B) Guidelines for antibiotic prophylaxis:


1. The use of prophylactic antibiotics in individuals with underlying cardiac
abnormalities undergoing bacteria introducing procedures is common practice to prevent the
introduced bacteria from forming an endocarditis in these cardiac tissue irregularities.
a. Dental and Upper Respiratory Procedures- 2 grams amoxicillin po 2 hours prior to
procedure
b. PCN-allergic Patients- single dose of clindamycin, cephelexin, or azithromycin
c. GI or GU surgery- IV/IM ampicillin and gentamicin for high risk patients or low risk
patients receive amoxicillin

VI. Venous Disease of the Lower Extremity (Top of Document)


1. Define and explain the typical causes, presentation, diagnostic studies, and
basic principles of treatment of each of the following:
varicose veins,
thrombophlebitis (deep and superficial),
chronic venous insufficiency,
stasis ulcers.
Raynaud’s Phenomenon or Disease
Deb/Current
Causes: Presentation: Diagnostic Tx:
Studies:
Varicose Veins Primary May have Duplex Non-
Pg.457 varicosities=Inheri dull,aching ultrasound to Surgical=knee-
ted vein wall or heaviness from detect precise high or thigh-high
valvular defect periods of location of elastic
Secondary=valve standing. incompetent compression
damage following Dilated, valves/Brodie- stockings, worn
thrombophlebitis, tortuous, Trendelenburg all day to reduce
trauma, DVT elongated veins test less accurate venous
on medial but can hypertension due
aspect of thigh differentiate to pooling and
and leg usually saphenofemoral removed at night.
visible. valve Leg elevation and
Smaller, flat incompetence exercise
blue-green from perforator recommended.
reticular veins, vein Surgical=indicatio
spider veins incompetence/Pt ns-persistent and
may lies down, put disabling pain,
accompany tourniquet on recurrent
v.v.’s midthigh, stand superficial
up, if varicosities thrombophlebitis,
stay collapsed, erosion of
valvular overlying
insufficiency is skin/stab avulsion
at surgery is
saphenofemoral combined with
junction. prevention of
reflux by high
ligation of
saphenofemoral
junction.
Stripping of entire
saphenous sys. is
rarely required.
Thrombophlebi often 50% pts have Duplex Anticoagulation
tis of Deep multifactorial, no sx/pts may ultrasound-most with heparin
Veins Virchow’s triad complain of widely used test (initial bolus 100
Pg.458 (stasis, vascular dull ache, in initial eval., units/kg then
injury, and tightness, or includes both B 10units/kg/h, to
hypercoagulability pain in calf or mode image and goal partial
) defines events leg when Doppler flow thromboplastin
that predispose. walking/PE=sli analysis. time of 1.5-2
Trauma to ght edema of (findings times normal).
endothelium— involved calf, suggestive=abse Warfarin is started
platelet aggregates palpable cord, nce of after therapeutic
form on vein wall distention of spontaneous heparinization, the
and deposition of superficial flow, loss of two therapies
fibrin, leukocytes, venous flow variation should overlap to
and collaterals, or with respiration, avoid possibility
erythrocytes/in 7- low-grade fever and failure to of
10 days thrombus and increase flow hypercoagulable
becomes adherent tachycardia. velocity with state. 3-6 months
to vein wall and Homan’s sign distal of warfarin to
inflamm. changes (dorsiflex foot) augmentation) maintain goal INR
develop. is pos. in only Ascending 2.0-3.0
50%. Can have contrast
cyanosis of skin venography=rare
ly used study b/c
invasive and
expose pt
ionizing
radiation. Use
with pt. highly
suspected, but
U/S equivocal.
Thrombophlebi Maybe Pt has dull pain, Plastic IV Administration
tis of spontaneous in pt induration, catheters should NSAID drugs,
Superficial with varicose redness, and be observed local heat,
Veins veins, tenderness daily for signs of
elevation.
Pg.461 pregnant/postpartu correspond to local Ambulation is
m women, or pts dilated, inflammation encouraged. Sx
with thrombosed and removed if usually resolve in
thromboangiitis superf. local reaction 7-10 days.
obliterans or veins/chills and develops (no Excision of
Behcets disease. high fever specific tests involved vein
Maybe also suggest septic mentioned) recommended for
trauma or suppurative sx persist over 2
phlebitis weeks on
treatment, or
recurrent phlebitis
to same vein
segment./ Septic
requires tx with
IV antibiotics.
Chronic Most often 2ndary Progressive Can use tests It is incurable but
Venous to DVT, other edema of leg already manageable.
Insufficiency causes=leg that begins at mentioned above Bedrest with leg
Pg.462 trauma, varicose ankle and calf just to elevation, chronic
veins, neoplastic and accompany differentiate care=intermittent
obstruction to by dull aching betw. This and elevation of legs
pelvic veins, or discomfort/Ede the above during day and
congenital or ma is worst at things. Other eleva. during
acquired end of day and disorders don’t night, avoid long
arteriovenous improves with respond to periods sitting or
fistula elevation. Stasis elevation like standing, daily use
dermatitis, this does.(no compression
brownish mention of any stockings, regular
pigmentation, other tests) exercise
brawny
induration, and
ulceration
develop with
long standing
dx. Skin is thin,
shiny, atrophic,
and cyanotic
Stasis Ulcers May be acute or Maybe acute N/A Weeping
Pg.462 chronic, if pts weeping dermatitis=bedrest
don’t adhere to dermatitis, or , elevation, wet
lifelong routine large and saline
intermittent leg chronic ulcer compresses/syste
elevation, mic antibiotics
exercise, and use and topical
compression antifungals only if
stockings active infection
suspected/venous
ulcerations=wet-
to-dry dressings
and ace wrap
compression or
Unna boot
(layered dressing
of medicated
bandage)./Large,
chronic=wide
debridement and
skin grafting
Raynauds If idiopathic- Intermittent Must evaluate pt Warmth and
Pg. 453 called raynaud’s attacks of pallor for possible protection of
Phenomenon or disease/ if assoc. of hands or inciting systemic hands are the
Disease with autoimmune fingers brought disorders. H&P basics of tx. In R.
dx, on by cold or and serologic phen., wounds
myeloproliferative emotional testing maybe heal slowly, and
disorders, multiple stress, helpful in infections are hard
myeloma, progressing to excluding to control. Wear
cryoglobulinemia, cyanosis and collagen- gloves,
arterial occlusive then rubor on vascular moisturizing
dx, etc. its called rewarming. disorders/ r/o loction to avoid
raynaud’s Mild carpal tunnel fissured dry skin.
phenomenon discomfort, syndrome/ r/o Smoking cessation
paresthesias, arterial occlusive is imperative,
numbness, and disease address stress
trace edema/in management.
Raynauds dx- OCP’s beta-
dx is blockers, and
symmetric/in R. ergotamines are
phen.-changes assoc. with
most noticeable exacerbation of
in one hand or symptoms./
even one or two Vasodilator drugs
fingers some benefit with
only/infrequentl pts that sx not
y feet and toes adequately
are involved. controlled with
simpler measures
(nifedipine,
diltiazem)/Bypass
surgery for severe
R. phen. Assoc
with
reconstructible
arterial occlusive
dx.
Sandra G, Noble pp.632-633, Taber's, Current pp. 487
Topic: Circulation: Leg Ulcers, Blood vessel and lymphatic disorders
1. D: Definition
2. C: Cause
3. Cx: Complications
4. PP: pathophysiology
5. RF: Risk Factors
6. SS: Signs and Symptoms
7. Dx: Diagnostic Studies (Dx)
8. Tx: Treatment

In Noble, Post-phlebitic syndrome (syn, thrombophlebitis) and Varicose Veins are addressed
together. Both arise from incompetent venous valves. Varicose Veins or Dz (incl. Raynauds)
can lead to venous stasis that sometimes lead to the thrombophlebitis.
Varicose Veins (VV)
D: enlarged, twisted, superficial veins most common in LE or esophagus
C: high venous pressure due to incompetent venous valves from congenital (primary)
or acquired (secondary) conditions
PP:
RF: pregnancy, obesity, portal HTN (esophageal), prolonged standing (occupational)
SS: pain, swelling, skin ulcers, bleeding 2° to trauma
Dx: Ultrasound (US), Physical Exam (PE)
Tx: Preventive: rest, elevation, external support (stockings)
Surgical: ligation and excision

Thrombophlebitis: (syn. Phlebitis, DVT)


D: inflammation of a vein in conjunction with formation of a thrombus
C: Virchow's triad: stasis, hypercoagulability, vascular Injury (SHI)
PP: High venous pressure leads due to incompetent valves leads to capillary leakage of
fluid and RBCs. Tissue reacts to Hb from RBCs with inflammation and fibrosis.
Ulceration is probably secondary to stasis, hyoxemia and necrosis of tissue.
RF: same as VV
SS: varicose veins, pitting edema progressing to induration (hardening), itchy scaling
rash around ankle followed by brown pigmentation, then ulceration.
Dx: Clinical.
Can apply tourniquet to elevated extremity to that superficial veins are empty.
Then stand pt, release tourniquet. Distal veins enlarge quickly is valve dysfunction
is present.
If in doubt, venogram may show valve destruction or prior endothelial damage
(feathery appearance)
Tx: Difficult. Support hose when ambulatory. Elevation while sleeping. Surgical tx is
declining as veins are seen as needed for subsequent bypass.

Chronic Venous Insufficiency/ complication of Venous stasis Ulcers


D: chronic elevation in venous pressure (valve destruction secondary to thrombophlebitis
leads to flow disturbance with activity.
C: most often DVT…but neg. hx of phlebitis in 25%. Other causes: leg trauma, varicose
veins, neoplastic obstruction of pelvic veins, congenital or acquired AV fistula.
SS: Progressive edema with dull, achy pain, worse at end day. Varicosities, subcutaneal
and cutaneous edema and fibrosis, hyperpigmentation(brown in whites), later
dermatitis and ulceration.
Skin: shiny, atrophic, cyanotic. Cellulitis. Possible weepy dermatitis. Ulcers are
PAINLESS, with irregular outline. Healing produces thin scar susceptible to trauma
chronically.
Dx: may be hard to differentiate from primary varicose veins or acute deep venous
thrombosis.
Tx: bed rest, elevation, wet compress. Antibiotics if infected. Topical hydrocortisone .
Wet to dry dressings for ulcers. Unna boot (layered dressing of medicated bandage,
then gauze, the elastic wrap). Weekly debridment in large, chronic ulcers. Venous
reconstructive surgery.

Raynauds dz, reversible, idiopathic. (idiopathic (eg idiot, we don't know the cause) dx in
pts with suggestive sx persisting over 3 yr w/o evidence of associated dz, as per
Current).
Raynaud's phenomenon(if associated with a trigger).
Prognosis, dz vs phenom: Distinction implicates prognosis. The phenomenon can
progress to atrophy, gangrene. Dz incident in females 15-45. Hard to distinguish
between dz and phenom.
D: episodic vasospastic disorder characterized by digital pallor, cyanosis and rubor(upon
rewarming) in response to cold or environmental stress
C: precipitating factors: Regional: automimmune dz, myeloproliferative disorder,
multiple myeloma, cryoglobulinemia, myxedema (hypothyroidism), arterial
occlusive dz. Incidence 10% (80% female). Abn. Sympathetic nervous stystem
suspected etiology.
DDx: scleroderma, SLE, RA, Carpal tunnel, hx of frostbite, chemotherapy, etc.
SS: pain, paresthesia, numbness, trace edema. Fingers normal between attacks. Can
involve toes.
Tx: preventive: warmth, protective (ie Gloves!), moisture, avoid vasocontrictors (ie
nicotine, smoking cessation), stress mngmt,. d/c drugs assoc. with exacerbation of
symptoms, eg oral contraceptives, betablocker, and ergotamines (?).
Pharmacotherapy: aspirin, to decrease thrombotic complications. Vasodilators:
nifedipine, diltiazem, SSRIs.
Surgery in severe cases of arterial occlusive dz.
Prognosis: that of associated dz
Anonymous
Varicose Veins
• The superficial veins of the lower extremities lie in the subcutaneous tissue and lack the
support afforded by muscle and fascial compartments. It is unclear whether the primary
problem is a congenital incompetence of valves or a weakness of the venous wall itself,
which causes dilation of the vein lumen and subsequent valve inadequacy. In any event,
a self-perpetuating cycle ensues of venous reflux leading to further vein dilation and
valve failure. In time, the poorly supported veins widen, elongate, and become tortuous.
Factors that raise intraluminal vein pressure such as repeated pregnancies, obesity, or
wearing of tight garments that constrict the thigh may be of importance. The final
common pathway remains valvular incompetence.
• Clinical presentation usually involve local aching or burning pain most commonly
involving the veins of the greater saphenous system and its tributaries occurring in the
medial and anterior thigh, calf, and ankle region. “Tiredness”, “heaviness”, or a
“bursting” sensation are commonly reported particularly after being on your feet all day.
Itching due to a stasis dermatitis may occur in the region of a severe or chronic varix.
• Diagnostic studies include demonstrating the incompetent valves of varicose veins by
applying a tourniquet on an elevated extremity so that the superficial veins are empty.
The patient then stands; release of the tourniquet allows the vein distally to enlarge
quickly if incompetent valves are present.
• Treatment includes using heavy gauge elastic stockings. This prevents symptoms,
edema, and further enlargement of the veins. Panty girdles or garters are never worn.
Obese patients are encouraged to lose weight and prolonged standing should be avoided.
Ligation and stripping of veins have decreased in popularity since the veins may be
needed in the future for arterial bypass. Goroll 242-43, Noble 632-33

Thrombophlebitis (deep and superficial)


• Three venous systems exist in the lower extremities. The superficial network involves
the greater and lesser saphenous system. The deep system is composed of the anterior
and posterior tibial and peroneal veins, popliteal veins, and superficial and deep femoral
veins. The third system is the communicating veins which permit flow from the
superficial to the deep system. The cause of acute thrombus formation in the superficial
venous system is often unclear, but in most instances factors contributing to intimal
damage, stasis, and hypercoagulability (Virchow’s triad) comprise the pathophysiologic
determinants. The condition almost always arises in varicose veins and is clearly a result
of static blood flow in these channels. In DVT (deep vein thrombophlebitis), the
immediate cause is an identifiable self-limited event such as surgery, new onset of
paralysis, stasis, trauma, pregnancy, or some other acquired factor in half of the cases; in
the other half of cases, no clinical precipitant is evident. Estrogen use, immobilization,
and CHF are among the precipitants of acquired causes of DVT.
• Clinical presentation in superficial thrombophlebitis is pain, tenderness, and erythema
along the course of the vein which may also be palpated as a tender cord or knot. In
DVT, the patient classically complains of pain in the limb, worsens with motion, walking
or dependency, and gets better with rest or elevation of the extremity. Leg edema below
the level of the clot, pain on compression of the knee, a Homan’s sign, and a palpable
cord are cited as the classic physical findings. With extensive DVT, a dusky cyanosis
may appear. Unilateral leg edema stands out as the most sensitive indicator of DVT.
• Diagnostic studies are typically used to identify DVT while superficial thrombophlebitis
diagnosis is made on clinical grounds. Studies include compression venous ultrasound
which has emerged as the most sensitive and specific of available noninvasive methods
for diagnosis of DVT, especially when standard two-dimensional (duplex scanning) is
enhanced by color-flow Doppler technology (so called triplex scanning). Doppler
ultrasound sensitivity and specificity exceed 97% in DVT above the knee. It is less
sensitive for DVT below the knee (about 60%), but it is not clinically important because
the risk of embolization associated with a clot below the knee is negligible. Impedence
plethysmography remains in widespread use as a noninvasive alternative to Doppler
ultrasound. D-dimer testing is best applied in conjunction with noninvasive vascular
studies to rule out DVT because the specificity is only 48-65%. The gold standard for
diagnosis is contrast venography but several disadvantages render it undesirable as an
initial test. Not only can it be quite uncomfortable, but it occasionally induces phlebitis.
• Treatment of superficial thrombophlebitis is a combination of local heat and
compression with a good elastic stocking. Anti-inflammatory agents such as aspirin may
be useful. Women taking birth control pills should probably discontinue their use. The
patient should avoid sitting or standing but remain ambulatory to minimize the chance of
developing a DVT. If it extends above the knee, consideration of anticoagulation or
ligation of the saphenous vein at the level of the saphenofemoral junction in the groin
may be indicated, and surgical consultation should be considered. DVT above the knee is
associated with a high risk of thromboembolization. Standard therapy of acute DVT
involves prompt hospitalization for immediate intravenous treatment with unfractionated
heparin and initiation of oral anticoagulation with warfarin. Heparin and warfarin are
continued until the Prothrombin time (as measured by INR >2.0) has been in therapeutic
range for 3-4 days at which time heparin is stopped, warfarin is continued, and the patient
is discharged. The advent of low-molecular-weight heparin provides the opportunity for
outpatient treatment of acute DVT. Its principal mechanism is the same as for
unfractionated heparin but there is reduced interaction with platelets and thrombin
making possible equivalent efficacy with less risk for bleeding. DVT below the knee
pose little risk for embolization so long as the clot does not propagate up into the thigh.
Outpatients suspected of having DVT, but without evidence of extension above the knee
do not require immediate anticoagulation; however they should be followed closely with
serial evaluation for evidence of propagation. Repeat noninvasive study using Doppler
ultrasound or plethysmography is indicated. Goroll 247-48

Chronic Venous Insufficiency and Stasis Ulcers (Goroll 243-47)


• It is also called postphlebitic syndrome, a common chronic disorder that is particularly
disabling if stubborn venous ulcers develop.
1. Although the superficial venous system may be secondarily involved with varicose
veins, the principal defect lies in the deep venous system. DVT is believed to be the
underlying etiology in most instances.
2. The phlebitic inflammatory process deforms or destroys venous valves in the deep
system, and their incompetence results in reflux and increased venous pressure.
3. Venous hypertension results in edema usually most prominent in the calf and ankle
region. Venous hypertension leads not only to interstitial fluid accumulation but also
extravasation of plasma proteins and red blood cells into subcutaneous tissues.
4. In time, this results in browny induration of the skin and pigmentation of the
thickened but fragile tissue.
5. The presence of edema and continual high venous pressure results in reduced local
capillary flow and relative hypoxia, which further increase the likelihood of tissue
breakdown and subsequent healing difficulties.
6. Eventually these processes and accompanying infection lead to damage of
lymphatics, aggravating swelling and local tissue breakdown resulting in stasis ulcers.
• Clinical presentation usually center around swelling or ulceration of the lower leg.
Chronic recurrent swelling causes a sensation of tightness or bursting and heaviness or
aching of the limb. With chronicity, brawny induration, hyperpigmentation and skin
ulceration ensue.
• Diagnostic studies are used to distinguish the leg edema from other causes such as
lymphatic obstruction, hypoalbuminemia, and DVT. Any accompanying leg ulcers must
be differentiated from those due to arterial insufficiency. Noninvasive studies such as
Doppler and two-dimensional ultrasound techniques are helpful.
• Treatment
1. Management before ulceration
• Knee length heavyweight elastic stockings is prescribed and must be worn
religiously from the moment the patient gets out of bed until retiring at night.
• Periodic elevation during the day is essential in most patients. It must be
emphasized to the patient that the leg must be above the level of the heart for
this to be effective.
• Mild diuretic therapy (HCTZ 50 mg qd) may be of some help.
• The chronic and incurable nature of the problem must be made clear to the
patient with reassurance given that the symptoms and problems are
controllable and preventable by strict adherence to the above program
2. Management after ulceration
• Preferred treatment is an extended period of bedrest with elevation of the
involved extremity above the heart at all times. That is combined with wet to
dry saline dressings to the ulceration applied three times daily.
• For patients who cannot afford extensive time off their feet, an Unna Paste
Venous Boot may be used. It is a flesh-colored gauze roll bandage
impregnated with zinc oxide, calamine, glycerin, and gelatin. Properly
applied, this medicated bandage can supply good compression, does not
require much patient cooperation, and allows the patient to remain
ambulatory. It is changed every 7-10 days.
• Surgical referral is advisable for recurrent or nonhealing ulcerations, because
surgical interruption of incompetent communicating veins underlying these
areas, together with stripping and ligation of associated superficial
varicosities, may be indicated. In recent years, some surgeons have reported
good results with direct repair of incompetent venous valves or interposition
of a competent valve from an arm vein into the deep venous system of the leg.

Raynaud’s Phenomenon or Disease Goroll 919-921


• Raynaud’s phenomenon is strongly suggested when patients report bilateral blanching
and discomfort in the fingers in response to cold or stress, followed by purplish
discoloration and reactive erythema. Alteration in dermal blood flow is a basic
mechanism of thermoregulation and is also a common response to stress-induced
catecholamine release. In patients with Raynaud’s phenomenon, this vascular reactivity
is exaggerated. Clinically, it is classified as primary if no evidence of an associated
condition can be found and secondary if one can be found. Secondary Raynaud’s
phenomenon is associated with connective tissue disease such as scleroderma and
systemic sclerosis. A host of other conditions can cause vasoocclusive Raynaud’s
phenomenon including atherosclerotic disease, occupational vibratory injury, and
neurovascular compression.
• Clinical presentation includes an initial pallor of the hands due to digital artery
vasoconstriction which may be followed by cyanosis due to slow blood flow. When the
vessels reopen, a reactive hyperemia occurs imparting a bright red color to the digits.
During the first phase numbness is usual. Pain is more common in the reactive
hyperemic stage. Episodes may last minutes to hours; they may terminate spontaneously
or by warming the digits.
• Diagnostic studies depend on whether the patient is judged to have definite primary
disease on the basis of a careful history and physical. In primary disease, no further
testing is required. Primary disease is suggested by onset in the teens, female sex,
occurrence of multiple mild attacks every day, symmetric involvement, precipitation of
attacks by stress, normal skin except for livedo reticularis (painless reddish blue mottling
of the skin of the extremities due to exposure to the cold and emotional distress). Patients
who present with a presentation suggestive of a secondary disease such as skin rash,
morning stiffness, arthalgias, joint swelling, fatigue, and fever should receive further
evaluation. ANA testing, CBC, and serum globulin is used to screen for connective
tissue disease and common hematologic problems. Noninvasive studies of arterial flow
(plethysmography, Doppler ultrasonography) can confirm anatomic vascular
compromise.
• Treatment of Raynaud’s:
1. Prevention by keeping the trunk and extremities warm on cold days is essential.
2. Smoking cessation and elimination of passive smoking is also essential
3. Occupational precipitants such as repetitive activity that leads to carpal tunnel
syndrome should be reduced or eliminated.
4. Calcium channel blockers have proved symptomatically useful for patients with
primary disease and many who have secondary disease.
5. Alpha blocker therapy is also capable of providing symptomatic relief.
6. Patients with vasospastic disease accompanied by substantial endothelial injury and
platelet activation may not obtain adequate symptomatic relief with the sole use of
vasodilator drugs. The addition of aspirin or dipyridamole has helped to heal some
ulcers but has not reduced vasospastic symptoms.
7. Fish oil supplements rich in omega-3 fatty acids can impair platelet activation and
stimulate vasodilation through prostacyclin synthesis.
8. IV prostacyclin and iloprost (a prostacyclin analogue) are being studied; they are
both vasodilators and inhibitors of platelet activation. Effects last up to 3 months
after a single 5 day course of therapy.
9. Sympathectomy has been used as a measure of last resort. Patients who respond to
temporary ganglionic blockade with an injected anesthetic are the best candidates.
Anonymous
-Varicose veins: Noble pg. 632 Distended, tortuous veins with incompetent venous valves-as a
result of high venous pressure distending and stretching the veins or destructing the valves from
thrombophlebitis- that may appear following phlebitis or pregnancy or with no apparent
instigating cause. HX: Usually pt’s have no symptoms, some pts. c/o fatigue or aches in the
lower part of the leg or swelling at the end of the day. PE: Can be seen as dilated, tortuous,
sacculated (containing a little bag or sac) superficial veins. Diagnostic studies: Incompetent
valves can be demonstrated by applying a tourniquet on an elevated extremity (this makes the
superficial veins empty). The pt. stands, release the tourniquet and this allows the vein distally to
enlarge quickly if incompetent valves are present. TX: Uncomplicated varicose veins respond
well to heavy-gauge elastic stockings.
-Thrombophlebitis (deep and superficial): Goroll pg. 243 -Inflammation of a vein associated
with thrombus formation. Superficial- cause: intimal damage, stasis and hypercoagulabililty.
LE: The condition almost always arises in varicose veins and is a result of static blood flow in
these channels. If found in UE: it is usually after having an IV. PE: Pain, tenderness and
erythema along the course of the vein, may also palpate a tender cord or knot. TX: UE/LE:
Combo of local heat and compression (with a good elastic stocking), anti-inflammatory agents.
LE: Women on OCP’s should dc use and pts. should avoid sitting/standing for long periods of
time to avoid chance of developing deep vein thrombosis. Sxs usually resolve in 1-2 weeks.
Deep- cause: heredity-factor V Leiden, acquired-adenocarcinoma, pregnancy, estrogen use,
immobilization, CHF, trauma, recent surgery. HX/PE: Pt. c/o unilateral leg edema (> 3cm
difference in calf diameter)-with or without calf pain- and a + Homan’s sign. . If untreated, may
lead to PE. A PE alone in inconclusive and a thorough history from pt. should be done as well.
Studies: Duplex scanning and enhanced by color-flow Doppler technology. Contrast
venography is the gold standard for diagnosis. Don’t use as initial test. TX: DVT above the
knee-prompt hospitalization for IV unfractionated heparin and initiation of coumadin.
-Chronic Venous Insufficiency: Goroll pg. 243 AKA: postphlebitic syndrome Common
chronic disorder that can be very disabling if stubborn venous ulcer develop. The defect lies in
the deep venous system. High venous pressure generated by muscular contraction forces blood
through damaged valves in communicating veins toward the superficial system, resulting in
“ambulatory venous hypertension”. This then results in edema, usually in calf and ankle, and
leaking out of plasma proteins and RBC’s in subcutaneous tissues. In time, this results in
brawny induration of skin and pigmentation of thickened but fragile tissue. HX: pt often c/o
chronic recurrent swelling which causes a sensation of tightness, heaviness or aching in limb.
SXS are often worse at end of the day, disappears overnight. Chronic conditions note brawny
induration, hyperpigmentation and skin ulceration. TX: Patient education-emphasize elastic
support (heavyweight), periodic elevation during day. A mild diuretic may be helpful with
stubborn edema.
-Stasis Ulcers: Noble pg. 802 cause: common mechanism of too much hydrostatic pressure in
the superficial venous system of the leg. Most are caused by incompetent valves in deep veins.
HX/PE: poor nutritional status, diabetes, underlying diseases, CHF, vasculitis, connective tissue
disease, hx of DVT, smoking. Stasis dermatitis is usually pigmented and indurated. Stasis
ulcerations develop within areas of dermatitis or indurated cellulites. Occur often above the
medial malleolus due to poor vascular supply and sparse subcutaneous tissue. May/may not be
painful, base of ulcer usually moist with exuberant granulation tissue. Studies: CBC, glucose,
ESR, albumin, protein and thyroid screen. Vascular studies are needed when pulses are
nonpalpable. TX: Moist wound dressing and compression therapy is best treatment. Once
healed, need for preventive care should be discussed with patient.
-Raynaud’s Phenomenon: Noble pg. 631 Episodic, ischemic attacks of the digits induced by
cold or emotional stimuli. cause: Primary syndrome: no underlying cause can be found. Occurs
more frequently in women. Secondary syndrome: connective tissue diseases (scleroderma,
SLE), traumatic vasospastic disease, hammer hand syndrome, drugs (B-blockers), Neurogenic
causes (thoracic outlet syndrome, carpal tunnel syndrome). HX/PE: c/o sharply demarcated
blanching or cyanosis during an “attack”. May be one-two fingers but later all fingers or both
hands are involved. Episodes may last minutes to hours, may terminate spontaneously or by
warming the digits. Studies: WBC, Hgb, ESR, protein electrophoresis, UA, complement levels,
ANA, RA. TX: Reassurance that the prognosis is benign. Keep body and extremities warm to
prevent reflex sympathetic vasoconstriction. Wear loose-fitting warm clothing. No smoking.
Nifedipine, a calcium channel blocker, decreases the frequency, severity and duration of
vasospastic attacks and is most effective treatment.
Anonymous
A) Varicose Veins: Currents p 457-458
1. Cause- Most commonly in the lower extremities these are abnormally dilated,
elongated, and torturous alterations in the saphenous veins and their tributaries. Breakdown in
valves allow pooling and backflow of venous blood.
2. Presentation- Patients will present with dull, aching heaviness or a feeling of fatigue
brought about by standing for long periods. Itching from an associated eczematoid dermatitis
may occur above the ankle.
3. Diagnostic Studies- Primary varicose veins must be differentiated from secondary
varicose veins especially if in young patients. Genetic testing can be performed to rule out
inherited Klipper-Trenaunay Syndrome. Pain should also be distinguished from arthritis,
radiculopathy, or arterial insufficiency.
4. Treatment- support hose is usually the treatment of choice with 20-30 mmHg
pressure appropriate. Surgery is indicated with persistent debilitating pain and recurrent
thrombophlebitis.

B) Thrombophlebitis (deep veins): Currents p 458-460


1. Cause- Although the cause us often multifactorial, Virchow's triad of venous stasis,
vascular injury, and hypercoagulability define the events that predispose a vein to the
development of thrombophlebitis.
2. Presentation- Many are asymptomatic but patients may complain of dull ache,
tightness, or pain in the calf or leg especially when walking. Physical examination may reveal
slight edema, the presence of a tight cord, or a low grade fever.
3. Diagnostic Studies- Duplex ultrasonography is the most widely used based on the
concept that a normal vein is compressible with the ultrasound wand, but an inflamed one is not.
Ascending contrast venography used to be used but with its invasive means ultrasound is the
most common.
4. Treatment- Standard treatment is with as systemic anticoagulant of heparin followed
by Warfarin therapy that inhibits the synthesis of natural anticoagulant proteins C and S.

C) Thrombophlebitis (superficial veins): Currents p 461-462


1. Cause- These may occur spontaneously in patients with varicose veins, in pregnant or
postpartum women, in patients with thromboangitis, or as a result of trauma such as a direct blow
to the leg.
2. Presentation- Patients usually experience a dull pain in the region of the involved
vein. Induration, redness, and tenderness correspond to dilated, thrombosed superficial veins.
3. Diagnostic Studies- Physical exam reveals a linear rather than circular lesion and the
distribution along the course of a superficial vein help to differentiate superficial phlebitis from
cellulites or deep vein thrombosis.
4. Treatment- Treatment consists of administration of anti-inflammatory drugs, local
heat, and elevation. Ambulation is also encouraged.

D) Chronic Venous Insufficiency: Currents p 462


1. Cause- This is most often secondary to deep venous thrombosis. Other possible
causes are leg trauma, varicose veins, obstruction of the pelvic veins, or congenital or acquired
arteriovenous fistula. These result in a chronic elevation in venous pressure.
2. Presentation- Presentation often is progressive edema of the leg that begins at the
ankle and calf and is accompanied by a dull aching discomfort. This is often worst at the end of
the day and improved by elevating the extremity. Varicosities are often present, stasis
dermatitis, brownish pigmentation, brawny induration, and ulceration develop with long-standing
disease.
3. Diagnostic Studies- Diagnosis is by exclusion with CHF and chronic renal failure
also causing the symptoms. Rule out these as well as other venous insufficiencies making this a
diagnosis of exclusion.
4. Treatment- Treatment must start with the realization that this is an incurable but
manageable disorder.
Chronic care of the leg includes: 1-intermittent elevation of the legs during the day
and at night. 2-avoidance of long periods of standing or sitting with legs down. 3- daily use of
knee or thigh high ted hose or compression stockings. 4- regular exercise

E) Stasis Ulcers: Currents p 462-463


1. Cause- chronic venous stasis and increased venous pressure
2. Presentation- weeping dermatitis that transforms into an open ulcer
3. Diagnostic Studies- wound culture to ensure the ulcer is sterile or if organisms
isolated treat accordingly
4. Treatment- If there are organisms present in the ulcer treat systemically accordingly.
The ulcer itself can be treated with wet-to-dry dressings and ace wrap compression.
Alternatively the unna boot (medicated gauze wrap) can be applied but it must be kept dry,
covered by an elastic wrap. Occasionally in chronic cases venous repair surgically must be
performed and in cases of large nonhealing ulcers debridement must be performed.
F) Raynaud's Phenomenon or Disease: Currents p 453-454
1. Cause- This is caused by a vasospastic disorder characterized by digital color change
(white-blue-red) with exposure to cold environment or emotional stress. If idiopathic this is
referred to as Raynaud's disease, but if associated with a precipitating systemic ailment such as
autoimmune disorders, myeloproliferative disorders, etc. it is referred to as Raynaud's
Phenomenon.
2. Presentation- Intermittent attacks of pallor of the hands or fingers brought on by
exposure to cold or emotional stress progressing to cyanosis and then rubor. Mild discomfort,
parastesias, numbness, and trace edema often accompany the color changes. If Raynaud's
disease the extremities affected are symmetric, but if Raynaud's Phenomenon the extremity may
be unilateral or only affect a few fingers on one hand.
3. Diagnostic Studies- Directed history, physical exam and serologic testing are
necessary to rule out other collagen-vascular disorders such as scleroderma, SLE, and RA.
4. Treatment- mainly treatment is with warmth and protection of the hands

2. Identify and recognize the following platelet disorders:


Thrombocytopenia (ITP, HIT, TTP and qualitative disorders)
Platelet Dysfunction
Deb/Current, pg.505-509
ITP (idiopathic thrombocytopenic purpura)-IgG autoantibody is formed that binds to
platelets, other hematopoietic cell lines normal, no systemic illness, spleen non-palpable,
normal bone marrow with normal or incre. Megakaryocytes
HIT (I think it must have been called this Hemolytic-uremic syndrome) microangiopathic
hemolytic anemia, thrombocytopenia and renal failure, elevated serum LDH, normal
coagulation tests, absence of neurologic abnormalities
TTP(thrombotic thrombocytopenic purpura) thrombocytopenia, microangiopathic hemolytic
anemia, neurologic and renal abnormalities, fever, normal coagulation tests, elev. Serum
LDH
Qualitative disorders=Von Willebrands dx(autosomal dominant, prolonged bleeding time,
reduced levels of factor VIII antigen, factor VIII coagulant activity)
Platelet Dysfunction=Glanzmann’s Thrombasthenia (platelets unable to aggregate b/c lack
receptors for fibrinogen)/Bernard-Soulier Syndrome (platelets can’t adhere to
subendothelium b/c lack receptors for von Willebrand factor)/Storage pool dx=mild bleeding
disorders
Anonymous Goroll 523-24, Noble 1047
Thrombocytopenia (ITP, HIT, TTP, and qualitative disorders
Thrombocytopenia is defined as a platelet count below 100,000 and is associated with a
prolonged bleeding time. The risk for serious bleeding does not occur until the platelet count
falls well below 50,000. Easy bruising is seen with counts of 30,000 to 50,000. Spontaneous
bruising, menorrhagia, and prolonged bleeding with trauma arise as the count drops below
20,000. Below 10,000, spontaneous epistaxis, GI and GU bleeding, and an increased risk for
CNS hemorrhage occur.
• Idiopathic Thrombocytopenic Purpura is one of the major causes of platelet
destruction. The majority of cases in children follow infection, most often viral, but
some do not have a known precipitating cause. Although a platelet bound
immunoglobin G (IgG) antibody has been detected in 80% of cases and found useful for
diagnostic purposes, its pathophysiologic significance remains to be determined. The
condition is most common in women, young adults, and children. It may present as
bleeding in the context of aspirin or NSAID use, an incidental finding on routine blood
count, or as menorrhagia, epistaxis, or purpuric rash following a viral infection.
Petechiae may be seen especially in the dependent extremities. Splenomegaly is not
seen. The physical examination is otherwise normal.
• Thrombotic Thrombocytopenic Purpura causes platelet consumption rather than
outright destruction, as hyaline-platelet complexes form in small vessels trapping
platelets. No consumption of clotting factors or fibrin deposition is seen. The cause is
unknown. It is a rare disorder. The use of platelet aggregation inhibitor ticlopidine has
been associated it. In most instances, an immune mechanism is postulated.
Microangiopathic hemolytic anemia ensues, fever develops, and microvascular damage
to the kidneys and CNS may become manifest. The combination of anemia, a peripheral
smear showing schistocytes, and thrombocytopenia in conjunction with fever, neurologic
symptoms, and renal dysfunction comprises the clinical syndrome. It was once a highly
fatal disorder, but now is being treated with simultaneous administration of fresh frozen
plasma and plasmaphoresis. High-dose adrenal corticosteroids may provide some benefit
and in some cases splenectomy may also be a benefit.
• Heparin-induced Thrombocytopenia is the most common cause of drug-induced
thrombocytopenia. It occurs after 6-8 days of intravenous heparin therapy although it can
occur sooner especially in patients recently exposed to heparin, and it may develop after
exposure to subcutaneous therapy or even heparin flushes used to maintain catheter
patency. It is an immune-mediated thrombocytopenia. The diagnosis is based on the
clinical presentation with other causes excluded. A rapid fall in the platelet count to
levels below 10,000 can occur, and acute hemorrhage may ensue. Paradoxical
thromboembolism is the most worrisome complication. Cessation of heparin in
mandatory and alternative anticoagulation with a low-molecular weight heparinoid is
indicated.

Qualitative Platelet Disorders (Platelet Dysfunction)


• Platelet function defects can be classified according to the step in platelet activity that is
affected: adhesion, aggregation, activation, secretion, or acceleration of coagulation.
Patients with isolated qualitative platelet defects have a prolonged bleeding time in
conjunction with normal-appearing platelets that are adequate in number.
o Defective adhesion, the most important condition being von Willebrand’s disease
which is a autosomal dominant disease. It is associated with decreased secretion or
abnormal synthesis of the von Willebrand’s factor needed for platelet adherence to a
site of vascular endothelial injury. Mucous membrane bleeding is a frequent
manifestation. The severity of bleeding is variable and most severe among
homozygous persons who may bleed from the GI tract. Hemarthrosis is rare. The
release of von Willebrand’s factor in the classic form of the disease can be stimulated
by desmopressin.
o Defective Activation and secretion results in problems with impaired production or
an impaired response to prostaglandin-dependent activators such as thromboxane A2,
which attracts platelets and constricts vessels. NSAIDS affect platelet activation and
secretion by inhibiting cyclooxygenase which helps convert arachidonic acid to
thrombaxane. Irreversible inhibition of cyclooxygenase occurs with even small doses
of aspirin and persists for the life span of the platelet (7 days). Reversible inhibition
is seen with the use of other NSAIDS and omega-3 fatty acids (found in oily fish).
Severe bleeding does not result, but an underlying bleeding diathesis may be
aggravated.
o Defective Aggregation occurs in patients with a rare hereditary defect in platelet
aggregation. They are missing a bridging protein called glycoprotein IIb/IIIa.
o Defective Acceleration of Coagulation occurs when platelets aren’t able to bind
factors V and X on their surface. Those clotting factors normally greatly accelerate
the rate of prothrombin conversion. These patients have a mildly prolonged
prothrombin time, normal bleeding time, and normal platelet aggregation.
Anonymous Goroll, Chap 115
Thrombocytopenia- A condition of decreased platelets. Normal is 150,000-350,000/microlitre.
Mechanism is either inadequate production of platelets, or destruction of platelets, caused by
many different drugs or disease states. Prolonged bleeding times when count reaches 50000-
75000, risk of spontaneous hemorrhage when counts reach 10000-20000. Physical signs to
include petechiae, especially in dependent extremities, ecchymoses, menorrhagia, melena,
epistaxis and gingival bleeding.
Platelet dysfunction- Platelet reactions consist of 3 physiologic processes: 1. Adhesion to
subendothelial collagen. 2. Activation, shape change, and secretion of a number of substances. 3.
Aggregation between adjacent activated platelets. Deficiencies in platelet membrane receptors or
on Von Willebrands factor can lead to lack of adhesion, and platelet dysfunction. Platelet defects
usually lead to petechiae, whereas coagulation defects usually lead to ecchymoses.
Thrombotic thrombocytopenic purpura- A severe thrombocytopenia. This is a primary
disorder of platelet consumption. TTP syndrome consists of thrombocytopenia, microangiopathic
hemolytic anemia, transient neurological deficits, renal impairment and fever. Both TTP and
DIC are life-threatening emergencies and require hematologic consultation. TTP responds to
fresh frozen plasma, plasmapheresis. Also, possible splenectomy and high dose adrenal
corticosteroids.
Anonymous Noble p 1047
Thrombocytopenia- reduction in the number of platelets, due to increased destruction,
decreased production, or decreased survival.

A) Thrombotic Thrombocytopenic Purpura (TTP)- this is a primary disorder of platelet


consumption. This is characterized by thrombocytopenia, mocroangiopathic hemolytic anemia,
often transient neurologic deficits, renal impairement, and fever.

B) Heparin Induced Thrombocytopenia (HIT)- this is the most common cause of drug-
induced thrombocytopenia usually occurring after 6-8 days of intravenous heparin therapy,
although it may occur sooner.

C) Idiopathic (immune) Thrombocytopenia Purpura (ITP)- this is the most common corm
of immumologic thrombocytopenia. It occurs in acute (often in children) and chronic (often in
adults) forms resulting in an immune mediated destruction of platelets. In 90% of cases these
have an acute onset often preceeded by a viral illness and resolve on their own over the course of
1-3 months.

D) Qualitative Platelet Disorders- Functional platelet defects produce a long bleeding time in
the presence of a normal platelet count and may predispose to bleeding. Inherited disorders are
uncommon with von Willebrand's Disease being one, acquired platelet disorders are much more
common.
1. von Willebrand's Disease- a disease characterized by a defect in von Willebrand's
factor which is a component of clotting Factor VIII necessary for normal platelet adhesion.
2. Acquired Aspirin Induced- aspirin impair platelet prostaglandin production
irreversibly
3. Acquired NSAID Induced- other NSAIDs impair prostaglandin production
reversible
4. Uremia- produces a qualitative defect sometimes improved by dialysis
5. Myeloproliferative Disorders- can be associated with a platelet defect resulting from
an abnormal stem cell.

B) Platelet Dysfunction: I don't know what they want here. The question is pretty vague and
all these prior are platelet dysfunctions. I guess pay attention in class. Books do not really have
anything that jumps out at me,

3. Describe typical signs or symptoms and general approach for each of the
following bleeding disorders:
a. Factor VIII deficiency
b. Vitamin K deficiency
c. Liver disease
d. DIC
Brent CMDT 512-16
a. Factor VIII deficiency (Hemophilia A)- Most common severe bleeding disorder and 2nd only
to von Willbrand’s disease as the most common congenital disorder. Bleeding may occur
anywhere. The most common sites of bleeding are into joints, muscles, and from the GI tract.
Spontaneous hemarthroses are virtually diagnostic of thisdisorder. Treatment Is the infusion of
factor VIII concentrates.
b. Vit K deficiency- No specific clinical features and bleeding may occur at any site. Treatment
includes a single dose of 15mg SQ Vit K. Labs will normalize within12-24hrs.
c. Liver disease- bleeding at any site. Excessive fibrinolysis may lead to oozing at venipuncture
sites. Most liver disease is clinically obvious. Fresh frozen plasma is
the treatment of choice.
d. Disseminated Intravascular Coagulation (DIC)- bleeding and thrombosis with bleeding the
more common. Bleeding at any site. Thrombosis causing digital ischemia and gangrene but
renal cortical necrosis and hemorrhagic adrenal infection. DIC may also secondarily produce
microangiopathic hemolytic anemia. Treatment is focused on the underlying cause of DIC.
Although controversial, when DIC is producing serious clinical consequences and the underlying
cause is not rapidly reversible. Heparin in combination with replacement therapy (platelets) can
be used.
jam, CMDT 512-514, Noble on-line 1048
a. Factor VIII deficiency CMDT 512-514
• Hereditary form is hemophilia A. Common sites of bleeding are into joints, muscles, and
from GI tract. Patients with mild hemophilia bleed only after major trauma or surgery;
severe disease bleeds spontaneously. PTT will be prolonged; PT, bleeding time, and
fibrinogen level will be normal. Standard treatment is infusion of factor VIII
concentrates.
• Acquired factor VIII antibodies produce a severe bleeding disorder. PTT is prolonged;
fibrinogen, PT, and plt count are normal. Treatment is with cyclophosphamide, usually
combined with prednisone. In the interim, aggressive factor VIII replacement may be
necessary.
b. Vitamin K deficiency - Noble 1048 Vitamin K deficiency is most often seen in seriously ill
patients, especially in the postoperative state when patients are poorly nourished and receiving
antibiotics. Poor nutrition removes vitamin K from the diet, and antibiotics suppress gut bacteria
that produce vitamin K. Vitamin K deficiency produces an initial rise in the prothrombin time
because of the rapid decline of factor VII activity, the vitamin K-dependent factor with the
shortest metabolic half-life. An elevation of the activated partial thromboplastin time follows as
other vitamin K-dependent factors decline (factors IX, X, and II). This condition, which often
leads to serious bleeding, is easily reversed by parenteral vitamin K1 therapy (5 to 10 mg
subcutaneous).
c. Liver disease - Noble 1048 Hemostatic failure in liver disease involves both the platelet and
coagulation phases of clotting. Mild thrombocytopenia is frequently encountered because of
hypersplenism that accompanies portal hypertension. Qualitative defects in platelet function are
probably not a major factor.
Coagulation is impaired primarily because of decreased factor synthesis, but abnormal factors
may be produced, excessive consumption can occur, and fibrinolysis may be enhanced as
contributing causes of hemostatic failure. A low fibrinogen, one of the last factors to be reduced,
is a poor prognostic sign.
Treatment depends on the severity of the coagulopathy and the presence of bleeding and usually
includes FFP. Treatment simply for the purpose of correcting an abnormal PT and aPTT,
however, is not recommended, since it takes a large volume of plasma to correct the abnormality,
the correction is short lived, and the protein load contained in the plasma may be enough to
induce hepatic encephalopathy in a patient who is so predisposed. Platelet transfusions can be
given in the face of clinically important bleeding with a very low platelet count, but generally
they are not indicated in hypersplenism.
d. DIC – Noble 1048 DIC often occurs in critically ill patients and is common in the intensive
care setting. However, DIC can also occur in relatively well patients, a result of certain
underlying diseases such as malignancy. Its onset can be fulminant and rapidly fatal or can be
more subtle and gradual. Although its name implies a disorder of intravascular clotting, its
clinical expression is often one of a diffuse hemorrhagic disorder.
DIC involves the pathologic activation of coagulation by an underlying disease process that leads
to fibrin clot formation and secondary fibrinolysis, which then cause further consumption of
coagulation factors, platelets, and red cells. In the fulminant syndrome bleeding results from
factor deficiency (primarily factors I, II, V, VIII, and XIII), thrombocytopenia, excessive
fibrinolysis, and high levels of FDP superimposed on a vascular system already damaged by
diffuse microvascular thrombi. Bleeding is typically manifested by diffuse superficial
hemorrhage in the form of ecchymoses and petechiae as well as oozing from the gingiva, the oral
mucosa, or the gastrointestinal and urinary tracts. Most hemorrhage tends to be from the
microvasculature, although major vascular hemorrhage and central nervous system bleeding can
occur.
The most important component in the treatment of DIC is correction of the underlying disease.
Supportive measures include FFP and platelet transfusions. Recent studies suggest a benefit to
infusions of antithrombin concentrates, but it is too premature to recommend their use. In rare
cases low doses of intravenous or subcutaneous heparin may be useful to interrupt the process of
consumption by neutralizing activated coagulation factors. This may be most helpful in well-
characterized conditions such as acute promyelocytic leukemia associated with a high incidence
of DIC. Efficacy of therapy can be monitored by looking for a decrease in FDP, or D-dimer, an
increase in fibrinogen, or the normalization of the PT and aPTT.
Anonymous
• Factor VIII deficiency: It is otherwise known as hemophilia A which is a sex-linked
recessive trait. It exists as varying degrees of deficiency that roughly correlate with
clinical severity. Severe (classic) hemophiliacs have 1% or less of normal factor VIII:C
activity levels on assay, moderate hemophiliacs have 2-5%, and mild hemophiliacs have
5-25% of normal levels. Hemophilia with levels between 25-50% of normal is
sometimes called subhemophilia. Most carrier females have about 50% of normal factor
VIII:C activity. The major symptom of hemophilia is excessive bleeding. Most bleeding
episodes follow trauma. In classic cases, the trauma may be very slight. In mild cases,
the trauma must usually be more significant such as a surgical operation or dental
extraction. Bleeding into joints (hemarthrosis) is a characteristic finding in classic and
moderate factor VIII:C activity deficiency. Bleeding from the mouth, urinary tract, and
GI tract and intracranial hemorrhage are also relatively common in severe cases. Blood
product transfusion is the treatment. The biologic half-life of factor VIII:C activity is
approximately 8-12 hours. Factor VIII:C activity is unstable, and in freshly drawn and
anticoagulated blood stored in the refrigerator the activity decreases to 40% by 24 hours.
The current therapeutic sources of factor VIII:C include fresh frozen plasma, factor
VIII:C concentrate, and cryoprecipitate. As a rule of thumb, one factor VIII unit in
therapeutic material per kilogram of body weight should increase factor VIII activity
levels by 2% of normal with a half-life of 12 hours. Ravel 93
• Vitamin K deficiency: The major clotting factors of the extrinsic pathway (II, VII, X)
depend for their synthesis and modification on a healthy liver and an adequate dietary
intake of vitamin K. Some vitamin K also derives from bacterial production by gut
flora. Hereditary deficiencies of extrinsic pathway factors are rare. Most bleeding traced
to the extrinsic pathway is the consequence of impaired vitamin K production or liver
disease. Causes include hepatocellular insufficiency, cholestasis (which impairs
absorption of lipids-soluble vitamin K), poor dietary intake, and use of broad-spectrum
antibiotics that kill normal gut flora. The characteristic lab findings is a prolongation of
the PT. Prolongation of the PT also occurs with warfarin anticoagulant therapy.
Warfarin inhibits the vitamin K-dependent postsynthetic modification of factors II, VII,
IX, and X; this prevents them from being able to bind calcium and achieve biologic
activity. Serious bleeding problems can result due to vitamin K deficiency.
Administration of fresh-frozen plasma provides rapid correction of the INR.
Administration of vitamin K is also effective but somewhat slower taking 12-48 hours to
reduce the INR to a safe level. IV, SQ, and oral preparations are available. High dose
IV vitamin K (1-10 mg) is the most potent and most rapidly acting of the various
formulations. Goroll 525, 538
• Liver Disease: Impairment of the synthetic capacity of the liver is one of the most
common causes for an acquired hemostatic defect. Hemostatic failure usually reflects
the degree of liver failure and is usually subtle in acute liver failure unless the destruction
of parenchymal tissue is fulminant. Hemostatic failure in liver disease involves both the
platelet and coagulation phases of clotting. Mild thrombocytopenia is frequently
encountered because of hypersplenism that accompanies portal hypertension.
Coagulation is impaired primarily because of decreased factor synthesis, but abnormal
factors may be produced, excessive consumption can occur, and fibrinolysis may be
enhanced as contributing causes of hemostatic failure. Treatment depends on the
severity of the coagulopathy and the presence of bleeding and usually includes fresh
frozen plasma. Treatment simply for the purpose of correcting an abnormal PT and
aPTT, however, in not recommended, since it takes a large volume of plasma to correct
the abnormality, the correction is short lived, and the protein load contained in the
plasma may be enough to induce hepatic encephalopathy in a patient who is
predisposed. Platelet transfusions can be given in the face of clinically important
bleeding with a very low platelet count, but generally they are not indicated in
hypersplenism. Noble 1048
Disseminated Intravascular Coagulation: DIC occurs is critically ill patients and is common
in the intensive care setting. It involves the pathologic activation of coagulation by an
underlying disease process that leads to fibrin clot formation and secondary fibrinolysis, which
then can cause further consumption of coagulation factors, platelets, and red cells. In the
fulminant syndrome bleeding results from factor deficiency (primarily factors I, II, V, VIII, and
XIII), thrombocytopenia, excessive fibrinolysis, and high levels of FDP superimposed on a
vascular system already damaged by diffuse microvascular thrombi. Bleeding is typically
manifested by diffuse superficial hemorrhage in the form of ecchymoses and petechiae as well as
oozing from the gingival,
• the oral mucosa, or the GI and urinary tracts. Most hemorrhage tends to be from the
microvasculature, although major vascular hemorrhage and CNS bleeding can occur.
The most important component in the treatment of DIC is correction of the underlying
disease. Supportive measures include fresh frozen plasma and platelet transfusions.
Efficacy of therapy can be monitored by looking for a decrease in FDP, or D-dimer, in
increase in fibrinogen, or the normalization of the PT and a PTT. Noble 1048
Anonymous Goroll, pp 529-530. Noble pp 1030, 1048-1052
Disease Signs/Symptoms Labs General Approach
Factor Cardinal sign of a platelet Detection difficult if factor Administration of
VIII defect is presence of levels >25%, as PTT will be Factor VIII concentrate
Deficiency petechiae. Coalesced normal. Chorionic villi is the mainstay of
petechiae are referred to as analysis in utero can detect. therapy. Basic
purpura. Ecchymoses is management includes
usually indicative of a specifying guidelines
coagulation disorder. for permissible
Platelet-related bleeding physical activity and
also tends to occur in the teaching proper first
oral mucosa or GI tract aid. If the degree of
and bleeding tends to be bleeding has been only
immediate in onset since mild to moderate, then
platelets are responsible one can encourage
for the primary hemostatic participation in
plug. Can also present as noncontact sports and
delayed-onset bleeding other activities that
and bleeding into entail little risk of
intramuscular tissues and injury. Goal is to allow
joints, as well as skin and as much normal
GI tract. activity as possible.
First aid treatment of
an acute hemarthrosis
should be learned by
the family. Immobilize
the joint and apply ice
packs to reduce pain
and swelling. Splinting
and elastic bandages
can help ensure that a
position of good joint
function is maintained.
Pain control is
important.
ASA/NSAIDS must be
avoided. Tylenol and
codeine work well
when given in
adequate doses for
short periods of time.
Genetic counseling is
also an important
component of care.
Vitamin K Same as above, but in the Rapid initial rise in PT. Easily reversed with
Deficiency presence of a concurrent Elevated aPTT follows as parenteral vitamin K1
serious illness other vitamin K-dependent therapy (5-10mg SQ)
factors decline.
Liver Same as above, but in the PT is prognostically helpful. Tx depends on severity
Disease presence of liver of coagulopathy and
dysfunction the presence of
bleeding and usually
includes FFP. Tx
simply to correct
abnormal PT and aPTT
not recommended.
Platelet transfusing
indicated in face of
clinically important
bleeding with a very
low platelet count, but
generally not indicated
for hypersplenism.
DIC Generally occurs in the Blood smear +/- in low- The most important
presence of serious illness, grade, + in fulminant. component of
but can occur in relatively Platelets low normal to low treatment is correction
well patients w/ certain in low-grade, very low in of underlying disease.
underlying diseases such fulminant. PT, PTT normal Supportive Tx include
as malignancy. In the in low-grade, long in FFP and platelet
fulminant syndrome, fulminant. TT transfusions. Efficacy
bleeding results from normal/short/long in low- of Tx can be monitored
factor deficiency, grade, long in fulminant. by looking for decrease
thrombocytopenia, Fibrinogen in FDP, or D-dimer an
excessive fibrinolysis and elevated/normal/low normal increase in fibrinogen
high levels of FDP in low-grade, low in or the normalization of
superimposed on a fulminant. Fibrin monomers the PT and aPTT
vascular system already +/- in low-grade, + in
damaged by diffuse fulminant. D-dimer mildly
microvascular thrombi. elevated in low-grade, high
Bleeding is typically in fulminant.
manifested by diffuse
superficial hemorrhage in
the form of ecchymoses
and petechiae as well as
oozing from the gingiva,
oral mucosa or GI/Urinary
tracts. Most hemorrhage
tends to be from the
microvasculature,
although major vascular
hemorrhage and central
nervous system bleeding
can occur.
Anonymous Noble p 1047-1048
A) Factor VIII Deficiency- von Willebrand's can cause a factor VIII deficiency. Von
Willebrand's factor has a platelet function of helping platelets to adhere to the subendothelium of
vessels, however it also has a protective job of binding to Factor VIII and protecting it from
degradation. Factor VIII protein is coded for by a gene on the X chromosome and is the protein
deficient in classic hemophilia.

B) Vitamin K Deficiency- This is most commonly seen in seriously ill patients, especially in
the postoperative state where patients are poorly nourished and receiving antibiotics. Poor
nutrition removes vitamin K from the body and antibiotics kill gut bacteria that produce vitamin
K. Vitamin K is necessary for the production of Factor VII and a decline in this is evidenced by
an elevation in the prothrombin time.

C) Liver Disease- Impairement of the synthetic capacity of the liver is one of the most common
causes for an acquired hemostatic defect. Liver disease involves both the platelet and
coagulation phases of clotting. Coagulation is a problem mainly due to decreased factor
production and thrombocytopenia is a problem due to hypersplenism and portal hypertension that
increase destruction of platelets.

D) Disseminated Intravascular Coagulation (DIC)- This commonly occurs in critically ill


patients frequently in intensive care patients. DIC involves the pathologic activation of
coagulation by an underlying disease process that leads to fibrin clot formation and secondary
fibrinolysis, which then causes further consumption of coagulation factors, platelets, and red
cells. In fulminent cases bleeding is manifested due to the factor deficiency.

4. Identify the following treatment modalities for DVTs:


a. Heparin
b. Low molecular Weight Heparin
c. Warfarin
d. Fibrinolytics
e. Antiplatelet agents
Brent CMDT 460
a. Heparin- prevention 5000 units SQ bid. For DVT, initial bolous 100u/kg followed by 10
u/kg/h dosed until PTT is 1.5-2x’s normal.
b. Low-molecular-weight heparin (LMWH) or enoxaparin. Prevention 30mg SQ bid.1 mg/kg
SQ bid.
c. Warfarin- initiated after the heparinization. The 2 therapies should overlap. 1st episode of
DVT, warfarin is recommended for 3-6mos while maintaining an INR of 2.0-3.0. After 2nd
DVT, warfarin is continued indefinitely.
d. Fibrinolytics (alteplase, rreleplase, urokinase, streptokinase)- to be effective should be
administered within 1wk after clot formation.
e. Antiplatelet agents- ASA and ticlopidine,
jam, CMDT 460, Noble 735
a. Heparin – initial bolus 100units/kg followed by 10units/kg/h, dosed to goal PTT (partial
thromboplastin time)of 1.5-2 times normal. This reduces risk of pulmonary embolism and
decreases rate of thrombophlebitis recurrence by 80%
b. Low molecular Weight Heparin - enoxaparin, 30mg subQ BID. LMWH is used
prophylactically in orthopedic surgery patients and is associated with lower risk of
bleeding complications. Noble 735 on-line does not require follow-up with lab tests and
may be used on outpatient basis
c. Warfarin – started after therapeutic heparinization. The 2 therapies should overlap to
diminish the possibility of a hypercoagulable state. For first episode of uncomplicated
DVT, 3-6 months warfarin to maintain goal INR of 2.0-3.0. After second episode,
warfarin is continued indefinitely
d. Fibrinolytics – example is alteplase; faster clot lysis and increased venous patency
compared to heparin but risk of bleeding complications is higher. Alteplase should be
started within 1 week after clot formation before extensive fibrin cross-linking can occur.
Noble 735 tissue plasminogen activator, streptokinase, and urokinase are approved for
treatment of pulmonary embolism, but using these in patients with DVT but no PE is
controversial
e. Antiplatelet agents – I read over CMDT, Goroll, and Noble and could find no reference
for using these drugs in DVT or PE. According to Goroll 539-540, anti-platelet drugs are
used in primary and secondary prevention of MI, angina, stroke and other arterial/cardiac
problems. My assumption is that these agents aren’t used in treating DVT/PE.
Anonymous Noble Pg 734-735 Goroll Pg 247-248
a. Heparin – remains the initial treatment of choice for DVT, because it immediately
inhibits thrombus formation. Therapy is usually adjusted to maintain the activated partial
thromboplastin time (PTT) in a therapeutic range (46-70 seconds). Initial dose of
80U/Kg as IV bolus, then 18 U/Kg/hr IV. Check every 6 hrs and adjust accordingly until
therapeutic range is met, and thereafter it is checked daily. Heparin therapy is stopped
after the INR has reached at least 2.0 for 24 to 48 hours. (INR therapeutic range is 2.0 to
3.0)
b. Low molecular Weight Heparin- is shown to be at least as effective as unfractionated
heparin in preventing thromboembolic events. The efficacy, safety, and ease of use of
twice a day dosing, makes LMWH the emerging drug of choice for OUTPATIENT
therapy. TX: consists of 1 mg/Kg SQ every 12 hrs or 1.5 mg/Kg SQ every day
c. Warfarin – is usually started simultaneously with heparin therapy upon hospitalization
for DVT, so that in 3-5 days the INR is within the therapeutic range. Warfarin
(Coumadin) therapy is continued for 3 to 5 months post DVT.
d. Fibrinolytics-Streptokinase, tissue plasminogen activator, is considered for patients with
proximal DVT’s, especially in the illiofemoral system. Treatment should begin within
three days of onset of symptoms.
e. Antiplatelet agents- long-term aspirin therapy.
Anonymous Noble, pp 1050-1052
a.Heparin- Initial treatment of choice for DVT and PE, unless specific contraindications exist,
because it immediately inhibits thrombus formation. Although thrombus formation is arrested,
heparin does not prevent embolization of established thrombus. Target PTT is 1.5 to 2.5 times
control. Major complication is hemorrhage.
b.Low molecular Weight Heparin- approved for use in orthopedic DVT prophylaxis. Alternate
treatment to unfractionated heparin in DVT and PE. Major advantages are no IV administration,
no PTT monitoring and ease of outpatient therapy.
c.Warfarin- Drug of choice for continued anticoagulation following DVT/PE or chronic
anticoagulation. Initiated simultaneously with heparin and continued for 3-6 months. Dosage
adjusted to maintain INR of 2 to 2.5. Major complication is hemorrhage.
d.Fibrinolytics- aka Thrombolytics- Streptokinase, urokinase, t-PA and anisoylated
plasminogen-streptokinase activator complex. Recommended in fulminant PE and some cases of
DVT, but because of a high-risk profile they should be used with great care and in those with no
risk factors for bleeding, recent CVA or intracranial/intraspinal surgery, intracranial neoplasm,
arteriovenous malformation or aneurysm, severe uncontrolled hypertension or a known bleeding
diathesis. Therapy does not require close monitoring of coagulation parameters, since fixed
dosage is given in most cases.
e.Antiplatelet agents- ASA and most NSAIDS- inhibit platelet aggregation by inhibiting
cyclooxygenase (ASA irreversibly, NSAIDS reversibly). Small doses of ASA sufficient to effect
most circulating platelets. Effect from ASA lasts for lifetime of affected platelet. Ticlopidine and
Clopidogrel- similarly acting antiplatelet agents that mediate their effect by inhibiting the
adenosine diphosphate pathway for platelet activation. Onset of action is slow, w/ cumulative
effect over 8-10 days. Limited use due to serious side effects, including neutropenia and
syndrome of thrombotic thrombocytopenic purpura. Can be a substitute for patients unable to
take aspirin.
Anonymous Currents p 460/ Noble p 1050-1051
A) Heparin- The standard treatment for DVTs is systemic anticoagulation with heparin. The
goal is a thromboblastin time of 1.5 to 2 times longer than normal which reduces the risk of
pulmonary embolism and decreases the rate of thrombophlebitis recurrence by 80%

B) Low molecular weight heparin (LMWHs)- These are advantageous compared with
standard heparin in their greater ability to neutralize factor Xa compared with thrombin. They
have less binding to heparin-binding proteins and platelets, have a longer half-life, are more
uniformly absorbed from subcutaneous depots, and cause less heparin-induced
thrombocytopenia.

C) Warfarin- This is the most common oral anticoagulant used in patients requiring long term
anticoagulation. Warfarin exerts its anticoagulant effect through competitive interference with
vitamin K, which is essential for the normal synthesis of factors II, VII, IX, and X as well as the
antithrombotic factors protein S and protein C.
D) Fibrinolytics- Major ones used are streptokinase, urokinase, t-PA, and anisoylated
plasminogen-streptokinase. These agents are recommended in fulminant pulmonary embolism
and in some cases of deep vein thrombosis but they have a large risk profile and are entirely
contraindicated in any situation where bleeding is a concern. These are used for the clearance of
thrombi that obstruct indwelling catheters such as those used in cancer patients.

E) Antiplatelet Agents- Aspirin and most other NSAIDs inhibit platelet aggregation by
interfering with cyclooxygenase, an enzyme important in the generation of prostaglandins.
Aspirin irreversibly inhibits cyclooxygenase whereas other NSAIDs reversible inhibit it. Only
small amounts of Aspirin are needed to affect the majority of platelets in the system and this
effect lasts for the duration of the platelets life cycle of approximately 10 days.

Das könnte Ihnen auch gefallen